You are on page 1of 164

Introduction

A 24-item NCLEX style exam all about the ADPIE or the Nursing Process.

Topics

Nursing Process

Guidelines
Read each question carefully and choose the best answer. You are given one minute per question. Spend your time wisely! Answers and rationales are given below. Be sure to read them. If you need more clarifications, please direct them to the comments section.

Questions
1. Once a nurse assesses a clients condition and identifies appropriate nursing diagnoses, a: 1. Plan is developed for nursing care. 2. Physical assessment begins 3. List of priorities is determined. 4. Review of the assessment is conducted with other team members. 2. Planning is a category of nursing behaviors in which: 1. The nurse determines the health care needed for the client. 2. The Physician determines the plan of care for the client. 3. Client-centered goals and expected outcomes are established. 4. The client determines the care needed. 3. Priorities are established to help the nurse anticipate and sequence nursing interventions when a client has multiple problems or alterations. Priorities are determined by the clients: 1. Physician 2. Non Emergent, non-life threatening needs 3. Future well-being. 4. Urgency of problems 4. A client centered goal is a specific and measurable behavior or response that reflects a clients: 1. Desire for specific health care interventions 2. Highest possible level of wellness and independence in function. 3. Physicians goal for the specific client. 4. Response when compared to another client with a like problem. 5. For clients to participate in goal setting, they should be: 1. Alert and have some degree of independence. 2. Ambulatory and mobile. 3. Able to speak and write. 4. Able to read and write. 6. The nurse writes an expected outcome statement in measurable terms. An example is: 1. Client will have less pain. 2. Client will be pain free. 3. Client will report pain acuity less than 4 on a scale of 0-10. 4. Client will take pain medication every 4 hours around the clock. 7. As goals, outcomes, and interventions are developed, the nurse must: 1. Be in charge of all care and planning for the client. 2. Be aware of and committed to accepted standards of practice from nursing and other disciples.

3. Not change the plan of care for the client. 4. Be in control of all interventions for the client. 8. When establishing realistic goals, the nurse: 1. Bases the goals on the nurses personal knowledge. 2. Knows the resources of the health care facility, family, and the client. 3. Must have a client who is physically and emotionally stable. 4. Must have the clients cooperation. 9. To initiate an intervention the nurse must be competent in three areas, which include: 1. Knowledge, function, and specific skills 2. Experience, advanced education, and skills. 3. Skills, finances, and leadership. 4. Leadership, autonomy, and skills. 10. Collaborative interventions are therapies that require: 1. Physician and nurse interventions. 2. Nurse and client interventions. 3. Client and Physician intervention. 4. Multiple health care professionals. 11. Well formulated, client-centered goals should: 1. Meet immediate client needs. 2. Include preventative health care. 3. Include rehabilitation needs. 4. All of the above. 12. The following statement appears on the nursing care plan for an immunosuppressed client: The client will remain free from infection throughout hospitalization. This statement is an example of a (an): 1. Nursing diagnosis 2. Short-term goal 3. Long-term goal 4. Expected outcome 13. The following statements appear on a nursing care plan for a client after a mastectomy: Incision site approximated; absence of drainage or prolonged erythema at incision site; and client remains afebrile. These statements are examples of: 1. Nursing interventions 2. Short-term goals 3. Long-term goals 4. Expected outcomes. 14. The planning step of the nursing process includes which of the following activities? 1. Assessing and diagnosing 2. Evaluating goal achievement. 3. Performing nursing actions and documenting them. 4. Setting goals and selecting interventions. 15. The nursing care plan is: 1. A written guideline for implementation and evaluation. 2. A documentation of client care. 3. A projection of potential alterations in client behaviors 4. A tool to set goals and project outcomes. 16. After determining a nursing diagnosis of acute pain, the nurse develops the following appropriate client-centered goal: 1. Encourage client to implement guided imagery when pain begins. 2. Determine effect of pain intensity on client function.

3. Administer analgesic 30 minutes before physical therapy treatment. 4. Pain intensity reported as a 3 or less during hospital stay. 17. When developing a nursing care plan for a client with a fractured right tibia, the nurse includes in the plan of care independent nursing interventions, including: 1. Apply a cold pack to the tibia. 2. Elevate the leg 5 inches above the heart. 3. Perform range of motion to right leg every 4 hours. 4. Administer aspirin 325 mg every 4 hours as needed. 18. Which of the following nursing interventions are written correctly? (Select all that apply.) 1. Apply continuous passive motion machine during day. 2. Perform neurovascular checks. 3. Elevate head of bed 30 degrees before meals. 4. Change dressing once a shift. 19. A clients wound is not healing and appears to be worsening with the current treatment. The nurse first considers: 1. Notifying the physician. 2. Calling the wound care nurse 3. Changing the wound care treatment. 4. Consulting with another nurse. 20. When calling the nurse consultant about a difficult client-centered problem, the primary nurse is sure to report the following: 1. Length of time the current treatment has been in place. 2. The spouses reaction to the clients dressing change. 3. Clients concern about the current treatment. 4. Physicians reluctance to change the current treatment plan. 21. The primary nurse asked a clinical nurse specialist (CNS) to consult on a difficult nursing problem. The primary nurse is obligated to: 1. Implement the specialists recommendations. 2. Report the recommendations to the primary physician. 3. Clarify the suggestions with the client and family members. 4. Discuss and review advised strategies with CNS. 22. After assessing the client, the nurse formulates the following diagnoses. Place them in order of priority, with the most important (classified as high) listed first. 1. Constipation 2. Anticipated grieving 3. Ineffective airway clearance 4. Ineffective tissue perfusion. 23. The nurse is reviewing the critical paths of the clients on the nursing unit. In performing a variance analysis, which of the following would indicate the need for further action and analysis? 1. A clients family attending a diabetic teaching session. 2. Canceling physical therapy sessions on the weekend. 3. Normal VS and absence of wound infection in a post-op client. 4. A client demonstrating accurate medication administration following teaching. 24. The RN has received her client assignment for the day-shift. After making the initial rounds and assessing the clients, which client would the RN need to develop a care plan first? 1. A client who is ambulatory. 2. A client, who has a fever, is diaphoretic and restless.

3. A client scheduled for OT at 1300. 4. A client who just had an appendectomy and has just received pain medication.

Answers and Rationale


1. 2. 3. 4. 5. 6. 7. 8. 9. A B D B A C B B A

10. D 11. D 12. B 13. D 14. D 15. A 16. D. This is measurable and objective. 17. B. This does not require a physicians order. (A & D require an order; C is not appropriate for a fractured tibia) 18. C. It is specific in what to do and when. 19. B. Calling in the wound care nurse as a consultant is appropriate because he or she is a specialist in the area of wound management. Professional and competent nurses recognize limitations and seek appropriate consultation. (a. This might be appropriate after deciding on a plan of action with the wound care nurse specialist. The nurse may need to obtain orders for special wound care products. c. Unless the nurse is knowledgeable in wound management, this could delay wound healing. Also, the current wound management plan could have been ordered by the physician. d. Another nurse most likely will not be knowledgeable about wounds, and the primary nurse would know the history of the wound management plan.) 20. A. This gives the consulting nurse facts that will influence a new plan. (b, c, and d. These are all subjective and emotional issues/conclusions about the current treatment plan and may cause a bias in the decision of a new treatment plan by the nurse consultant.) 21. D. Because the primary nurse requested the consultation, it is important that they communicate and discuss recommendations. The primary nurse can then accept or reject the CNS recommendations. (a. Some of the recommendations may not be appropriate for this client. The primary nurse would know this information. A consultation requires review of the recommendations, but not immediate implementation. b. This would be appropriate after first talking with the CNS about recommended changes in the plan of care and the rationale. Then the primary nurse should call the physician. c. The client and family do not have the knowledge to determine whether new strategies are appropriate or not. Better to wait until the new plan of care is agreed upon by the primary nurse and physician before talking with the client and/or family.) 22. C, D, A, B. 23. B. 24. B. This clients needs are a priority.

Introduction
An NCLEX practice exam dedicated to Coronary Artery Disease and Hypertension. This exam contains 50 items about the two diseases.

Topics

Coronary Artery Disease Hypertension

Guidelines
Read each question carefully and choose the best answer. You are given one minute per question. Spend your time wisely! Answers and rationales are given below. Be sure to read them. If you need more clarifications, please direct them to the comments section.

Questions
1. A client is scheduled for a cardiac catheterization using a radiopaque dye. Which of the following assessments is most critical before the procedure? 1. Intake and output 2. Baseline peripheral pulse rates 3. Height and weight 4. Allergy to iodine or shellfish 2. A client with no history of cardiovascular disease comes into the ambulatory clinic with flu-like symptoms. The client suddenly complains of chest pain. Which of the following questions would best help a nurse to discriminate pain caused by a non-cardiac problem? 1. Have you ever had this pain before? 2. Can you describe the pain to me? 3. Does the pain get worse when you breathe in? 4. Can you rate the pain on a scale of 1-10, with 10 being the worst? 3. A client with myocardial infarction has been transferred from a coronary care unit to a general medical unit with cardiac monitoring via telemetry. A nurse plans to allow for which of the following client activities? 1. Strict bed rest for 24 hours after transfer 2. Bathroom privileges and self-care activities 3. Unsupervised hallway ambulation with distances under 200 feet 4. Ad lib activities because the client is monitored. 4. A nurse notes 2+ bilateral edema in the lower extremities of a client with myocardial infarction who was admitted 2 days ago. The nurse would plan to do which of the following next? 1. Review the intake and output records for the last 2 days 2. Change the time of diuretic administration from morning to evening 3. Request a sodium restriction of 1 g/day from the physician. 4. Order daily weights starting the following morning. 5. A client is wearing a continuous cardiac monitor, which begins to sound its alarm. A nurse sees no electrocardiogram complexes on the screen. The first action of the nurse is to: 1. Check the client status and lead placement 2. Press the recorder button on the electrocardiogram console. 3. Call the physician 4. Call a code blue

6. A nurse is assessing the blood pressure of a client diagnosed with primary hypertension. The nurse ensures accurate measurement by avoiding which of the following? 1. Seating the client with arm bared, supported, and at heart level. 2. Measuring the blood pressure after the client has been seated quietly for 5 minutes. 3. Using a cuff with a rubber bladder that encircles at least 80% of the limb. 4. Taking a blood pressure within 15 minutes after nicotine or caffeine ingestion. 7. IV heparin therapy is ordered for a client. While implementing this order, a nurse ensures that which of the following medications is available on the nursing unit? 1. Vitamin K 2. Aminocaproic acid 3. Potassium chloride 4. Protamine sulfate 8. A client is at risk for pulmonary embolism and is on anticoagulant therapy with warfarin (Coumadin). The clients prothrombin time is 20 seconds, with a control of 11 seconds. The nurse assesses that this result is: 1. The same as the clients own baseline level 2. Lower than the needed therapeutic level 3. Within the therapeutic range 4. Higher than the therapeutic range 9. A client who has been receiving heparin therapy also is started on warfarin. The client asks a nurse why both medications are being administered. In formulating a response, the nurse incorporates the understanding that warfarin: 1. Stimulates the breakdown of specific clotting factors by the liver, and it takes 2-3 days for this to exert an anticoagulant effect. 2. Inhibits synthesis of specific clotting factors in the liver, and it takes 3-4 days for this medication to exert an anticoagulant effect. 3. Stimulates production of the bodys own thrombolytic substances, but it takes 2 -4 days for this to begin. 4. Has the same mechanism of action as Heparin, and the crossover time is needed for the serum level of warfarin to be therapeutic. 10. A 60-year-old male client comes into the emergency department with complaints of crushing chest pain that radiates to his shoulder and left arm. The admitting diagnosis is acute myocardial infarction. Immediate admission orders include oxygen by NC at 4L/minute, blood work, chest x-ray, an ECG, and 2 mg of morphine given intravenously. The nurse should first: 1. Administer the morphine 2. Obtain a 12-lead ECG 3. Obtain the lab work 4. Order the chest x-ray 11. When administered a thrombolytic drug to the client experiencing an MI, the nurse explains to him that the purpose of this drug is to: 1. Help keep him well hydrated 2. Dissolve clots he may have 3. Prevent kidney failure 4. Treat potential cardiac arrhythmias. 12. When interpreting an ECG, the nurse would keep in mind which of the following about the P wave? Select all that apply. 1. Reflects electrical impulse beginning at the SA node 2. Indicated electrical impulse beginning at the AV node 3. Reflects atrial muscle depolarization 4. Identifies ventricular muscle depolarization 5. Has duration of normally 0.11 seconds or less.

13. A client has driven himself to the ER. He is 50 years old, has a history of hypertension, and informs the nurse that his father died of a heart attack at 60 years of age. The client is presently complaining of indigestion. The nurse connects him to an ECG monitor and begins administering oxygen at 2 L/minute per NC. The nurses next action would be to: 1. Call for the doctor 2. Start an intravenous line 3. Obtain a portable chest radiograph 4. Draw blood for laboratory studies 14. The nurse receives emergency laboratory results for a client with chest pain and immediately informs the physician. An increased myoglobin level suggests which of the following? 1. Cancer 2. Hypertension 3. Liver disease 4. Myocardial infarction 15. When teaching a client about propranolol hydrochloride, the nurse should base the information on the knowledge that propranolol hydrochloride: 1. Blocks beta-adrenergic stimulation and thus causes decreased heart rate, myocardial contractility, and conduction. 2. Increases norepinephrine secretion and thus decreases blood pressure and heart rate. 3. Is a potent arterial and venous vasodilator that reduces peripheral vascular resistance and lowers blood pressure. 4. Is an angiotensin-converting enzyme inhibitor that reduces blood pressure by blocking the conversion of angiotensin I to angiotensin II. 16. The most important long-term goal for a client with hypertension would be to: 1. Learn how to avoid stress 2. Explore a job change or early retirement 3. Make a commitment to long-term therapy 4. Control high blood pressure 17. Hypertension is known as the silent killer. This phrase is associated with the fact that hypertension often goes undetected until symptoms of other system failures occur. This may occur in the form of: 1. Cerebrovascular accident 2. Liver disease 3. Myocardial infarction 4. Pulmonary disease 18. During the previous few months, a 56-year-old woman felt brief twinges of chest pain while working in her garden and has had frequent episodes of indigestion. She comes to the hospital after experiencing severe anterior chest pain while raking leaves. Her evaluation confirms a diagnosis of stable angina pectoris. After stabilization and treatment, the client is discharged from the hospital. At her follow-up appointment, she is discouraged because she is experiencing pain with increasing frequency. She states that she is visiting an invalid friend twice a week and now cannot walk up the second flight of steps to the friends apartment without pain. Which of the following measures that the nurse could suggest would most likely help the client deal with this problem? 1. Visit her friend earlier in the day. 2. Rest for at least an hour before climbing the stairs. 3. Take a nitroglycerin tablet before climbing the stairs. 4. Lie down once she reaches the friends apartment. 19. Which of the following symptoms should the nurse teach the client with unstable angina to report immediately to her physician? 1. A change in the pattern of her pain 2. Pain during sex

3. Pain during an argument with her husband 4. Pain during or after an activity such as lawn mowing 20. The physician refers the client with unstable angina for a cardiac catheterization. The nurse explains to the client that this procedure is being used in this specific case to: 1. Open and dilate the blocked coronary arteries 2. Assess the extent of arterial blockage 3. Bypass obstructed vessels 4. Assess the functional adequacy of the valves and heart muscle. 21. As an initial step in treating a client with angina, the physician prescribes nitroglycerin tablets, 0.3mg given sublingually. This drugs principal effects are produced by: 1. Antispasmodic effect on the pericardium 2. Causing an increased myocardial oxygen demand 3. Vasodilation of peripheral vasculature 4. Improved conductivity in the myocardium 22. The nurse teaches the client with angina about the common expected side effects of nitroglycerin, including: 1. Headache 2. High blood pressure 3. Shortness of breath 4. Stomach cramps 23. Sublingual nitroglycerin tablets begin to work within 1 to 2 minutes. How should the nurse instruct the client to use the drug when chest pain occurs? 1. Take one tablet every 2 to 5 minutes until the pain stops. 2. Take one tablet and rest for 10 minutes. Call the physician if pain persists after 10 minutes. 3. Take one tablet, then an additional tablet every 5 minutes for a total of 3 tablets. Call the physician if pain persists after three tablets. 4. Take one tablet. If pain persists after 5 minutes, take two tablets. If pain still persists 5 minutes later, call the physician. 24. Which of the following arteries primarily feeds the anterior wall of the heart? 1. Circumflex artery 2. Internal mammary artery 3. Left anterior descending artery 4. Right coronary artery 25. When do coronary arteries primarily receive blood flow? 1. During inspiration 2. During diastolic 3. During expiration 4. During systole 26. Prolonged occlusion of the right coronary artery produces an infarction in which of the following areas of the heart? 1. Anterior 2. Apical 3. Inferior 4. Lateral 27. A murmur is heard at the second left intercostal space along the left sternal border. Which valve is this? 1. Aortic 2. Mitral 3. Pulmonic 4. Tricuspid 28. Which of the following blood tests is most indicative of cardiac damage?

1. Lactate dehydrogenase 2. Complete blood count (CBC) 3. Troponin I 4. Creatine kinase (CK) 29. Which of the following diagnostic tools is most commonly used to determine the location of myocardial damage? 1. Cardiac catheterization 2. Cardiac enzymes 3. Echocardiogram 4. Electrocardiogram (ECG) 30. Which of the following types of pain is most characteristic of angina? 1. Knifelike 2. Sharp 3. Shooting 4. Tightness 31. Which of the following parameters is the major determinant of diastolic blood pressure? 1. Baroreceptors 2. Cardiac output 3. Renal function 4. Vascular resistance 32. Which of the following factors can cause blood pressure to drop to normal levels? 1. Kidneys excretion of sodium only 2. Kidneys retention of sodium and water 3. Kidneys excretion of sodium and water 4. Kidneys retention of sodium and excretion of water 33. Baroreceptors in the carotid artery walls and aorta respond to which of the following conditions? 1. Changes in blood pressure 2. Changes in arterial oxygen tension 3. Changes in arterial carbon dioxide tension 4. Changes in heart rate 34. Which of the following terms describes the force against which the ventricle must expel blood? 1. Afterload 2. Cardiac output 3. Overload 4. Preload 35. Which of the following terms is used to describe the amount of stretch on the myocardium at the end of diastole? 1. Afterload 2. Cardiac index 3. Cardiac output 4. Preload 36. A 57-year-old client with a history of asthma is prescribed propranolol (Inderal) to control hypertension. Before administered propranolol, which of the following actions should the nurse take first? 1. Monitor the apical pulse rate 2. Instruct the client to take medication with food 3. Question the physician about the order 4. Caution the client to rise slowly when standing. 37. One hour after administering IV furosemide (Lasix) to a client with heart failure, a short burst of ventricular tachycardia appears on the cardiac monitor. Which of the following electrolyte imbalances should the nurse suspect?

1. Hypocalcemia 2. Hypermagnesemia 3. Hypokalemia 4. Hypernatremia 38. A client is receiving spironolactone to treat hypertension. Which of the following instructions should the nurse provide? 1. Eat foods high in potassium. 2. Take daily potassium supplements. 3. Discontinue sodium restrictions. 4. Avoid salt substitutes. 39. When assessing an ECG, the nurse knows that the P-R interval represents the time it takes for the: 1. Impulse to begin atrial contraction 2. Impulse to transverse the atria to the AV node 3. SA node to discharge the impulse to begin atrial depolarization 4. Impulse to travel to the ventricles 40. Following a treadmill test and cardiac catheterization, the client is found to have coronary artery disease, which is inoperative. He is referred to the cardiac rehabilitation unit. During his first visit to the unit he says that he doesnt understand why he needs to be there because there is nothing that can be done to make him better. The best nursing response is: 1. Cardiac rehabilitation is not a cure but can help restore you to many of your former activities. 2. Here we teach you to gradually change your lifestyle to accommodate your heart disease. 3. You are probably right but we can gradually increase your activities so that you can live a more active life. 4. Do you feel that you will have to make some changes in your life now? 41. To evaluate a clients condition following cardiac catheterization, the nurse will palpate the pulse: 1. In all extremities 2. At the insertion site 3. Distal to the catheter insertion 4. Above the catheter insertion 42. A clients physician orders nuclear cardiography and makes an appointment for a thallium scan. The purpose of injecting radioisotope into the bloodstream is to detect: 1. Normal vs. abnormal tissue 2. Damage in areas of the heart 3. Ventricular function 4. Myocardial scarring and perfusion 43. A client enters the ER complaining of severe chest pain. A myocardial infarction is suspected. A 12 lead ECG appears normal, but the doctor admits the client for further testing until cardiac enzyme studies are returned. All of the following will be included in the nursing care plan. Which activity has the highest priority? 1. Monitoring vital signs 2. Completing a physical assessment 3. Maintaining cardiac monitoring 4. Maintaining at least one IV access site 44. A client is experiencing tachycardia. The nurses understanding of the physiological basis for this symptom is explained by which of the following statements? 1. The demand for oxygen is decreased because of pleural involvement 2. The inflammatory process causes the body to demand more oxygen to meet its needs. 3. The heart has to pump faster to meet the demand for oxygen when there is lowered arterial oxygen tension. 4. Respirations are labored.

10

45. A client enters the ER complaining of chest pressure and severe epigastric distress. His VS are 158/90, 94, 24, and 99*F. The doctor orders cardiac enzymes. If the client were diagnosed with an MI, the nurse would expect which cardiac enzyme to rise within the next 3 to 8 hours? 1. Creatine kinase (CK or CPK) 2. Lactic dehydrogenase (LDH) 3. LDH-1 4. LDH-2 46. A 45-year-old male client with leg ulcers and arterial insufficiency is admitted to the hospital. The nurse understands that leg ulcers of this nature are usually caused by: 1. Decreased arterial blood flow secondary to vasoconstriction 2. Decreased arterial blood flow leading to hyperemia 3. Atherosclerotic obstruction of the arteries 4. Trauma to the lower extremities 47. Which of the following instructions should be included in the discharge teaching for a patient discharged with a transdermal nitroglycerin patch? 1. Apply the patch to a non hairy, nonfatty area of the upper torso or arms. 2. Apply the patch to the same site each day to maintain consistent drug absorption. 3. If you get a headache, remove the patch for 4 hours and then reapply. 4. If you get chest pain, apply a second patch right next to the first patch. 48. In order to prevent the development of tolerance, the nurse instructs the patient to: 1. Apply the nitroglycerin patch every other day 2. Switch to sublingual nitroglycerin when the patients systolic blood pressure elevates to >140 mm Hg 3. Apply the nitroglycerin patch for 14 hours each and remove for 10 hours at night 4. Use the nitroglycerin patch for acute episodes of angina only 49. Direct-acting vasodilators have which of the following effects on the heart rate? 1. Heart rate decreases 2. Heart rate remains significantly unchanged 3. Heart rate increases 4. Heart rate becomes irregular 50. When teaching a patient why spironolactone (Aldactone) and furosemide (Lasix) are prescribed together, the nurse bases teaching on the knowledge that: 1. Moderate doses of two different types of diuretics are more effective than a large dose of one type 2. This combination promotes diuresis but decreases the risk of hypokalemia 3. This combination prevents dehydration and hypovolemia 4. Using two drugs increases osmolality of plasma and the glomerular filtration rate

Answers and Rationale


Gauge your performance by counter checking your answers to the answers below. Learn more about the question by reading the rationale. If you have any disputes or questions, please direct them to the comments section. 1. Answer: 4. This procedure requires an informed consent because it involves injection of a radiopaque dye into the blood vessel. The risk of allergic reaction and possible anaphylaxis is serious and must be assessed before the procedure. 2. Answer: 3. Chest pain is assessed by using the standard pain assessment parameters. Options 1, 2, and 4 may or may not help discriminate the origin of pain. Pain of pleuropulmonary origin usually worsens on inspiration. 3. Answer: 2. On transfer from the CCU, the client is allowed self-care activities and bathroom privileges. Supervised ambulation for brief distances are encouraged, with distances gradually increased (50, 100, 200 feet). 4. Answer: 1. Edema, the accumulation of excess fluid in the interstitial spaces, can be measured by intake greater than output and by a sudden increase in weight. Diuretics should be given in the morning whenever possible to avoid nocturia. Strict sodium restrictions are reserved for clients with severe symptoms.

11

5. Answer: 1. Sudden loss of electrocardiogram complexes indicates ventricular asystole or possible electrode displacement. Accurate assessment of the client and equipment is necessary to determine the cause and identify the appropriate intervention. 6. Answer: 4. BP should be taken with the client seated with the arm bared, positioned with support and at heart level. The client should sit with the legs on the floor, feet uncrossed, and not speak during the recording. The client should not have smoked tobacco or taken in caffeine in the 30 minutes preceding the measurement. The client should rest quietly for 5 minutes before the reading is taken. The cuff bladder should encircle at least 80% of the limb being measured. Gauges other than a mercury sphygmomanometer should be calibrated every 6 months to ensure accuracy. 7. Answer: 4. The antidote to heparin is protamine sulfate and should be readily available for use if excessive bleeding or hemorrhage should occur. Vitamin K is an antidote for warfarin. 8. Answer: 3. The therapeutic range for prothrombin time is 1.5 to 2 times the control for clients at risk for thrombus. Based on the clients control value, the therapeutic range for this in dividual would be 16.5 to 22 seconds. Therefore the result is within therapeutic range. 9. Answer: 2. Warfarin works in the liver and inhibits synthesis of four vitamin K-dependent clotting factors (X, IX, VII, and II), but it takes 3 to 4 days before the therapeutic effect of warfarin is exhibited. 10. Answer: 1. Although obtaining the ECG, chest x-ray, and blood work are all important, the nurses priority action would be to relieve the crushing chest pain. 11. Answer: 2. Thrombolytic drugs are administered within the first 6 hours after onset of a MI to lyse clots and reduce the extent of myocardial damage. 12. Answer: 1, 3, 5. In a client who has had an ECG, the P wave represents the activation of the electrical impulse in the SA node, which is then transmitted to the AV node. In addition, the P wave represents atrial muscle depolarization, not ventricular depolarization. The normal duration of the P wave is 0.11 seconds or less in duration and 2.5 mm or more in height. 13. Answer: 2. Advanced cardiac life support recommends that at least one or two intravenous lines be inserted in one or both of the antecubital spaces. Calling the physician, obtaining a portable chest radiograph, and drawing blood are important but secondary to starting the intravenous line. 14. Answer: 4. Detection of myoglobin is one diagnostic tool to determine whether myocardial damage has occurred. Myoglobin is generally detected about one hour after a heart attack is experienced and peaks within 4 to 6 hours after infarction (Remember, less than 90 mg/L is normal). 15. Answer: 1. Propranolol hydrochloride is a beta-adrenergic blocking agent. Actions of propranolol hydrochloride include reducing heart rate, decreasing myocardial contractility, and slowing conduction. 16. Answer: 3. Compliance is the most critical element of hypertensive therapy. In most cases, hypertensive clients require lifelong treatment and their hypertension cannot be managed successfully without drug therapy. Stress management and weight management are important components of hypertension therapy, but the priority goal is related to compliance. 17. Answer: 1. Hypertension is referred to as the silent killer for adults, because until the adult has significant damage to other systems, the hypertension may go undetected. CVAs can be related to long-term hypertension. Liver or pulmonary disease is generally not associated with hypertension. Myocardial infarction is generally related to coronary artery disease. 18. Answer: 3. Nitroglycerin may be used prophylactically before stressful physical activities such as stair climbing to help the client remain pain free. Visiting her friend early in the day would have no impact on decreasing pain episodes. Resting before or after an activity is not as likely to help prevent an activity-related pain episode. 19. Answer: 1. The client should report a change in the pattern of chest pain. It may indicate increasing severity of CAD. 20. Answer: 2. Cardiac catheterization is done in clients with angina primarily to assess the extent and severity of the coronary artery blockage, A decision about medical management, angioplasty, or coronary artery bypass surgery will be based on the catheterization results. 21. Answer: 3. Nitroglycerin produces peripheral vasodilation, which reduces myocardial oxygen consumption and demand. Vasodilation in coronary arteries and collateral vessels may also increase blood flow to the ischemic areas of the heart. Nitroglycerin decreases myocardial oxygen demand. Nitroglycerin does not have an effect on pericardial spasticity or conductivity in the myocardium.

12

22. Answer: 1. Because of the widespread vasodilating effects, nitroglycerin often produces such side effects as headache, hypotension, and dizziness. The client should lie or shit down to avoid fainting. Nitro does not cause shortness of breath or stomach cramps. 23. Answer: 3. The correct protocol for nitroglycerin used involves immediate administration, with subsequent doses taken at 5minute intervals as needed, for a total dose of 3 tablets. Sublingual nitroglycerin appears in the bloodstream within 2 to 3 minutes and is metabolized within about 10 minutes. 24. Answer: 3. The left anterior descending artery is the primary source of blood flow for the anterior wall of the heart. The circumflex artery supplies the lateral wall, the internal mammary supplies the mammary, and the right coronary artery supplies the inferior wall of the heart. 25. Answer: 2. Although the coronary arteries may receive a minute portion of blood during systole, most of the blood flow to coronary arteries is supplied during diastole. Breathing patterns are irrelevant to blood flow. 26. Answer: 3. The right coronary artery supplies the right ventricle, or the inferior portion of the heart. Therefore, prolonged occlusion could produce an infarction in that area. The right coronary artery doesnt supply the anterior portion (left ventricle), lateral portion (some of the left ventricle and the left atrium), or the apical portion (left ventricle) of the heart. 27. Answer: 3. Abnormalities of the pulmonic valve are auscultated at the second left intercostal space along the left sternal border. Aortic valve abnormalities are heard at the second intercostal space, to the right of the sternum. Mitral valve abnormalities are heard at the fifth intercostal space in the midclavicular line. Tricupsid valve abnormalities are heard at the 3rd and 4th intercostal spaces along the sternal border. 28. Answer: 3. Troponin I levels rise rapidly and are detectable within 1 hour of myocardial inju ry. Troponin levels arent detectable in people without cardiac injury. 29. Answer: 4. The ECG is the quickest, most accurate, and most widely used tool to determine the location of myocardial infarction. Cardiac enzymes are used to diagnose MI but cant d etermine the location. An echocardiogram is used most widely to view myocardial wall function after an MI has been diagnosed. Cardiac catheterization is an invasive study for determining coronary artery disease and may also indicate the location of myocardial damage, but the study may not be performed immediately. 30. Answer: 4. The pain of angina usually ranges from a vague feeling of tightness to heavy, intense pain. Pain impulses originate in the most visceral muscles and may move to such areas as the chest, neck, and arms. 31. Answer: 4. Vascular resistance is the impedance of blood flow by the arterioles that most predominantly affects the diastolic pressure. Cardiac output determines systolic blood pressure. 32. Answer: 3. The kidneys respond to a rise in blood pressure by excreting sodium and excess water. This response ultimately affects systolic pressure by regulating blood volume. 33. Answer: 1. Baroreceptors located in the carotid arteries and aorta sense pulsatile pressure. Decreases in pulsatile pressure cause a reflex increase in heart rate. Chemoreceptors in the medulla are primarily stimulated by carbon dioxide. Peripheral chemoreceptors in the aorta and carotid arteries are primarily stimulated by oxygen. 34. Answer: 1. Afterload refers to the resistance normally maintained by the aortic and pulmonic valves, the condition and tone of the aorta, and the resistance offered by the systemic and pulmonary arterioles. Cardiac output is the amount of blood expelled from the heart per minute. Overload refers to an abundance of circulating volume. Preload is the volume of blood in the ventricle at the end of diastole. 35. Answer: 4. Preload is the amount of stretch of the cardiac muscle fibers at the end of diastole. The volume of blood in the ventricle at the end of diastole determines the preload. Afterload is the force against which the ventricle must expel blood. Cardiac index is the individualized measurement of cardiac output, based on the clients body surface area. Cardiac output is the amount of blood the heart is expelling per minute. 36. Answer: 3. Propranolol and other beta-adrenergic blockers are contraindicated in a client with asthma, so the nurse should question the physician before giving the dose. The other responses are appropriate actions for a client receiving propranolol, but questioning the physician takes priority. The clients apical pulse should always be checked before giving propranolol; if th e pulse rate is extremely low, the nurse should withhold the drug and notify the physician.

13

37. Answer: 3. Furosemide is a potassium-depleting diuretic than can cause hypokalemia. In turn, hypokalemia increases myocardial excitability, leading to ventricular tachycardia. 38. Answer: 4. Because spironolactone is a potassium-sparing diuretic, the client should avoid salt substitutes because of their high potassium content. The client should also avoid potassium-rich foods and potassium supplements. To reduce fluid-volume overload, sodium restrictions should continue. 39. Answer: 4. The P-R interval is measured on the ECG strip from the beginning of the P wave to the beginning of the QRS complex. It is the time it takes for the impulse to travel to the ventricle. 40. Answer: 1. Such a response does not have false hope to the client but is positive and realistic. The answer tells the client what cardiac rehabilitation is and does not dwell upon his negativity about it. 41. Answer: 3. Palpating pulses distal to the insertion site is important to evaluate for thrombophlebitis and vessel occlusion. They should be bilateral and strong. 42. Answer: 4. This scan detects myocardial damage and perfusion, an acute or chronic MI. It is a more specific answer than (1) or (2). Specific ventricular function is tested by a gated cardiac blood pool scan. 43. Answer: 3. Even though initial tests seem to be within normal range, it takes at least 3 hours for the cardiac enzyme studies to register. In the meantime, the client needs to be watched for bradycardia, heart block, ventricular irritability, and other arrhythmias. Other activities can be accomplished around the MI monitoring. 44. Answer: 3. The arterial oxygen supply is lowered and the demand for oxygen is increased, which results in the hearts hav ing to beat faster to meet the bodys needs for oxygen. 45. Answer: 1. Creatine kinase (CK, formally known as CPK) rises in 3-8 hours if an MI is present. When the myocardium is damaged, CPK leaks out of the cell membranes and into the bloodstream. Lactic dehydrogenase rises in 24-48 hours, and LDH-1 and LDH-2 rises in 8-24 hours. 46. Answer: 1. Decreased arterial flow is a result of vasospasm. The etiology is unknown. It is more problematic in colder climates or when the person is under stress. Hyperemia occurs when the vasospasm is relieved. 47. Answer: 1. A nitroglycerin patch should be applied to a non hairy, nonfatty area for the best and most consistent absorption rates. Sites should be rotated to prevent skin irritation, and the drug should be continued if headache occurs because tolerance will develop. Sublingual nitroglycerin should be used to treat chest pain. 48. Answer: 3. Tolerance can be prevented by maintaining an 8- to 12-hour nitrate-free period each day. 49. Answer: 3. Heart rate increases in response to decreased blood pressure caused by vasodilation. 50. Answer: 2. Spironolactone is a potassium-sparing diuretic; furosemide is a potassium-losing diuretic. Giving these together minimizes electrolyte imbalance.

Introduction
This is a 40-item examination about Hematologic Disorders like Hemophilia, Sickle Cell Disease, Anemia and Polycythemia Vera. This is an NCLEX style examination.

Topics

Hemophilia Sickle Cell Disease Anemia Polycythemia Vera

Guidelines
Read each question carefully and choose the best answer. You are given one minute per question. Spend your time wisely! Answers and rationales are given below. Be sure to read them. If you need more clarifications, please direct them to the comments section.

14

Questions
1. The nurse is preparing to teach a client with microcytic hypochromic anemia about the diet to follow after discharge. Which of the following foods should be included in the diet? 1. Eggs 2. Lettuce 3. Citrus fruits 4. Cheese 2. The nurse would instruct the client to eat which of the following foods to obtain the best supply of vitamin B12? 1. Whole grains 2. Green leafy vegetables 3. Meats and dairy products 4. Broccoli and Brussels sprouts 3. The nurse has just admitted a 35-year-old female client who has a serum B12 concentration of 800 pg/ml. Which of the following laboratory findings would cue the nurse to focus the client history on specific drug or alcohol abuse? 1. Total bilirubin, 0.3 mg/dL 2. Serum creatinine, 0.5 mg/dL 3. Hemoglobin, 16 g/dL 4. Folate, 1.5 ng/mL 4. The nurse understands that the client with pernicious anemia will have which distinguishing laboratory findings? 1. Schillings test, elevated 2. Intrinsic factor, absent. 3. Sedimentation rate, 16 mm/hour 4. RBCs 5.0 million 5. The nurse devises a teaching plan for the patient with aplastic anemia. Which of the following is the most important concept to teach for health maintenance? 1. Eat animal protein and dark leafy vegetables each day 2. Avoid exposure to others with acute infection 3. Practice yoga and meditation to decrease stress and anxiety 4. Get 8 hours of sleep at night and take naps during the day 6. A client comes into the health clinic 3 years after undergoing a resection of the terminal ileum complaining of weakness, shortness of breath, and a sore tongue. Which client statement indicates a need for intervention and client teaching? 1. I have been drinking plenty of fluids. 2. I have been gargling with warm salt water for my sore tongue. 3. I have 3 to 4 loose stools per day. 4. I take a vitamin B12 tablet every day. 7. A vegetarian client was referred to a dietitian for nutritional counseling for anemia. Which client outcome indicates that the client does not understand nutritional counseling? The client: 1. Adds dried fruit to cereal and baked goods 2. Cooks tomato-based foods in iron pots 3. Drinks coffee or tea with meals 4. Adds vitamin C to all meals 8. A client was admitted with iron deficiency anemia and blood-streaked emesis. Which question is most appropriate for the nurse to ask in determining the extent of the clients activity intolerance? 1. What activities were you able to do 6 months ago compared with the present? 2. How long have you had this problem? 3. Have you been able to keep up with all your usual activities? 4. Are you more tired now than you used to be?

15

9. The primary purpose of the Schilling test is to measure the clients ability to: 1. Store vitamin B12 2. Digest vitamin B12 3. Absorb vitamin B12 4. Produce vitamin B12 10. The nurse implements which of the following for the client who is starting a Schilling test? 1. Administering methylcellulose (Citrucel) 2. Starting a 24- to 48 hour urine specimen collection 3. Maintaining NPO status 4. Starting a 72 hour stool specimen collection 11. A client with pernicious anemia asks why she must take vitamin B12 injections for the rest of her life. What is the nurses best response? 1. The reason for your vitamin deficiency is an inability to absorb the vitamin because the stomach is not producing suffici ent acid. 2. The reason for your vitamin deficiency is an inability to absorb the vitamin because the stomach is not producing sufficient intrinsic factor. 3. The reason for your vitamin deficiency is an excessive excretion of the vitamin because of kidney dysfunction. 4. The reason for your vitamin deficiency is an increased requirement for the vitamin because of rapid red blood cell production. 12. The nurse is assessing a clients activity intolerance by having the client walk on a treadmill for 5 minutes. Which of the following indicates an abnormal response? 1. Pulse rate increased by 20 bpm immediately after the activity 2. Respiratory rate decreased by 5 breaths/minute 3. Diastolic blood pressure increased by 7 mm Hg 4. Pulse rate within 6 bpm of resting phase after 3 minutes of rest. 13. When comparing the hematocrit levels of a post-op client, the nurse notes that the hematocrit decreased from 36% to 34% on the third day even though the RBC and hemoglobin values remained stable at 4.5 million and 11.9 g/dL, respectively. Which nursing intervention is most appropriate? 1. Check the dressing and drains for frank bleeding 2. Call the physician 3. Continue to monitor vital signs 4. Start oxygen at 2L/min per NC 14. A client is to receive epoetin (Epogen) injections. What laboratory value should the nurse assess before giving the injection? 1. Hematocrit 2. Partial thromboplastin time 3. Hemoglobin concentration 4. Prothrombin time 15. A client states that she is afraid of receiving vitamin B12 injections because of the potential toxic reactions. What is the nurses best response to relieve these fears? 1. Vitamin B12 will cause ringing in the eats before a toxic level is reached. 2. Vitamin B12 may cause a very mild skin rash initially. 3. Vitamin B12 may cause mild nausea but nothing toxic. 4. Vitamin B12 is generally free of toxicity because it is water soluble. 16. A client with microcytic anemia is having trouble selecting food items from the hospital menu. Which food is best for the nurse to suggest for satisfying the clients nutritional needs and personal preferences?

16

1. Egg yolks 2. Brown rice 3. Vegetables 4. Tea 17. A client with macrocytic anemia has a burn on her foot and states that she had been watching television while lying on a heating pad. What is the nurses first response? 1. Assess for potential abuse 2. Check for diminished sensations 3. Document the findings 4. Clean and dress the area 18. Which of the following nursing assessments is a late symptom of polycythemia vera? 1. Headache 2. Dizziness 3. Pruritus 4. Shortness of breath 19. The nurse is teaching a client with polycythemia vera about potential complications from this disease. Which manifestations would the nurse include in the clients teaching plan? Select all that apply. 1. Hearing loss 2. Visual disturbance 3. Headache 4. Orthopnea 5. Gout 6. Weight loss 20. When a client is diagnosed with aplastic anemia, the nurse monitors for changes in which of the following physiological functions? 1. Bleeding tendencies 2. Intake and output 3. Peripheral sensation 4. Bowel function 21. Which of the following blood components is decreased in anemia? 1. Erythrocytes 2. Granulocytes 3. Leukocytes 4. Platelets 22. A client with anemia may be tired due to a tissue deficiency of which of the following substances? 1. Carbon dioxide 2. Factor VIII 3. Oxygen 4. T-cell antibodies 23. Which of the following cells is the precursor to the red blood cell (RBC)? 1. B cell 2. Macrophage 3. Stem cell 4. T cell 24. Which of the following symptoms is expected with hemoglobin of 10 g/dl? 1. None 2. Pallor

17

3. Palpitations 4. Shortness of breath 25. Which of the following diagnostic findings are most likely for a client with aplastic anemia? 1. Decreased production of T-helper cells 2. Decreased levels of white blood cells, red blood cells, and platelets 3. Increased levels of WBCs, RBCs, and platelets 4. Reed-Sternberg cells and lymph node enlargement 26. A client with iron deficiency anemia is scheduled for discharge. Which instruction about prescribed ferrous gluconate therapy should the nurse include in the teaching plan? 1. Take the medication with an antacid. 2. Take the medication with a glass of milk. 3. Take the medication with cereal. 4. Take the medication on an empty stomach. 27. Which of the following disorders results from a deficiency of factor VIII? 1. Sickle cell disease 2. Christmas disease 3. Hemophilia A 4. Hemophilia B 28. The nurse explains to the parents of a 1-year-old child admitted to the hospital in a sickle cell crisis that the local tissue damage the child has on admission is caused by which of the following? 1. Autoimmune reaction complicated by hypoxia 2. Lack of oxygen in the red blood cells 3. Obstruction to circulation 4. Elevated serum bilirubin concentration. 29. The mothers asks the nurse why her childs hemoglobin was normal at birth but now the child has S hemoglobin. Which of the following responses by the nurse is most appropriate? 1. The placenta bars passage of the hemoglobin S from the mother to the fetus. 2. The red bone marrow does not begin to produce hemoglobin S until several months after birth. 3. Antibodies transmitted from you to the fetus provide the newborn with temporary immunity. 4. The newborn has a high concentration of fetal hemoglobin in the blood for some time after birth. 30. Which of the following would the nurse identify as the priority nursing diagnosis during a toddlers vaso-occlusive sickle cell crisis? 1. Ineffective coping related to the presence of a life-threatening disease 2. Decreased cardiac output related to abnormal hemoglobin formation 3. Pain related to tissue anoxia 4. Excess fluid volume related to infection 31. A mother asks the nurse if her childs iron deficiency anemia is related to the childs frequent infections. The nurse responds based on the understanding of which of the following? 1. Little is known about iron-deficiency anemia and its relationship to infection in children. 2. Children with iron deficiency anemia are more susceptible to infection than are other children. 3. Children with iron-deficiency anemia are less susceptible to infection than are other children. 4. Children with iron-deficient anemia are equally as susceptible to infection as are other children. 32. Which statements by the mother of a toddler would lead the nurse to suspect that the child has iron-deficiency anemia? Select all that apply. 1. He drinks over 3 cups of milk per day. 2. I cant keep enough apple juice in the house; he must drink over 10 ounces per day. 3. He refuses to eat more than 2 different kinds of vegetables.

18

4. He doesnt like meat, but he will eat small amounts of it. 5. He sleeps 12 hours every night and take a 2-hour nap. 33. Which of the following foods would the nurse encourage the mother to offer to her child with iron deficiency anemia? 1. Rice cereal, whole milk, and yellow vegetables 2. Potato, peas, and chicken 3. Macaroni, cheese, and ham 4. Pudding, green vegetables, and rice 34. The physician has ordered several laboratory tests to help diagnose an infants bleeding disorder. Which of the following tests, if abnormal, would the nurse interpret as most likely to indicate hemophilia? 1. Bleeding time 2. Tourniquet test 3. Clot retraction test 4. Partial thromboplastin time (PTT) 35. Which of the following assessments in a child with hemophilia would lead the nurse to suspect early hemarthrosis? 1. Childs reluctance to move a body part 2. Cool, pale, clammy extremity 3. Eccymosis formation around a joint 4. Instability of a long bone in passive movement 36. Because of the risks associated with administration of factor VIII concentrate, the nurse would teach the clients family to recognize and report which of the following? 1. Yellowing of the skin 2. Constipation 3. Abdominal distention 4. Puffiness around the eyes 37. A child suspected of having sickle cell disease is seen in a clinic, and laboratory studies are performed. A nurse checks the lab results, knowing that which of the following would be increased in this disease? 1. Platelet count 2. Hematocrit level 3. Reticulocyte count 4. Hemoglobin level 38. A clinic nurse instructs the mother of a child with sickle cell disease about the precipitating factors related to pain crisis. Which of the following, if identified by the mother as a precipitating factor, indicates the need for further instructions? 1. Infection 2. Trauma 3. Fluid overload 4. Stress 39. Laboratory studies are performed for a child suspected of having iron deficiency anemia. The nurse reviews the laboratory results, knowing that which of the following results would indicate this type of anemia? 1. An elevated hemoglobin level 2. A decreased reticulocyte count 3. An elevated RBC count 4. Red blood cells that are microcytic and hypochromic 40. A pediatric nurse health educator provides a teaching session to the nursing staff regarding hemophilia. Which of the following information regarding this disorder would the nurse plan to include in the discussion? 1. Hemophilia is a Y linked hereditary disorder 2. Males inherit hemophilia from their fathers

19

3. Females inherit hemophilia from their mothers 4. Hemophilia A results from a deficiency of factor VIII

Answers and Rationale


1. Answer: 1. One of the microcytic, hypochromic anemias is iron-deficiency anemia. A rich source of iron is needed in the diet, and eggs are high in iron. Other foods high in iron include organ and muscle (dark) meats; shellfish, shrimp, and tuna; enriched, whole-grain, and fortified cereals and breads; legumes, nuts, dried fruits, and beans; oatmeal; and sweet potatoes. Dark green leafy vegetables and citrus fruits are good sources of vitamin C. Cheese is a good source of calcium. 2. Answer: 3. Good sources of vitamin B12 include meats and dairy products. Whole grains are a good source of thiamine. Green leafy vegetables are good sources of niacin, folate, and carotenoids (precursors of vitamin A). Broccoli and Brussels sprouts are good sources of ascorbic acid (vitamin C). 3. Answer: 4. The normal range of folic acid is 1.8 to 9 ng/mL, and the normal range of vitamin B12 is 200 to 900 pg/mL. A low folic acid level in the presence of a normal vitamin B12 level is indicative of a primary folic acid-deficiency anemia. Factors that affect the absorption of folic acid are drugs such as methotrexate, oral contraceptives, antiseizure drugs, and alcohol. The total bilirubin, serum creatinine, and hemoglobin values are within normal limits. 4. Answer: 2. The defining characteristic of pernicious anemia, a megaloblastic anemia, is lack of the intrinsic factor, which results from atrophy of the stomach wall. Without the intrinsic factor, vitamin B12 cannot be absorbed in the small intestines, and folic acid needs vitamin B12 for DNA synthesis of RBCs. The gastric analysis was done to determine the primary cause of the anemia. An elevated excretion of the injected radioactive vitamin B12, which is protocol for the first and second stage of the Schilling test, indicates that the client has the intrinsic factor and can absorb vitamin B12 into the intestinal tract. A sedimentation rate of 16 mm/hour is normal for both men and women and is a nonspecific test to detect the presence of inflammation. It is not specific to anemias. An RBC value of 5.0 million is a normal value for both men and women and does not indicate anemia. 5. Answer: 2. Clients with aplastic anemia are severely immunocompromised and at risk for infection and possible death related to bone marrow suppression and pancytopenia. Strict aseptic technique and reverse isolation are important measures to prevent infection. Although diet, reduced stress, and rest are valued in supporting health, the potentially fatal consequence of an acute infection places it as a priority for teaching the client about health maintenance. Animal meat and dark green leafy vegetables, good sources of vitamin B12 and folic acid, should be included in the daily diet. Yoga and meditation are good complimentary therapies to reduce stress. Eight hours of rest and naps are good for spacing and pacing activity and rest. 6. Answer: 4. Vitamin B12 combines with intrinsic factor in the stomach and is then carried to the ileum, where it is absorbed in the bloodstream. In this situation, vitamin B12 cannot be absorbed regardless of the amount of oral intake of sources of vitamin B12 such as animal protein or vitamin B12 tablets. Vitamin B12 needs to be injected every month, because the ileum has been surgically removed. Replacement of fluids and electrolytes is important when the client has continuous multiple loose stools on a daily basis. Warm salt water is used to soothe sore mucous membranes. Crohns disease and small bowel resection may cause several loose stools a day. 7. Answer: 3. Coffee and tea increase gastrointestinal motility and inhibit the absorption of nonheme iron. Clients are instructed to add dried fruits to dishes at every meal because dried fruits are a nonheme or nonanimal iron source. Cooking in iron cookware, especially acid-based foods such as tomatoes, adds iron to the diet. Clients are instructed to add a rich supply of vitamin C to every meal because the absorption of iron is increased when food with vitamin C or ascorbic acid is consumed. 8. Answer: 1. It is difficult to determine activity intolerance without objectively comparing activities from one time frame to another. Because iron deficiency anemia can occur gradually and individual endurance varies, the nurse can best assess the clients activity tolerance by asking the client to compare activities 6 months ago and at the present. Asking a client how l ong a problem has existed is a very open-ended question that allows for too much subjectivity for any definition of the clients activity tolerance. Also, the client may not even identify that a problem exists. Asking the client whether he is staying abreast of usual activities addresses whether the tasks were completed, not the tolerance of the client while the tasks were being completed or the resulting condition of the client after the tasks were completed. Asking the client if he is more tired now than usual does not address his activity tolerance. Tiredness is a subjective evaluation and again can be distorted by factors such as the gradual onset of the anemia or the endurance of the individual.

20

9. Answer: 3. Pernicious anemia is caused by the bodys inability to absorb vitamin B12. This results in a lack of intrinsic factor in the gastric juices. Schillings test helps diagnose pernicious anemia by determining the clients ability to absorb vitami n B12. 10. Answer: 2. Urinary vitamin B12 levels are measured after the ingestion of radioactive vitamin B12. A 24-to 48- hour urine specimen is collected after administration of an oral dose of radioactively tagged vitamin B12 and an injection of non-radioactive vitamin B12. In a healthy state of absorption, excess vitamin B12 is excreted in the urine; in a malabsorption state or when the intrinsic factor is missing, vitamin B12 is excreted in the feces. Citrucel is a bulk-forming agent. Laxatives interfere with the absorption of vitamin B12. The client is NPO 8 to 12 hours before the test but is not NPO during the test. A stool collection is not part of the Schilling test. If stool contaminates the urine collection, the results will be altered. 11. Answer: 2. Most clients with pernicious anemia have deficient production of intrinsic factor in the stomach. Intrinsic factor attaches to the vitamin in the stomach and forms a complex that allows the vitamin to be absorbed in the small intestine. The stomach is producing enough acid, there is not an excessive excretion of the vitamin, and there is not a rapid production of RBCs in this condition. 12. Answer: 2. The normal physiologic response to activity is an increased metabolic rate over the resting basal rate. The decrease in respiratory rate indicates that the client is not strong enough to complete the mechanical cycle of respiration needed for gas exchange. The post activity pulse is expected to increase immediately after activity but by no more than 50 bpm if it is strenuous activity. The diastolic blood pressure is expected to rise but by no more than 15 mm Hg. The pulse returns to within 6 bpm of the resting pulse after 3 minutes of rest. 13. Answer: 3. The nurse should continue to monitor the client, because this value reflects a normal physiologic response. The physician does not need to be called, and oxygen does not need to be started based on these laboratory findings. Immediately after surgery, the clients hematocrit reflects a falsely high value related to the bodys compensatory response to the stres s of sudden loss of fluids and blood. Activation of the intrinsic pathway and the renin-angiotensin cycle via antidiuretic hormone produces vasoconstriction and retention of fluid for the first 1 to 2 day post-op. By the second to third day, this response decreases and the clients hematocrit level is more refle ctive of the amount of RBCs in the plasma. Fresh bleeding is a less likely occurrence on the third post-op day but is not impossible; however, the nurse would have expected to see a decrease in the RBC and hemoglobin values accompanying the hematocrit. 14. Answer: 1. Epogen is a recombinant DNA form of erythropoietin, which stimulates the production of RBCs and therefore causes the hematocrit to rise. The elevation in hematocrit causes an elevation in blood pressure; therefore, the blood pressure is a vital sign that should be checked. The PTT, hemoglobin level, and PT are not monitored for this drug. 15. Answer: 4. Vitamin B12 is a water-soluble vitamin. When water-soluble vitamins are taken in excess of the bodys needs, they are filtered through the kidneys and excreted. Vitamin B12 is considered to be nontoxic. Adverse reactions that have occurred are believed to be related to impurities or to the preservative in B12 preparations. Ringing in the ears, skin rash, and nausea are not considered to be related to vitamin B12 administration. 16. Answer: 2. Brown rice is a source of iron from plant sources (nonheme iron). Other sources of non heme iron are whole-grain cereals and breads, dark green vegetables, legumes, nuts, dried fruits (apricots, raisins, dates), oatmeal, and sweet potatoes. Egg yolks have iron but it is not as well absorbed as iron from other sources. Vegetables are a good source of vitamins that may facilitate iron absorption. Tea contains tannin, which combines with nonheme iron, preventing its absorption. 17. Answer: 2. Macrocytic anemias can result from deficiencies in vitamin B12 or ascorbic acid. Only vitamin B12 deficiency causes diminished sensations of peripheral nerve endings. The nurse should assess for peripheral neuropathy and instruct the client in self-care activities for her diminished sensation to heat and pain. The burn could be related to abuse, but this conclusion would require more supporting data. The findings should be documented, but the nurse would want to address the cli ents sensations first. The decision of how to treat the burn should be determined by the physician. 18. Answer: 3. Pruritus is a late symptom that results from abnormal histamine metabolism. Headache and dizziness are early symptoms from engorged veins. Shortness of breath is an early symptom from congested mucous membrane and ineffective gas exchange. 19. Answer: 2, 3, 4, 5. Polycythemia vera, a condition in which too many RBCs are produced in the blood serum, can lead to an increase in the hematocrit and hypervolemia, hyperviscosity, and hypertension. Subsequently, the client can experience dizziness, tinnitus, visual disturbances, headaches, or a feeling of fullness in the head. The client may also experience cardiovascular

21

symptoms such as heart failure (shortness of breath and orthopnea) and increased clotting time or symptoms of an increased uric acid level such as painful swollen joints (usually the big toe). Hearing loss and weight loss are not manifestations associated with polycythemia vera. 20. Answer: 1. Aplastic anemia decreases the bone marrow production of RBCs, WBCs, and platelets. The client is at risk for bruising and bleeding tendencies. A change in the intake and output is important, but assessment for the potential for bleeding takes priority. Change in the peripheral nervous system is a priority problem specific to clients with vitamin B12 deficiency. Change in bowel function is not associated with aplastic anemia. 21. Answer: 1. Anemia is defined as a decreased number of erythrocytes (red blood cells). Leukopenia is a decreased number of leukocytes (white blood cells). Thrombocytopenia is a decreased number of platelets. Lastly, granulocytopenia is a decreased number of granulocytes (a type of white blood cells). 22. Answer: 3. Anemia stems from a decreased number of red blood cells and the resulting deficiency in oxygen and body tissues. Clotting factors, such as factor VIII, relate to the bodys ability to form blood clots and arent related to anemia , not is carbon dioxide of T antibodies. 23. Answer: 3. The precursor to the RBC is the stem cell. B cells, macrophages, and T cells and lymphocytes, not RBC precursors. 24. Answer: 1. Mild anemia usually has no clinical signs. Palpitations, SOB, and pallor are all associated with severe anemia. 25. Answer: 2. In aplastic anemia, the most likely diagnostic findings are decreased levels of all the cellular elements of the blood (pancytopenia). T-helper cell production doesnt decrease in aplastic anemia. Reed -Sternberg cells and lymph node enlargement occur with Hodgkins disease. 26. Answer: 4. Preferably, ferrous gluconate should be taken on an empty stomach. Ferrous gluconate should not be taken with antacids, milk, or whole-grain cereals because these foods reduce iron absorption. 27. Answer: 3. Hemophilia A results from a deficiency of factor VIII. Sickle cell disease is caused by a defective hemoglobin molecule. Christmas disease, also called hemophilia B, results in a factor IX deficiency. 28. Answer: 3. Characteristic sickle cells tend to cause log jams in capillaries. This results in poor circulation to local tissues, leading to ischemia and necrosis. The basic defect in sickle cell disease is an abnormality in the structure of RBCs. The erythrocytes are sickle-shaped, rough in texture, and rigid. Sickle cell disease is an inherited disease, not an autoimmune reaction. Elevated serum bilirubin concentrations are associated with jaundice, not sickle cell disease. 29. Answer: 4. Sickle cell disease is an inherited disease that is present at birth. However, 60% to 80% of a newborns hemoglobin is fetal hemoglobin, which has a structure different from that of hemoglobin S or hemoglobin A. Sickle cell symptoms usually occur about 4 months after birth, when hemoglobin S begins to replace the fetal hemoglobin. The gene for sickle cell disease is transmitted at the time of conception, not passed through the placenta. Some hemoglobin S is produced by the fetus near term. The fetus produces all its own hemoglobin from the earliest production in the first trimester. Passive immunity conferred by maternal antibodies is not related to sickle cell disease, but this transmission of antibodies is important to protect the infant from various infections during early infancy. 30. Answer: 3. For the child in a sickle cell crisis, pain is the priority nursing diagnosis because the sickled cells clump and obstruct the blood vessels, leading to occlusion and subsequent tissue ischemia. Although ineffective coping may be important, it is not the priority. Decreased cardiac output is not a problem with this type of vaso occlusive crisis. Typically, a sickle cell crisis can be precipitated by a fluid volume deficit or dehydration. 31. Answer: 2. Children with iron-deficiency anemia are more susceptible to infection because of marked decreases in bone marrow functioning with microcytosis. 32. Answer: 1, 2. Toddlers should have between 2 and 3 cups of milk per day and 8 ounces of juice per day. If they have more than that, then they are probably not eating enough other foods, including iron-rich foods that have the needed nutrients. 33. Answer: 2. Potato, peas, chicken, green vegetables, and rice cereal contain significant amounts of iron and therefore would be recommended. Milk and yellow vegetables are not good iron sources. Rice by itself also is not a good source of iron. 34. Answer: 4. PTT measures the activity of thromboplastin, which is dependent on intrinsic clotting factors. In hemophilia, the intrinsic clotting factor VIII (antihemophilic factor) is deficient, resulting in a prolonged PTT. Bleeding time reflects platelet

22

function; the tourniquet test measures vasoconstriction and platelet function; and the clot retraction test measures capillary fragility. All of these are unaffected in people with hemophilia. 35. Answer: 1. Bleeding into the joints in the child with hemophilia leads to pain and tenderness, resulting in restricted movement. Therefore, an early sign of hemarthrosis would be the childs reluctance to move a body part. If the bleeding into the joint continues, the area becomes hot, swollen, and immobilenot cool, pale, and clammy. Ecchymosis formation around a joint would be difficult to assess. Instability of a long bone on passive movement is not associated with joint hemarthrosis. 36. Answer: 1. Because factor VIII concentrate is derived from large pools of human plasma, the risk of hepatitis is always present. Clinical manifestations of hepatitis include yellowing of the skin, mucous membranes, and sclera. Use of factor VIII concentrate is not associated with constipation, abdominal distention, or puffiness around the eyes. 37. Answer: 3. A diagnosis is established based on a complete blood count, examination for sickled red blood cells in the peripheral smear, and hemoglobin electrophoresis. Laboratory studies will show decreased hemoglobin and hematocrit levels and a decreased platelet count, and increased reticulocyte count, and the presence of nucleated red blood cells. Increased reticulocyte counts occur in children with sickle cell disease because the life span of their sickled red blood cells is shortened. 38. Answer: 3. Pain crisis may be precipitated by infection, dehydration, hypoxia, trauma, or physical or emotional stress. The mother of a child with sickle cell disease should encourage fluid intake of 1 to 2 times the daily requirement to prevent dehydration. 39. Answer: 4. The results of a CBC in children with iron deficiency anemia will show decreased hemoglobin levels and microcytic and hypochromic red blood cells. The red blood cell count is decreased. The reticulocyte count is usually normal or slightly elevated. 40. Answer: 4. Males inherit hemophilia from their mothers, and females inherit the carrier status from their fathers. Hemophilia is inherited in a recessive manner via a genetic defect on the X-chromosome. Hemophilia A results from a deficiency of factor VIII. Hemophilia B (Christmas disease) is a deficiency of factor IX.

Introduction
In this 20-item NCLEX style exam, you knowledge about the different Peripheral Vascular Diseases will be challenged.

Topics

Peripheral Vascular Diseases Heparin and Warfarin Deep Vein Thrombosis

Guidelines
Read each question carefully and choose the best answer. You are given one minute per question. Spend your time wisely! Answers and rationales are given below. Be sure to read them. If you need more clarifications, please direct them to the comments section.

Questions
1. The most important factor in regulating the caliber of blood vessels, which determines resistance to flow, is: 1. Hormonal secretion 2. Independent arterial wall activity. 3. The influence of circulating chemicals 4. The sympathetic nervous system 2. With peripheral arterial insufficiency, leg pain during rest can be reduced by:

23

1. Elevating the limb above heart level 2. Lowering the limb so it is dependent 3. Massaging the limb after application of cold compresses 4. Placing the limb in a plane horizontal to the body 3. Buergers disease is characterized by all of the following except: 1. Arterial thrombosis formation and occlusion 2. Lipid deposits in the arteries 3. Redness or cyanosis in the limb when it is dependent 4. Venous inflammation and occlusion 4. A significant cause of venous thrombosis is: 1. Altered blood coagulation 2. Stasis of blood 3. Vessel wall injury 4. All of the above 5. When caring for a patient who has started anticoagulant therapy with warfarin (Coumadin), the nurse knows not to expect therapeutic benefits for: 1. At least 12 hours 2. The first 24 hours 3. 2-3 days 4. 1 week 6. Mike, a 43-year old construction worker, has a history of hypertension. He smokes two packs of cigarettes a day, is nervous about the possibility of being unemployed, and has difficulty coping with stress. His current concern is calf pain during minimal exercise that decreased with rest. The nurse assesses Mikes symptoms as being associated with peripheral arterial occlusive disease. The nursing diagnosis is probably: 1. Alteration in tissue perfusion related to compromised circulation 2. Dysfunctional use of extremities related to muscle spasms 3. Impaired mobility related to stress associated with pain 4. Impairment in muscle use associated with pain on exertion. 7. A 24-year old man seeks medical attention for complaints of claudication in the arch of the foot. A nurse also notes superficial thrombophlebitis of the lower leg. The nurse would next assess the client for: 1. Familial tendency toward peripheral vascular disease 2. Smoking history 3. Recent exposures to allergens 4. History of insect bites 8. Intravenous heparin therapy is ordered for a client. While implementing this order, a nurse ensures that which of the following medications is available on the nursing unit? 1. Vitamin K 2. Aminocaproic acid 3. Potassium chloride 4. Protamine sulfate 9. A client who has been receiving heparin therapy also is started on warfarin sodium (coumadin). The client asks the nurse why both medications are being administered. In formulating a response, the nurse incorporates the understanding that warfarin sodium: 1. Stimulates the breakdown of specific clotting factors by the liver, and it takes 2-3 days for this is exhibit an anticoagulant effect. 2. Inhibits synthesis of specific clotting factors in the liver, and it takes 3 to 4 days for this medication to exert an anticoagulation effect.

24

3. Stimulates production of the bodys own thrombolytic substances, but it takes 2 -4 days for it to begin. 4. Has the same mechanism action of heparin, and the crossover time is needed for the serum level of warfarin sodium to be therapeutic. 10. A nurse has an order to begin administering warfarin sodium (coumadin) to a client. While implementing this order, the nurse ensures that which of the following medications is available on the nursing unit as the antidote for Coumadin? 1. Vitamin K 2. Aminocaproic acid 3. Potassium chloride 4. Protamine sulfate 11. A nurse is assessing the neurovascular status of a client who returned to the surgical nursing unit 4 hours ago after undergoing aortoiliac bypass graft. The affected leg is warm, and the nurse notes redness and edema. The pedal pulse is palpable and unchanged from admission. The nurse interprets that the neurovascular status is: 1. Normal because of the increased blood flow through the leg 2. Slightly deteriorating and should be monitored for another hour 3. Moderately impaired, and the surgeon should be called. 4. Adequate from the arterial approach, but venous complications are arising. 12. A client is admitted with a venous stasis leg ulcer. A nurse assesses the ulcer, expecting to note that the ulcer: 1. Has a pale colored base 2. Is deep, with even edges 3. Has little granulation tissue 4. Has brown pigmentation around it. 13. In preparation for discharge of a client with arterial insufficiency and Raynauds disease, client teaching instructions should include: 1. Walking several times each day as an exercise program. 2. Keeping the heat up so that the environment is warm 3. Wearing TED hose during the day 4. Using hydrotherapy for increasing oxygenation 14. A client comes to the outpatient clinic and tells the nurse that he has had legs pains that began when he walks but cease when he stops walking. Which of the following conditions would the nurse assess for? 1. An acute obstruction in the vessels of the legs 2. Peripheral vascular problems in both legs 3. Diabetes 4. Calcium deficiency 15. Which of the following characteristics is typical of the pain associated with DVT? 1. Dull ache 2. No pain 3. Sudden onset 4. Tingling 16. Cancer can cause changes in what component of Virchows triad? 1. Blood coagulability 2. Vessel walls 3. Blood flow 4. Blood viscosity 17. Varicose veins can cause changes in what component of Virchows triad? 1. Blood coagulability 2. Vessel walls

25

3. Blood flow 4. Blood viscosity 18. Which technique is considered the gold standard for diagnosing DVT? 1. Ultrasound imaging 2. Venography 3. MRI 4. Doppler flow study 19. A nurse is assessing a client with an abdominal aortic aneurysm. Which of the following assessment findings by the nurse is probably unrelated to the aneurysm? 1. Pulsatile abdominal mass 2. Hyperactive bowel sounds in that area 3. Systolic bruit over the area of the mass 4. Subjective sensation of heart beating in the abdomen. 20. A nurse is caring for a client who had a percutaneous insertion of an inferior vena cava filter and was on heparin therapy before surgery. The nurse would inspect the surgical site most closely for signs of: 1. Thrombosis and infection 2. Bleeding and infection 3. Bleeding and wound dehiscence. 4. Wound dehiscence and evisceration.

Answers and Rationale


1. Answer: 4 2. Answer: 2 3. Answer: 2 4. Answer: 4 5. Answer: 3 6. Answer: 1 7. Answer: 2. The mixture of arterial and venous manifestations (claudication and phlebitis, respectively) in the young male client suggests Buergers disease. This is an uncommon disorder characterized by inflammation and thrombosis of smaller arteries and veins. This disorder typically is found in young adult males who smoke. The cause is not known precisely but is suspected to have an autoimmune component. 8. Answer: 4. The antidote to heparin is protamine sulfate and should be readily available for use if excessive bleeding or hemorrhage should occur 9. Answer: 2. Warfarin sodium works in the liver and inhibits synthesis of four vitamin K-dependent clotting factors (X, IX, VII, and II), but it takes 3 to 4 days before the therapeutic effect of warfarin is exhibited. 10. Answer: 1. The antidote to warfarin (Coumadin) is Vitamin K and should be readily available for use if excessive bleeding or hemorrhage should occur. 11. Answer: 1. An expected outcome of surgery is warmth, redness, and edema in the surgical extremity because of increased blood flow. Options 2, 3, and 4 are incorrect interpretations. 12. Answer: 4. Venous leg ulcers, also called stasis ulcers, tend to be more superficial than arterial ulcers, and the ulcer bed is pink. The edges of the ulcer are uneven, and granulation tissue is evident. The skin has a brown pigmentation from accumulation of metabolic waste products resulting from venous stasis. The client also exhibits peripheral edema. (options 1, 2, and 3 is due to tissue malnutrition; and thus us an arterial problem) 13. Answer: 2. The clients instructions should include keeping the environment warm to prevent vasoconstriction. Wearing gloves, warm clothes, and socks will also be useful when preventing vasoconstriction, but TED hose would not be therapeutic. Walking would most likely increase pain.

26

14. Answer: 2. Intermittent claudication is a condition that indicates vascular deficiencies in the peripheral vascular system. If an obstruction were present, the leg pain would persist when the client stops walking. Low calcium levels may cause leg cramps but would not necessarily be related to walking. 15. Answer: 3. DVT is associated with deep leg pain of sudden onset, which occurs secondary to the occlusion. A dull ache is more commonly associated with varicose veins. A tingling sensation is associated with an alteration in arterial blood flow. If the thrombus is large enough, it will cause pain. 16. Answer: 1 17. Answer: 3 18. Answer: 2 19. Answer: 2. Not all clients with abdominal aortic aneurysms exhibit symptoms. Those who do describe a feeling of the hear t beating in the abdomen when supine or be able to feel the mass throbbing. A pulsati le mass may be palpated in the middle and upper abdomen. A systolic bruit may be auscultated over the mass. Hyperactive bowel sounds are not related specifically to an abdominal aortic aneurysm. 20. Answer: 2. After inferior vena cava insertion, the nurse inspects the surgical site for bleeding and signs and symptoms of infection. Otherwise, care is the same as for any post-op client.

Introduction
A simple 10 item NCLEX style examination about Valvular Diseases.

Topics

Valvular Diseases

Guidelines
Read each question carefully and choose the best answer. You are given one minute per question. Spend your time wisely! Answers and rationales are given below. Be sure to read them. If you need more clarifications, please direct them to the comments section.

Questions
1. A 68-year-old woman is scheduled to undergo mitral valve replacement for severe mitral stenosis and mitral regurgitation. Although the diagnosis was made during childhood, she did not have any symptoms until 4 years ago. Recently, she noticed increased symptoms, despite daily doses of digoxin and furosemide. During the initial interview with the nice lady, the nurse would most likely learn that the clients childhood health history included: 1. Chicken pox 2. poliomyelitis 3. Rheumatic fever 4. meningitis 2. Which of the following signs and symptoms would most likely be found in a client with mitral regurgitation? 1. Exertional dyspnea 2. Confusion 3. Elevated creatine phosphokinase concentration 4. Chest pain 3. The nurse expects that a client with mitral stenosis would demonstrate symptoms associated with congestion in the: 1. Aorta 2. Right atrium 3. Superior vena cava 4. Pulmonary circulation

27

4. Because a client has mitral stenosis and is a prospective valve recipient, the nurse preoperatively assesses the clients past compliance with medical regimens. Lack of compliance with which of the following regimens would pose the greatest health hazard to this client? 1. Medication therapy 2. Diet modification 3. Activity restrictions 4. Dental care 5. Good dental care is an important measure in reducing the risk of endocarditis. A teaching plan to promote good dental care in a client with mitral stenosis should include demonstration of the proper use of: 1. A manual toothbrush 2. An electric toothbrush 3. An irrigation device 4. Dental floss 6. A client has been admitted to the hospital with a diagnosis of suspected bacterial endocarditis. The complication the nurse will constantly observe for is: 1. Presence of heart murmur 2. Systemic emboli 3. Fever 4. Congestive heart failure 7. Cholesterol, frequently discussed in relation to atherosclerosis, is a substance that: 1. May be controlled by eliminating food sources 2. Is found in many foods, both plant and animal sources 3. All persons would be better off without because it causes the disease process 4. Circulates in the blood, the level of which usually decreases when unsaturated fats are substituted for saturated fats. 8. When teaching a client with a cardiac problem, who is on a high-unsaturated fatty-acid diet, the nurse should stress the importance of increasing the intake of: 1. Enriched whole milk 2. Red meats, such as beef 3. Vegetables and whole grains 4. Liver and other glandular organ meats 9. A 2-gram sodium diet is prescribed for a client with severe hypertension. The client does not like the diet, and the nurse hears the client request that the spouse Bring in some good home-cooked food. It would be most effective for the nurse to plan to: 1. Call in the dietician for client teaching 2. Wait for the clients family and discuss the diet with the client and family 3. Tell the client that the use of salt is forbidden, because it will raise BP 4. Catch the family members before they go into the clients room and tell them about the diet. 10. What criteria should the nurse use to determine normal sinus rhythm for a client on a cardiac monitor? Check all that apply. 1. The RR intervals are relatively consistent 2. One P wave precedes each QRS complex 3. Four to eight complexes occur in a 6-second strip 4. The ST segment is higher than the PR interval 5. The QRS complex ranges from 0.12 to 0.2 seconds

Answers and Rationale


Gauge your performance by counter checking your answers to the answers below. Learn more about the question by reading the rationale. If you have any disputes or questions, please direct them to the comments section.

28

1. Answer: 3. Most clients with mitral stenosis have a history of rheumatic fever or bacterial endocarditis. 2. Answer: 1. Weight gain, due to fluid retention and worsening heart failure, causes exertional dyspnea in clients with mitral regurgitation. The rise in left atrial pressure that accompanies mitral valve disease is transmitted backward into pulmonary veins, capillaries, and arterioles and eventually to the right ventricle. Signs and symptoms of pulmonary and systemic venous congestion follow. 3. Answer: 4. When mitral stenosis is present, the left atrium has difficulty emptying its contents into the left ventricle. Hence, because there is no valve to prevent backward flow into the pulmonary vein, the pulmonary circulation is under pressure. 4. Answer: 1. Preoperatively, anticoagulants may be prescribed for the client with advanced valvular heart disease to prevent emboli. Post-op, all clients with mechanical valves and some with bioprostheses are maintained indefinitely on anticoagulation therapy. Adhering strictly to a dosage schedule and observing specific precautions are necessary to prevent hemorrhage or thromboembolism. Some clients are maintained on lifelong antibiotic prophylaxis to prevent recurrence from rheumatic fever. Episodic prophylaxis is required to prevent infective endocarditis after dental procedures or upper respiratory, GI, or GU surgery. 5. Answer: 1. Daily dental care and frequent checkups by a d entist who is informed about the clients condition are required to maintain good oral health. Use of an electric toothbrush, an irrigation device, or dental floss may cause gums to bleed and allow bacteria to enter mucous membranes and the bloodstream, increasing the risk of endocarditis. 6. Answer: 2. Emboli are the major problem; those arising in the right heart chambers will terminate in the lungs and left chamber emboli may travel anywhere in the arteries. Heart murmurs, fever, and night sweats may be present, but do not indicate a problem with emboli. CHF may be a result, but this is not as dangerous an outcome as emboli. 7. Answer: 4. Cholesterol is a sterol found in tissue; it is attributed in part to diets high in saturated fats. 8. Answer: 3. Vegetables and whole grains are low in fat and may reduce the risk for heart disease. 9. Answer: 2. Clients families should be included in dietary teaching; families provide support that promotes adherence. 10. Answers: 1 and 2. (1) The consistency of the RR interval indicates a regular rhythm. (2) A normal P wave before each complex indicates the impulse originated in the SA node. (3) The number of complexes in a 6-second strip is multiplied by 10 to approximate the heart rate; normal sinus rhythm is 60 to 100. (4) Elevation of the ST segment is a sign of cardiac ischemia and is unrelated to the rhythm. (5) The QRS duration should be less than 0.12 seconds; the PR interval should be 0.12 to 0.2 seconds.

This NCLEX exam has 80 questions that covers the diseases of the Gastrointestinal and Digestive System.

Topics

Liver Failure Liver Disorders Ostomy Care

Guidelines
Read each question carefully and choose the best answer. You are given one minute per question. Spend your time wisely! Answers and rationales are given below. Be sure to read them. If you need more clarifications, please direct them to the comments section.

Questions
1. A patient with chronic alcohol abuse is admitted with liver failure. You closely monitor the patients blood pressure because of which change that is associated with the liver failure?

29

1. Hypoalbuminemia 2. Increased capillary permeability 3. Abnormal peripheral vasodilation 4. Excess rennin release from the kidneys 2. Youre assessing the stoma of a patient with a healthy, well-healed colostomy. You expect the stoma to appear: 1. Pale, pink and moist 2. Red and moist 3. Dark or purple colored 4. Dry and black 3. Youre caring for a patient with a sigmoid colostomy. The stool from this colostomy is: 1. Formed 2. Semisolid 3. Semiliquid 4. Watery 4. Youre advising a 21 y.o. with a colostomy who reports problems with flatus. What food should you recommend? 1. Peas 2. Cabbage 3. Broccoli 4. Yogurt 5. You have to teach ostomy self care to a patient with a colostomy. You tell the patient to measure and cut the wafer: 1. To the exact size of the stoma. 2. About 1/16 larger than the stoma. 3. About 1/8 larger than the stoma. 4. About 1/4 larger than the stoma. 6. Youre performing an abdominal assessment on Brent who is 52 y.o. In which order do you proceed? 1. Observation, percussion, palpation, auscultation 2. Observation, auscultation, percussion, palpation 3. Percussion, palpation, auscultation, observation 4. Palpation, percussion, observation, auscultation 7. Youre doing preoperative teaching with Gertrude who has ulcerative colitis who needs surgery to create an ileoanal reservoir. Which information do you include? 1. A reservoir is created that exits through the abdominal wall. 2. A second surgery is required 12 months after the first surgery. 3. A permanent ileostomy is created. 4. The surgery occurs in two stages. 8. Youre caring for Carin who has just had ileostomy surgery. During the first 24 hours post-op, how much drainage can you expect from the ileostomy? 1. 100 ml 2. 500 ml 3. 1500 ml 4. 5000 ml 9. Youre preparing a teaching plan for a 27 y.o. named Jeff who underwent surgery to close a temporary ileostomy. Which nutritional guideline do you include in this plan? 1. There is no need to change eating habits. 2. Eat six small meals a day. 3. Eat the largest meal in the evening. 4. Restrict fluid intake.

30

10. Arthur has a family history of colon cancer and is scheduled to have a sigmoidoscopy. He is crying as he tells you, I know that I have colon cancer, too. Which response is most therapeutic? 1. I know just how you feel. 2. You seem upset. 3. Oh, dont worry about it, everything will be just fine. 4. Why do you think you have cancer? 11. Youre caring for Beth who underwent a Billroth II procedure (surgical removal of the pylorus and duodenum) for treatment of a peptic ulcer. Which findings suggest that the patient is developing dumping syndrome, a complication associated with this procedure? 1. Flushed, dry skin. 2. Headache and bradycardia. 3. Dizziness and sweating. 4. Dyspnea and chest pain. 12. Youre developing the plan of care for a patient experiencing dumping syndrome after a Billroth II procedure. Which dietary instructions do you include? 1. Omit fluids with meals. 2. Increase carbohydrate intake. 3. Decrease protein intake. 4. Decrease fat intake. 13. Youre caring for Lewis, a 67 y.o. patient with liver cirrhosis who developed ascites and requires paracentesis. Relief of which symptom indicated that the paracentesis was effective? 1. Pruritus 2. Dyspnea 3. Jaundice 4. Peripheral Neuropathy 14. Youre caring for Jane, a 57 y.o. patient with liver cirrhosis who developed ascites and requires paracentesis. Before her paracentesis, you instruct her to: 1. Empty her bladder. 2. Lie supine in bed. 3. Remain NPO for 4 hours. 4. Clean her bowels with an enema. 15. After abdominal surgery, your patient has a severe coughing episode that causes wound evisceration. In addition to calling the doctor, which intervention is most appropriate? 1. Irrigate the wound & organs with Betadine. 2. Cover the wound with a saline soaked sterile dressing. 3. Apply a dry sterile dressing & binder. 4. Push the organs back & cover with moist sterile dressings. 16. Youre caring for Betty with liver cirrhosis. Which of the following assessment findings leads you to suspect hepatic encephalopathy in her? 1. Asterixis 2. Chvosteks sign 3. Trousseaus sign 4. Hepatojugular reflex 17. You are developing a care plan on Sally, a 67 y.o. patient with hepatic encephalopathy. Which of the following do you include? 1. Administering a lactulose enema as ordered. 2. Encouraging a protein-rich diet.

31

3. Administering sedatives, as necessary. 4. Encouraging ambulation at least four times a day. 18. You have a patient with achalasia (incomplete muscle relaxtion of the GI tract, especially sphincter muscles). Which medications do you anticipate to administer? 1. Isosorbide dinitrate (Isordil) 2. Digoxin (Lanoxin) 3. Captopril (Capoten) 4. Propranolol (Inderal) 19. The student nurse is preparing a teaching care plan to help improve nutrition in a patient with achalasia. You include which of the following: 1. Swallow foods while leaning forward. 2. Omit fluids at mealtimes. 3. Eat meals sitting upright. 4. Avoid soft and semi soft foods. 20. Britney, a 20 y.o. student is admitted with acute pancreatitis. Which laboratory findings do you expect to be abnormal for this patient? 1. Serum creatinine and BUN 2. Alanine aminotransferase (ALT) and aspartate aminotransferase (AST) 3. Serum amylase and lipase 4. Cardiac enzymes 21. A patient with Crohns disease is admitted after 4 days of diarrhea. Which of the following urine specific gravity values do you expect to find in this patient? 1. 1.005 2. 1.011 3. 1.020 4. 1.030 22. Your goal is to minimize Davids risk of complications after a herniorrhaphy. You instruct the patient to: 1. Avoid the use of pain medication. 2. Cough and deep breathe Q2H. 3. Splint the incision if he cant avoid sneezing or coughing. 4. Apply heat to scrotal swelling. 23. Janice is waiting for discharge instructions after her herniorrhaphy. Which of the following instructions do you include? 1. Eat a low-fiber diet. 2. Resume heavy lifting in 2 weeks. 3. Lose weight, if obese. 4. Resume sexual activity once discomfort is gone. 24. Develop a teaching care plan for Angie who is about to undergo a liver biopsy. Which of the following points do you include? 1. Youll need to lie on your stomach during the test. 2. Youll need to lie on your right side after the test. 3. During the biopsy youll be asked to exhale deeply and hold it. 4. The biopsy is performed under general anesthesia. 25. Stephen is a 62 y.o. patient that has had a liver biopsy. Which of the following groups of signs alert you to a possible pneumothorax? 1. Dyspnea and reduced or absent breath sounds over the right lung 2. Tachycardia, hypotension, and cool, clammy skin

32

3. Fever, rebound tenderness, and abdominal rigidity 4. Redness, warmth, and drainage at the biopsy site 26. Michael, a 42 y.o. man is admitted to the med-surg floor with a diagnosis of acute pancreatitis. His BP is 136/76, pulse 96, Resps 22 and temp 101. His past history includes hyperlipidemia and alcohol abuse. The doctor prescribes an NG tube. Before inserting the tube, you explain the purpose to patient. Which of the following is a most accurate explanation? 1. It empties the stomach of fluids and gas. 2. It prevents spasms at the sphincter of Oddi. 3. It prevents air from forming in the small intestine and large intestine. 4. It removes bile from the gallbladder. 27. Jason, a 22 y.o. accident victim, requires an NG tube for feeding. What should you immediately do after inserting an NG tube for liquid enteral feedings? 1. Aspirate for gastric secretions with a syringe. 2. Begin feeding slowly to prevent cramping. 3. Get an X-ray of the tip of the tube within 24 hours. 4. Clamp off the tube until the feedings begin. 28. Stephanie, a 28 y.o. accident victim, requires TPN. The rationale for TPN is to provide: 1. Necessary fluids and electrolytes to the body. 2. Complete nutrition by the I.V. route. 3. Tube feedings for nutritional supplementation. 4. Dietary supplementation with liquid protein given between meals. 29. Type A chronic gastritis can be distinguished from type B by its ability to: 1. Cause atrophy of the parietal cells. 2. Affect only the antrum of the stomach. 3. Thin the lining of the stomach walls. 4. Decrease gastric secretions. 30. Matt is a 49 y.o. with a hiatal hernia that you are about to counsel. Health care counseling for Matt should include which of the following instructions? 1. Restrict intake of high-carbohydrate foods. 2. Increase fluid intake with meals. 3. Increase fat intake. 4. Eat three regular meals a day. 31. Jerod is experiencing an acute episode of ulcerative colitis. Which is priority for this patient? 1. Replace lost fluid and sodium. 2. Monitor for increased serum glucose level from steroid therapy. 3. Restrict the dietary intake of foods high in potassium. 4. Note any change in the color and consistency of stools. 32. A 29 y.o. patient has an acute episode of ulcerative colitis. What diagnostic test confirms this diagnosis? 1. Barium Swallow. 2. Stool examination. 3. Gastric analysis. 4. Sigmoidoscopy. 33. Eleanor, a 62 y.o. woman with diverticulosis is your patient. Which interventions would you expect to include in her care? 1. Low-fiber diet and fluid restrictions. 2. Total parenteral nutrition and bed rest. 3. High-fiber diet and administration of psyllium. 4. Administration of analgesics and antacids.

33

34. Regina is a 46 y.o. woman with ulcerative colitis. You expect her stools to look like: 1. Watery and frothy. 2. Bloody and mucous. 3. Firm and well-formed. 4. Alternating constipation and diarrhea. 35. Donald is a 61 y.o. man with diverticulitis. Diverticulitis is characterized by: 1. Periodic rectal hemorrhage. 2. Hypertension and tachycardia. 3. Vomiting and elevated temperature. 4. Crampy and lower left quadrant pain and low-grade fever. 36. Brenda, a 36 y.o. patient is on your floor with acute pancreatitis. Treatment for her includes: 1. Continuous peritoneal lavage. 2. Regular diet with increased fat. 3. Nutritional support with TPN. 4. Insertion of a T tube to drain the pancreas. 37. Glenda has cholelithiasis (gallstones). You expect her to complain of: 1. Pain in the right upper quadrant, radiating to the shoulder. 2. Pain in the right lower quadrant, with rebound tenderness. 3. Pain in the left upper quadrant, with shortness of breath. 4. Pain in the left lower quadrant, with mild cramping. 38. After an abdominal resection for colon cancer, Madeline returns to her room with a Jackson-Pratt drain in place. The purpose of the drain is to: 1. Irrigate the incision with a saline solution. 2. Prevent bacterial infection of the incision. 3. Measure the amount of fluid lost after surgery. 4. Prevent accumulation of drainage in the wound. 39. Anthony, a 60 y.o. patient, has just undergone a bowel resection with a colostomy. During the first 24 hours, which of the following observations about the stoma should you report to the doctor? 1. Pink color. 2. Light edema. 3. Small amount of oozing. 4. Trickles of bright red blood. 40. Your teaching Anthony how to use his new colostomy. How much skin should remain exposed between the stoma and the ring of the appliance? 1. 1/16 2. 1/4 3. 1/2 4. 1 41. Claire, a 33 y.o. is on your floor with a possible bowel obstruction. Which intervention is priority for her? 1. Obtain daily weights. 2. Measure abdominal girth. 3. Keep strict intake and output. 4. Encourage her to increase fluids. 42. Your patient has a GI tract that is functioning, but has the inability to swallow foods. Which is the preferred method of feeding for your patient? 1. TPN 2. PPN

34

3. NG feeding 4. Oral liquid supplements 43. Youre patient is complaining of abdominal pain during assessment. What is your priority? 1. Auscultate to determine changes in bowel sounds. 2. Observe the contour of the abdomen. 3. Palpate the abdomen for a mass. 4. Percuss the abdomen to determine if fluid is present. 44. Before bowel surgery, Lee is to administer enemas until clear. During administration, he complains of intestinal cramps. What do you do next? 1. Discontinue the procedure. 2. Lower the height of the enema container. 3. Complete the procedure as quickly as possible. 4. Continue administration of the enema as ordered without making any adjustments. 45. Leigh Ann is receiving pancrelipase (Viokase) for chronic pancreatitis. Which observation best indicates the treatment is effective? 1. There is no skin breakdown. 2. Her appetite improves. 3. She loses more than 10 lbs. 4. Stools are less fatty and decreased in frequency. 46. Ralph has a history of alcohol abuse and has acute pancreatitis. Which lab value is most likely to be elevated? 1. Calcium 2. Glucose 3. Magnesium 4. Potassium 47. Anna is 45 y.o. and has a bleeding ulcer. Despite multiple blood transfusions, her HGB is 7.5g/dl and HCT is 27%. Her doctor determines that surgical intervention is necessary and she undergoes partial gastrectomy. Postoperative nursing care includes: 1. Giving pain medication Q6H. 2. Flushing the NG tube with sterile water. 3. Positioning her in high Fowlers position. 4. Keeping her NPO until the return of peristalsis. 48. Sitty, a 66 y.o. patient underwent a colostomy for ruptured diverticulum. She did well during the surgery and returned to your med-surg floor in stable condition. You assess her colostomy 2 days after surgery. Which finding do you report to the doctor? 1. Blanched stoma 2. Edematous stoma 3. Reddish-pink stoma 4. Brownish-black stoma 49. Sharon has cirrhosis of the liver and develops ascites. What intervention is necessary to decrease the excessive accumulation of serous fluid in her peritoneal cavity? 1. Restrict fluids 2. Encourage ambulation 3. Increase sodium in the diet 4. Give antacids as prescribed 50. Katrina is diagnosed with lactose intolerance. To avoid complications with lack of calcium in the diet, which food should be included in the diet?

35

1. Fruit 2. Whole grains 3. Milk and cheese products 4. Dark green, leafy vegetables 51. Nathaniel has severe pruritus due to having hepatitis B. What is the best intervention for his comfort? 1. Give tepid baths. 2. Avoid lotions and creams. 3. Use hot water to increase vasodilation. 4. Use cold water to decrease the itching. 52. Rob is a 46 y.o. admitted to the hospital with a suspected diagnosis of Hepatitis B. Hes jaundiced and reports weakness. Which intervention will you include in his care? 1. Regular exercise. 2. A low-protein diet. 3. Allow patient to select his meals. 4. Rest period after small, frequent meals. 53. Youre discharging Nathaniel with hepatitis B. Which statement suggests understanding by the patient? 1. Now I can never get hepatitis again. 2. I can safely give blood after 3 months. 3. Ill never have a problem with my liver again, even if I drink alcohol. 4. My family knows that if I get tired and start vomiting, I may be getting sick again. 54. Gail is scheduled for a cholecystectomy. After completion of preoperative teaching, Gail states,If I lie still and avoid turning after the operation, Ill avoid pain. Do you think this is a good idea? What is the best response? 1. Youll need to turn from side to side every 2 hours. 2. Its always a good idea to rest quietly after surgery. 3. The doctor will probably order you to lie flat for 24 hours. 4. Why dont you decide about activity after you return from the recovery room? 55. Youre caring for a 28 y.o. woman with hepatitis B. Shes concerned about the duration of her recovery. Which response isnt appropriate? 1. Encourage her to not worry about the future. 2. Encourage her to express her feelings about the illness. 3. Discuss the effects of hepatitis B on future health problems. 4. Provide avenues for financial counseling if she expresses the need. 56. Elmer is scheduled for a proctoscopy and has an I.V. The doctor wrote an order for 5mg of I.V. diazepam(Valium). Which order is correct regarding diazepam? 1. Give diazepam in the I.V. port closest to the vein. 2. Mix diazepam with 50 ml of dextrose 5% in water and give over 15 minutes. 3. Give diazepam rapidly I.V. to prevent the bloodstream from diluting the drug mixture. 4. Question the order because I.V. administration of diazepam is contraindicated. 57. Annabelle is being discharged with a colostomy, and youre teaching her about colostomy care. Which statement correctly describes a healthy stoma? 1. At first, the stoma may bleed slightly when touched. 2. The stoma should appear dark and have a bluish hue. 3. A burning sensation under the stoma faceplate is normal. 4. The stoma should remain swollen away from the abdomen. 58. A patient who underwent abdominal surgery now has a gaping incision due to delayed wound healing. Which method is correct when you irrigate a gaping abdominal incision with sterile normal saline solution, using a piston syringe?

36

1. Rapidly instill a stream of irrigating solution into the wound. 2. Apply a wet-to-dry dressing to the wound after the irrigation. 3. Moisten the area around the wound with normal saline solution after the irrigation. 4. Irrigate continuously until the solution becomes clear or all of the solution is used. 59. Hepatic encephalopathy develops when the blood level of which substance increases? 1. Ammonia 2. Amylase 3. Calcium 4. Potassium 60. Your patient recently had abdominal surgery and tells you that he feels a popping sensation in his incision during a coughing spell, followed by severe pain. You anticipate an evisceration. Which supplies should you take to his room? 1. A suture kit. 2. Sterile water and a suture kit. 3. Sterile water and sterile dressings. 4. Sterile saline solution and sterile dressings. 61. Findings during an endoscopic exam include a cobblestone appearance of the colon in your patient. The findings are characteristic of which disorder? 1. Ulcer 2. Crohns disease 3. Chronic gastritis 4. Ulcerative colitis 62. What information is correct about stomach cancer? 1. Stomach pain is often a late symptom. 2. Surgery is often a successful treatment. 3. Chemotherapy and radiation are often successful treatments. 4. The patient can survive for an extended time with TPN. 63. Dark, tarry stools indicate bleeding in which location of the GI tract? 1. Upper colon. 2. Lower colon. 3. Upper GI tract. 4. Small intestine. 64. A patient has an acute upper GI hemorrhage. Your interventions include: 1. Treating hypovolemia. 2. Treating hypervolemia. 3. Controlling the bleeding source. 4. Treating shock and diagnosing the bleeding source. 65. You promote hemodynamic stability in a patient with upper GI bleeding by: 1. Encouraging oral fluid intake. 2. Monitoring central venous pressure. 3. Monitoring laboratory test results and vital signs. 4. Giving blood, electrolyte and fluid replacement. 66. Youre preparing a patient with a malignant tumor for colorectal surgery and subsequent colostomy. The patient tells you hes anxious. What should your initial step be in working with this patient? 1. Determine what the patient already knows about colostomies. 2. Show the patient some pictures of colostomies. 3. Arrange for someone who has a colostomy to visit the patient. 4. Provide the patient with written material about colostomy care.

37

67. Your patient, Christopher, has a diagnosis of ulcerative colitis and has severe abdominal pain aggravated by movement, rebound tenderness, fever, nausea, and decreased urine output. This may indicate which complication? 1. Fistula. 2. Bowel perforation. 3. Bowel obstruction. 4. Abscess. 68. A patient has a severe exacerbation of ulcerative colitis. Long-term medications will probably include: 1. Antacids. 2. Antibiotics. 3. Corticosteroids. 4. Histamine2-receptor blockers. 69. The student nurse is teaching the family of a patient with liver failure. You instruct them to limit which foods in the patients diet? 1. Meats and beans. 2. Butter and gravies. 3. Potatoes and pastas. 4. Cakes and pastries. 70. An intubated patient is receiving continuous enteral feedings through a Salem sump tube at a rate of 60ml/hr. Gastric residuals have been 30-40ml when monitored Q4H. You check the gastric residual and aspirate 220ml. What is your first response to this finding? 1. Notify the doctor immediately. 2. Stop the feeding, and clamp the NG tube. 3. Discard the 220ml, and clamp the NG tube. 4. Give a prescribed GI stimulant such as metoclopramide (Reglan). 71. Your patient with peritonitis is NPO and complaining of thirst. What is your priority? 1. Increase the I.V. infusion rate. 2. Use diversion activities. 3. Provide frequent mouth care. 4. Give ice chips every 15 minutes. 72. Kevin has a history of peptic ulcer disease and vomits coffee-ground emesis. What does this indicate? 1. He has fresh, active upper GI bleeding. 2. He needs immediate saline gastric lavage. 3. His gastric bleeding occurred 2 hours earlier. 4. He needs a transfusion of packed RBCs. 73. A 53 y.o. patient has undergone a partial gastrectomy for adenocarcinoma of the stomach. An NG tube is in place and is connected to low continuous suction. During the immediate postoperative period, you expect the gastric secretions to be which color? 1. Brown. 2. Clear. 3. Red. 4. Yellow. 74. Your patient has a retractable gastric peptic ulcer and has had a gastric vagotomy. Which factor increases as a result of vagotomy? 1. Peristalsis. 2. Gastric acidity. 3. Gastric motility. 4. Gastric pH.

38

75. Christina is receiving an enteral feeding that requires a concentration of 80 ml of supplement mixed with 20 ml of water. How much water do you mix with an 8 oz (240ml) can of feeding? 1. 60 ml. 2. 70 ml. 3. 80 ml. 4. 90 ml. 76. Which stoma would you expect a malodorous, enzyme-rich, caustic liquid output that is yellow, green, or brown? 1. Ileostomy. 2. Ascending colostomy. 3. Transverse colostomy. 4. Descending colostomy. 77. George has a T tube in place after gallbladder surgery. Before discharge, what information or instructions should be given regarding the T tube drainage? 1. If there is any drainage, notify the surgeon immediately. 2. The drainage will decrease daily until the bile duct heals. 3. First, the drainage is dark green; then it becomes dark yellow. 4. If the drainage stops, milk the tube toward the puncture wound. 78. Your patient Maria takes NSAIDS for her degenerative joint disease, has developed peptic ulcer disease. Which drug is useful in preventing NSAID-induced peptic ulcer disease? 1. Calcium carbonate (Tums) 2. Famotidine (Pepcid) 3. Misoprostol (Cytotec) 4. Sucralfate (Carafate) 79. The student nurse is participating in colorectal cancer-screening program. Which patient has the fewest risk factors for colon cancer? 1. Janice, a 45 y.o. with a 25-year history of ulcerative colitis 2. George, a 50 y.o. whose father died of colon cancer 3. Herman, a 60 y.o. who follows a low-fat, high-fiber diet 4. Sissy, a 72 y.o. with a history of breast cancer 80. Youre patient, post-op drainage of a pelvic abscess secondary to diverticulitis, begins to cough violently after drinking water. His wound has ruptured and a small segment of the bowel is protruding. Whats your priority? 1. Ask the patient what happened, call the doctor, and cover the area with a water-soaked bedsheet. 2. Obtain vital signs, call the doctor, and obtain emergency orders. 3. Have a CAN hold the wound together while you obtain vital signs, call the doctor and flex the patients knees. 4. Have the doctor called while you remain with the patient, flex the patients knees, and cover the wound with sterile towels soaked in sterile saline solution.

Answers and Rationale


1. Answer: 1. Blood pressure decreases as the body is unable to maintain normal oncotic pressure with liver failure, so patients with liver failure require close blood pressure monitoring. Increased capillary permeability, abnormal peripheral vasodilation, and excess rennin released from the kidneys arent direct ramifications of liver failure. 2. Answer: 2. Good circulation causes tissues to be moist and red, so a healthy, well-healed stoma appears red and moist. 3. Answer: 1. A colostomy in the sigmoid colon produces a solid, formed stool. 4. Answer: 4. High-fiber foods stimulate peristalsis, and a result, flatus. Yogurt reduces gas formation. 5. Answer: 2. A proper fit protects the skin, but doesnt impair circulation. A 1/16 should be cut. 6. Answer: 2. Observation, auscultation, percussion, palpation

39

7. Answer: 4. An ileoanal reservoir is created in two stages. The two surgeries are about 2 to 3 months apart. First, diseased intestines are removed and a temporary loop ileostomy is created. Second, the loop ileostomy is closed and stool goes to the reservoir and out through the anus. 8. Answer: 3. The large intestine absorbs large amounts of water so the initial output from the ileostomy may be as much as 1500 to 2000 ml/24 hours. Gradually, the small intestine absorbs more fluid and the output decreases. 9. Answer: 2. To avoid overloading the small intestine, encourage the patient to eat six small, regularly spaced meals. 10. Answer: 2. Making observations about what you see or hear is a useful therapeutic technique. This way, you acknowledge that you are interested in what the patient is saying and feeling. 11. C After a Billroth II procedure, a large amount of hypertonic fluid enters the intestine. This causes extracellular fluid to move rapidly into the bowel, reducing circulating blood volume and producing vasomotor symptoms. Vasomotor symptoms produced by dumping syndrome include dizziness and sweating, tachycardia, syncope, pallor, and palpitations. 12. A Gastric emptying time can be delayed by omitting fluids from your patients meal. A diet low in carbs and high in fat & protein is recommended to treat dumping syndrome. 13. B Ascites puts pressure on the diaphragm. Paracentesis is done to remove fluid and reducing pressure on the diaphragm. The goal is to improve the patients breathing. The others are signs of cirrhosis that arent relieved by paracentesis. 14. A A full bladder can interfere with paracentesis and be punctured inadvertently. 15. B Cover the organs with a sterile, nonadherent dressing moistened with normal saline. Do this to prevent infection and to keep the organs from drying out. 16. A Asterixis is an early neurologic sign of hepatic encephalopathy elicited by asking the patient to hold her arms stretched out. Asterixis is present if the hands rapidly extend and flex. 17. A You may administer the laxative lactulose to reduce ammonia levels in the colon. 18. A Achalasia is characterized by incomplete relaxation of the LES, dilation of the lower esophagus, and a lack of esophageal peristalsis. Because nitrates relax the lower esophageal sphincter, expect to give Isordil orally or sublingually. 19. C Eating in the upright position aids in emptying the esophagus. Doing the opposite of the other three also may be helpful. 20. C Pancreatitis involves activation of pancreatic enzymes, such as amylase and lipase. These levels are elevated in a patient with acute pancreatitis. 21. D The normal range of specific gravity of urine is 1.010 to 1.025; a value of 1.030 may be seen with dehydration. 22. C Teach the pt to avoid activities that increase intra-abdominal pressure such as coughing, sneezing, or straining with a bowel movement. 23. C Because obesity weakens the abdominal muscles, advise weight loss for the patient who has had a hernia repair. 24. B After a liver biopsy, the patient is placed on the right side to compress the liver and to reduce the risk of bleeding or bile leakage. 25. A Signs and Symptoms of pneumothorax include dyspnea and decreased or absent breath sounds over the affected lung (right lung). 26. A An NG tube is inserted into the patients stomach to drain fluid and gas. 27. A Aspirating the stomach contents confirms correct placement. If an X-ray is ordered, it should be done immediately, not in 24 hours. 28. B TPN is given I.V. to provide all the nutrients your patient needs. TPN isnt a tube feeding nor is it a liquid dietary supplement. 29. A Type A causes changes in parietal cells. 30. B Increasing fluids helps empty the stomach. A high carb diet isnt restricted and fat intake shouldnt be increased. 31. A Diarrhea d/t an acute episode of ulcerative colitis leads to fluid & electrolyte losses so fluid replacement takes priority. 32. D Sigmoidoscopy allows direct observation of the colon mucosa for changes, and if needed, biopsy. 33. C She needs a high-fiber diet and a psyllium (bulk laxative) to promote normal soft stools. 34. B Stools from ulcerative colitis are often bloody and contain mucus. 35. D One sign of acute diverticulitis is crampy lower left quadrant pain. A low-grade fever is another common sign. 36. C With acute pancreatitis, you need to rest the GI tract by TPN as nutritional support.

40

37. A The gallbladder is located in the RUQ and a frequent sign of gallstones is pain radiating to the shoulder. 38. D A Jackson-Pratt drain promotes wound healing by allowing fluid to escape from the wound. 39. D After creation of a colostomy, expect to see a stoma that is pink, slightly edematous, with some oozing. Bright red blood, regardless of amount, indicates bleeding and should be reported to the doctor. 40. A Only a small amount of skin should be exposed and more than 1/16 of skin allows the excre tement to irritate the skin. 41. B Measuring abdominal girth provides quantitative information about increases or decreases in the amount of distention. 42. C Because the GI tract is functioning, feeding methods involve the enteral route which bypasses the mouth but allows for a major portion of the GI tract to be used. 43. B The first step in assessing the abdomen is to observe its shape and contour, then auscultate, palpate, and then percuss. 44. B Lowering the height decreases the amount of flow, allowing him to tolerate more fluid. 45. D Pancrelipase provides the exocrine pancreatic enzyme necessary for proper protein, fat, and carb digestion. With increased fat digestion and absorption, stools become less frequent and normal in appearance. 46. B Glucose level increases and diabetes mellitus may result d/t the pancreatic damage to the islets of langerhans. 47. D After surgery, she remains NPO until peristaltic activity returns. This decreases the risk for abdominal distention and obstruction. 48. D A brownish-black color indicates lack of blood flow, and maybe necrosis. 49. A Restricting fluids decrease the amount of body fluid and the accumulation of fluid in the peritoneal space. 50. D Dark green, leafy vegetables are rich in calcium. 51. A For pruritus, care should include tepid sponge baths and use of emollient creams and lotions. 52. D Rest periods and small frequent meals is indicated during the acute phase of hepatitis B. 53. D Hepatitis B can recur. Patients who have had hepatitis are permanently barred from donating blood. Alcohol is metabolized by the liver and should be avoided by those who have or had hepatitis B. 54. A To prevent venous stasis and improve muscle tone, circulation, and respiratory function, encourage her to move after surgery. 55. A Telling her not to worry minimizes her feelings. 56. A Diazepam is absorbed by the plastic I.V. tubing and should be given in the port closest to the vein. 57. A For the first few days to a week, slight bleeding normally occurs when the stoma is touched because the surgical site is still new. She should report profuse bleeding immediately. 58. D To wash away tissue debris and drainage effectively, irrigate the wound until the solution becomes clear or all the solution is used. 59. A Ammonia levels increase d/t improper shunting of blood, causing ammonia to enter systemic circulation, which carries it to the brain. 60. D Saline solution is isotonic, or close to body fluids in content, and is used along with sterile dressings to cover an eviscerated wound and keep it moist. 61. B Crohns disease penetrates the mucosa of the colon through all layers and destroys the colon in patches, which creates a cobblestone appearance. 62. A Stomach pain is often a late sign of stomach cancer; outcomes are particularly poor when the cancer reaches that point. Surgery, chemotherapy, and radiation have minimal positive effects. TPN may enhance the growth of the cancer. 63. C Melena is the passage of dark, tarry stools that contain a large amount of digested blood. It occurs with bleeding from the upper GI tract. 64. A A patient with an acute upper GI hemorrhage must be treated for hypovolemia and hemorrhagic shock. You as a nurse cant diagnose the problem. Controlling the bleeding may require surgery or intensive med ical treatment. 65. D To stabilize a patient with acute bleeding, NS or LR solution is given I.V. until BP rises and urine output returns to 30ml/hr. 66. A Initially, you should assess the patients knowledge about colostomies and how it will affect his li festyle.

41

67. B An inflammatory condition that affects the surface of the colon, ulcerative colitis causes friability and erosions with bleeding. Patients with ulcerative colitis are at increased risk for bowel perforation, toxic megacolon, hemorrhage, cancer, and other anorectal and systemic complications. 68. C Medications to control inflammation such as corticosteroids are used for long-term treatment. 69. A Meats and beans are high-protein foods. In liver failure, the liver is unable to metabolize protein adequately, causing protein by-products to build up in the body rather than be excreted. 70. B A gastric residual greater than 2 hours worth of feeding or 100-150ml is considered too high. The feeding should be stopped; NG tube clamped, and then allow time for the stomach to empty before additional feeding is added. 71. C Frequent mouth care helps relieve dry mouth. 72. C Coffee-ground emesis occurs when there is upper GI bleeding that has undergone gastric digestion. For blood to appear as coffee-ground emesis, it would have to be digested for approximately 2 hours. 73. C Normally, drainage is bloody for the first 24 hours after a partial gastrectomy; then it changes to brown-tinged and then to yellow or clear. 74. D If the vagus nerve is cut as it enters the stomach, gastric acid secretion is decreased, but intestinal motility is also decreased and gastric emptying is delayed. Because gastric acids are decreased, gastric pH increases. 75. A Dosage problem. Its 80/20 = 240/X. X=60. 76. A The output from an Ileostomy is described. 77. B As healing occurs from the bile duct, bile drains from the tube; the amount of bile should decrease. Teach the patient to expect dark green drainage and to notify the doctor if drainage stops. 78. C Misoprostol restores prostaglandins that protect the stomach from NSAIDS, which diminish the prostaglandins. 79. C 80. D

Introduction
Questions about Substance Abuse, Alcoholism, Therapeutic Communication and more are included in the 25item NCLEXsample exam.

Topics

Substance Abuse Therapeutic Communication Alcohol Abuse and Withdrawal

Guidelines
Read each question carefully and choose the best answer. You are given one minute per question. Spend your time wisely! Answers and rationales are given below. Be sure to read them. If you need more clarifications, please direct them to the comments section.

Questions
1. The nurse is planning activities for a client who has bipolar disorder with aggressive social behavior. Which of the following activities would be most appropriate for this client? 1. Ping pong 2. Writing 3. Chess 4. Basketball

42

2. A client is admitted to the hospital with a diagnosis of major depression, severe, single episode. The nurse assesses the client and identifies a nursing diagnosis of imbalanced nutrition related to poor nutritional intake. The most appropriate nursing intervention related to this diagnosis is: 1. Explain to the client the importance of a good nutritional intake 2. Weight the client 3 times per week before breakfast 3. Report the nutritional concern to the psychiatrist and obtain a nutritional consultation as soon as possible. 4. Consult with the nutritionist, offer the client several small meals per day, and schedule brief nursing interactions with the client during these times. 3. In planning activities for the depressed client, especially during the early stages of hospitalization, which of the following plans is best? 1. Provide an activity that is quiet and solitary to avoid increased fatigue, such as working on a puzzle or reading a book. 2. Plan nothing until the client asks to participate in milieu. 3. Offer the client a menu of daily activities and insist the client participate in all of them 4. Provide a structured daily program of activities and encourage the client to participate. 4. The depressed client verbalizes feelings of low self-esteem and self-worth typified by statements such as Im such a failure I cant do anything right! The best nursing response would be: 1. To tell the client this is not true; that we all have a purpose in life. 2. To remain with the client and sit in silence; this will encourage the client to verbalize feelings 3. To reassure the client that you know how the client is feeling and that things will get better 4. To identify recent behaviors or accomplishments that demonstrates skill ability. 5. A client with a diagnosis of major depression, recurrent with psychotic features is admitted to the mental health unit. To create a safe environment for the client, the nurse most importantly devises a plan of care that deals specifically with the clients: 1. Disturbed thought processes 2. Imbalanced nutrition 3. Self-care deficit 4. Deficient knowledge 6. A depressed client is ready for discharge. The nurse feels comfortable that the client has a good understanding of the disease process when the client states: 1. Ill never let this happen to me again. I wont let my boss or my job or my family get to me! 2. Its important for me to eat well, exercise, and to take my medication. If I begin to lose my appetite or not sleep well, Ive got to get in to see my doctor. 3. Ive learned that Im a good person and that I am worthy of giving and receiving love. I dont need anyone; I have myself to rely on! 4. I dont know what happened to me. Ive always been able to make decisions for myself and for my business. I dont ever want to feel so weak or vulnerable again! 7. The nurse assesses a client with the admitting diagnosis of bipolar affective disorder, mania. The symptom presented by the client that requires the nurses immediate intervention is the clients: 1. Outlandish behaviors and inappropriate dress 2. Grandiose delusions of being a royal descendent of King Arthur. 3. Nonstop physical activity and poor nutritional intake 4. Constant, incessant talking that includes sexual innuendoes and teasing the staff 8. The nurse reviews the activity schedule for the day and plans which activity for the manic client? 1. Brown-bag luncheon and book review 2. Tetherball 3. Paint-by-number activity 4. Deep breathing and progressive relaxation group

43

9. A hospitalized client is being considered for ECT. The client appears calm, but the family is anxious. The clients mother begins to cry and states My sons brain will be destroyed. How can the doctor do this to him? The nurses best response is: 1. It sounds as though you need to speak with the psychiatrist 2. Your son has decided to have this treatment. You should be supportive to him. 3. Perhaps youd like to see the ECT room and speak to the staff. 4. It sounds as though you have some concerns about the ECT procedure. Why dont we sit down together and discuss any concerns you may have. 10. The manic client announces to everyone in the dayroom that a stripper is coming to perform this evening. When the nurse firmly states that this will not happen, the manic client becomes verbally abusive and threatens physical violence to the nurse. Based on the analysis of this situation, the nurse determines that the most appropriate action would be to: 1. With assistance, escort the manic client to her room and administer Haldol as prescribed if needed 2. Tell the client that smoking privileges are revoked for 24 hours 3. Orient the client to time, person, and place 4. Tell the client that the behavior is not appropriate. 11. Select all nursing interventions for a hospitalized client with mania who is exhibiting manipulative behavior. 1. Communicate expected behaviors to the client 2. Enforce rules and inform the client the he or she will not be allowed to attend group therapy sessions. 3. Ensure that the client knows that he or she is not in charge of the nursing unit 4. Be clear with the client regarding the consequences of exceeding limits set regarding behavior. 5. Assist the client in testing out alternative behaviors for obtaining needs 12. A woman comes into the ER in a severe state of anxiety following a car accident. The most appropriate nursing intervention is to: 1. Remain with the client 2. Put the client in a quiet room 3. Teach the client deep breathing 4. Encourage the client to talk about their feelings and concern. 13. When planning the discharge of a client with chronic anxiety, the nurse directs the goals at promoting a safe environment at home. The most appropriate maintenance goal should focus on which of the following? 1. Continued contact with a crisis counselor 2. Identifying anxiety-producing situations 3. Ignoring feelings of anxiety 4. Eliminating all anxiety from daily situations 14. The nurse is monitoring a client who abuses alcohol for signs of alcohol withdrawal. Which of the following would alert the nurse to the potential for delirium tremors? 1. Hypertension, changes in LOC, hallucinations 2. Hypotension, ataxia, hunger 3. Stupor, agitation, muscular rigidity 4. Hypotension, coarse hand tremors, agitation 15. The spouse of a client admitted to the mental health unit for alcohol withdrawal says to the nurse I should get out of this bad situation. The most helpful response by the nurse would be: 1. I agree with you. You should get out of this situation. 2. What do you find difficult about this situation? 3. Why dont you tell your husband about this? 4. This is not the best time to make that decision. 16. The nurse determines that the wife of an alcoholic client is benefiting from attending Al-Anon group when she hears the wife say:

44

1. My attendance at the meetings has helped me to see that I provoke my husbands violence. 2. I no longer feel that I deserve the beatings my husband inflicts on me. 3. I can tolerate my husbands destructive behavior now that I know they are common with alcoholics. 4. I enjoy attending the meetings because they get me out of the house and away from my husband. 17. The client has been hospitalized and is participating in a substance abuse therapy group sessions. On discharge, the client has consented to participate in AA community groups. The nurse is monitoring the clients response to the substance abuse sessions. Which statement by the client best indicates that the client has developed effective coping response styles and has processed information effectively for self use? 1. I know Im ready to be discharged. I feel I can say no and leave a group of friends if they are drinking No Problem. 2. This group has really helped a lot. I know it will be different when I go home. But Im sure that my family and friends w ill all help me like the people in this group have Theyll all help me I know they will They wont let me go back to my old ways. 3. Im looking forward to leaving here. I know that I will miss all of you. So, Im happy and Im sad, Im excited and Im scared. I know that I have to work hard to be strong and that everyone isnt going to be as helpful as you people. 4. Ill keep all my appointments; go to all my AA groups; Ill do everything Im supposed to Nothing will go wrong that way. 18. A hospitalized client with a history of alcohol abuse tells the nurse, I am leaving now. I have to go. I dont want anymore treatment. I have things that I have to do right away. The client has not been discharged. In fact, the client is scheduled for an important diagnostic test to be performed in 1 hour. After the nurse discusses the clients concerns with the client, the client dresses and begins to walk out of the hospital room. The most important nursing action is to: 1. Restrain the client until the physician can be reached 2. Call security to block all areas 3. Tell the client that the client cannot return to this hospital again if the client leaves now. 4. Call the nursing supervisor. 19. Select the appropriate interventions for caring for the client in alcohol withdrawal. 1. Monitor vital signs 2. Provide stimulation in the environment 3. Maintain NPO status 4. Provide reality orientation as appropriate 5. Address hallucinations therapeutically 20. Which of the following nursing actions would be included in a care plan for a client with PTSD who states the experience was bad luck? 1. Encourage the client to verbalize the experience 2. Assist the client in defining the experience 3. Work with the client to take steps to move on with his life 4. Help the client accept positive and negative feelings 21. Which of the following psychological symptoms would the nurse expect to find in a hospitalized client who is the only survivor of a train accident? 1. Denial 2. Indifference 3. Perfectionism 4. Trust 22. Which of the following communication guidelines should the nurse use when talking with a client experiencing mania? 1. Address the client in a light and joking manner 2. Focus and redirect the conversation as necessary 3. Allow the client to talk about several different topic 4. Ask only open ended questions to facilitate conversations

45

23. What information is important to include in the nutritional counseling of a family with a member who has bipolar disorder? 1. If sufficient roughage isnt eaten while taking lithium, bowel prob lems will occur. 2. If the intake of carbohydrates increases, the lithium level increases. 3. If the intake of calories is reduced, the lithium level will increase 4. If the intake of sodium increases, the lithium level will decrease. 24. In conferring with the treatment team, the nurse should make which of the following recommendations for a client who tells the nurse that everyday thoughts of suicide are present? 1. A no-suicide contract 2. Weekly outpatient therapy 3. A second psychiatric opinion 4. Intensive inpatient treatment 25. Which of the following short term goals is most appropriate for a client with bipolar disorder who is having difficulty sleeping? 1. Obtain medication for sleep 2. Work on solving a problem 3. Exercise before bedtime 4. Develop a sleep ritual

Answers and Rationale


1. Answer: B. Solitary activities that require a short attention span with mild physical exertion are the most appropriate activities for a client who is exhibiting aggressive behavior. Writing, walks with staff, and finger painting are activities that minimize stimuli and provide a constructive release for tension. Competitive games can stimulate aggression and increase psychomotor activity. 2. Answer: D. Change in appetite is one of the major symptoms of depression. Reporting to the psychiatrist and nutritionist is to some degree correct but lacks the method as to how one would increase food intake. 3. Answer: D. A depressed person experiences a depressed mood and is often withdrawn. The person also experiences difficulty concentrating, loss of interest or pleasure, low energy, fatigue, and feelings of worthlessness and poor self-esteem. The plan of care needs to provide successful experiences in a stimulating yet structured environment. Option 3 is a forceful and absolute approach. 4. Answer: D. Feelings of low self-esteem and worthlessness are common symptoms of the depressed client. An effective plan of care to enhance the clients personal self-esteem is to provide experiences for the client that are challenging but will not be met with failure. Reminders of the clients past accomplishments or personal successes are ways to interrupt the clients negativ e self talk and distorted cognitive view of self. Silence may be interpreted as agreement. Options 1 and 3 give advice and devalue the clients feelings. 5. Answer: A. major depression, recurrent, with psychotic features alerts the nurse that in addition to the criteria that designate the diagnosis of major depression, one also must deal with the clients psychosis. P sychosis is defined as a state in which a persons mental capacity to recognize reality and to communicate and relate to others is impaired, thus interfering with the persons capacity to deal with the demands of life. Altered thought processes generally i ndicate a state of increased anxiety in which hallucinations and delusions prevail. Although all of the nursing diagnoses may be appropriate because the client is experiencing psychosis, option 1 is correct. 6. Answer: B. The exact cause of depression is not known but is believed to be related to biochemical disruption of neurotransmitters in the brain. Diet, exercise, and medication are recognized treatment for the disease process. 7. Answer: C. Mania is a mood characterized by excitement, euphoria, hyperactivity, excessive energy, decreased need for sleep, and impaired ability to concentrate or complete a single train of thought. Mania is a period when the mood is predominately elevated, expansive, or irritable. All options reflect a clients possible symp tomatology. Option 3, however, clearly presents a problem that compromises ones physiological integrity and needs to be addressed immediately.

46

8. Answer: B. A person who is experiencing mania is overactive and full of energy, lacks concentration, and has poor impulse control. The client needs an activity that will allow use of excess energy yet not endanger others during the process. Options 1, 3, and 4 are relatively sedate activities that require concentration, a quality that is lacking in the manic state. Such activities lead to increased frustration and anxiety for the client. Tetherball is an exercise that uses the large muscle groups of the body and is a great way to expend the increased energy that the client is experiencing. 9. Answer: D. The nurse encourages the client and the family to verbalize fears and concerns. The other options avoid dealing with concerns and are blocks to communication. 10. Answer: A. The client is at risk for injury to self and others and therefore should be escorted out of the dayroom. Antipsychotic medications are useful to manage the manic client. Hyperactive and agitated behavior usually responds to Haldol. Option 2 may increase the agitation that already exists in this client. Orientation will not halt the behavior. Telling the client that the behavior is not appropriate already has been attempted by the nurse. 11. Answers: A, D, and E. Interventions for dealing with the client exhibiting manipulative behavior include setting clear, consistent, and enforceable limits on manipulative behaviors; being clear with the client regarding the consequences of exceeding limits set; following through with the consequences in a non-punishment manner; and assisting the client in identifying strengths and in testing out alternative behaviors for obtaining needs. Enforcing rules and informing the client that he or she will not be allowed to attend group therapy sessions is a violation of the clients rights. Ensuring the client knows that he or she is n ot in charge of the nursing unit is inappropriate, power struggles need to be avoided. 12. Answer: A. If a client with severe anxiety is left alone; the client may feel abandoned and become overwhelmed. Placing the client in a quiet room is also important, but the nurse must stay with the client. Teaching the client deep breathing or relaxation is not possible until the anxiety decreases. Encouraging the client to discuss concerns and feelings would not take place until the anxiety has decreased. 13. Answer: B. Recognizing situations that produce anxiety allows the client to prepare to cope with anxiety or avoid a specific stimulus. Counselors will not be available for all anxiety-producing situations, and this option does not encourage the development of internal strengths. Ignoring feelings will not resolve anxiety. Elimination anxiety from life is impossible. 14. Answer: A. Some of the symptoms associated with delirium tremors typically are anxiety, insomnia, anorexia, hypertension, disorientation, hallucinations, and changes in LOC, agitation, fever, and delusions. 15. Answer: B. The most helpful response is one that encourages the client to problem solve. Giving advice implies that the nurse knows what is best and can foster dependency. The nurse should not agree with the client, nor should the nurse request that the client provide explanations. 16. Answer: B. Al-Anon support groups are protected, supportive opportunity for spouses and significant others to learn what to expect and to obtain excellent pointers about successful behavior changes. Option 2 is the most healthy response because is exemplifies and understanding that the alcoholic partner is responsible for his behavior and cannot be allowed to blame family members for loss of control. 17. Answer: C. In the defense mechanism of denial the person denies reality. Option 1 identifies denial. In option 2 the client is relying heavily on others, and the clients focus of control is external. In option 4 the client is concrete and procedure or iented; again the client identifies that Nothing will go wrong that way if the client follows all the directions. In option 3 the client is expressing real concern and ambivalence about discharge from the hospital. The client also demonstrates reality in that statement. 18. Answer: D. A nurse can be charged with false imprisonment if a client is made to believe wrongfully that the client cannot leave the hospital. Most health care facilities have documents that the client is asked to sign that relate to the clients responsibilities when the client leaves against medical advice. The client should be asked to sign this document before leaving. The nurse should request that the client wait to speak to the physician before leaving, but if the client refuses to do so, the nurse cannot hold him against his will. Restraining the client and calling security to block exits constitutes false imprisonment. Any client has a right to health care and cannot be told otherwise. 19. Answers: A, D, and E. When the client is experiencing withdrawal of alcohol, the priority of care is to prevent the client from harming himself or others. The nurse would provide a low stimulating environment to maintain the client in as calm a state as possible. The nurse would monitor vital signs closely and report abnormal findings. The nurse would reorient the client to reality frequently and would address hallucinations therapeutically. Adequate nutritional and fluid intake needs to be maintained.

47

20. Answer: B. The client must define the experience as traumatic to realize the situation wasnt under his personal control. Encouraging the client to verbalize the experience without first addressing the denial isnt a useful strategy. The client ca n move on with life only after acknowledging the trauma and processing the experience. Acknowledgement of the actual trauma and verbalization of the event should come before the acceptance of feelings. 21. Answer: A. Denial can act as a protective response. The client tends to be overwhelmed and disorganized by the trauma, not indifferent to it. Perfectionism is more commonly seen in clients with eating disorders, not in clients with PTSD. Clients who have had a severe trauma often experience an inability to trust others. 22. Answer: B. To decrease stimulation, the nurse should attempt to redir ect and focus the clients communication, not allow the client to talk about different topics. By addressing the client in a light and joking manner, the conversation may contribute to the clients feeling out of control. For a manic client, its best to ask closed questions because open-minded questions may enable the client to talk endlessly, again possibly contributing to the clients feeling out of control. 23. Answer: D. Any time the level of sodium increases, such as with a change in the dietary intake, the levels of lithium will decrease. 24. Answer: D. For a client thinking about suicide on a daily basis, inpatient care would be the best intervention. Although a nosuicide contract is an important strategy, this client needs additional care. The client needs a more intensive level of care than weekly outpatient therapy. Immediate intervention is paramount, not a second psychiatric opinion. 25. Answer: D. A sleep ritual or nighttime routine helps the client to relax and prepare for sleep. Obtaining sleep medication is a temporary solution. Working on problem solving may excite the client rather than tire him. Exercise before retiring is inappropriate.

Introduction
This is a 45-item examination that can help you in your NCLEX test. Questions here include topics like Renal Failure, Dialysis and more.

Topics

Renal Failure Dialysis

Guidelines
Read each question carefully and choose the best answer. You are given one minute per question. Spend your time wisely! Answers and rationales are given below. Be sure to read them. If you need more clarifications, please direct them to the comments section.

Questions
1. Dialysis allows for the exchange of particles across a semipermeable membrane by which of the following actions? 1. Osmosis and diffusion 2. Passage of fluid toward a solution with a lower solute concentration 3. Allowing the passage of blood cells and protein molecules through it. 4. Passage of solute particles toward a solution with a higher concentration. 2. A client is diagnosed with chronic renal failure and told she must start hemodialysis. Client teaching would include which of the following instructions? 1. Follow a high potassium diet 2. Strictly follow the hemodialysis schedule 3. There will be a few changes in your lifestyle. 4. Use alcohol on the skin and clean it due to integumentary changes.

48

3. A client is undergoing peritoneal dialysis. The dialysate dwell time is completed, and the dwell clamp is opened to allow the dialysate to drain. The nurse notes that the drainage has stopped and only 500 ml has drained; the amount the dialysate instilled was 1,500 ml. Which of the following interventions would be done first? 1. Change the clients position. 2. Call the physician. 3. Check the catheter for kinks or obstruction. 4. Clamp the catheter and instill more dialysate at the next exchange time. 4. A client receiving hemodialysis treatment arrives at the hospital with a blood pressure of 200/100, a heart rate of 110, and a respiratory rate of 36. Oxygen saturation on room air is 89%. He complains of shortness of breath, and +2 pedal edema is noted. His last hemodialysis treatment was yesterday. Which of the following interventions should be done first? 1. Administer oxygen 2. Elevate the foot of the bed 3. Restrict the clients fluids 4. Prepare the client for hemodialysis. 5. A client has a history of chronic renal failure and received hemodialysis treatments three times per week through an arteriovenous (AV) fistula in the left arm. Which of the following interventions is included in this clients plan of care? 1. Keep the AV fistula site dry. 2. Keep the AV fistula wrapped in gauze. 3. Take the blood pressure in the left arm 4. Assess the AV fistula for a bruit and thrill 6. Which of the following factors causes the nausea associated with renal failure? 1. Oliguria 2. Gastric ulcers 3. Electrolyte imbalances 4. Accumulation of waste products 7. Which of the following clients is at greatest risk for developing acute renal failure? 1. A dialysis client who gets influenza 2. A teenager who has an appendectomy 3. A pregnant woman who has a fractured femur 4. A client with diabetes who has a heart catherization 8. In a client in renal failure, which assessment finding may indicate hypocalcemia? 1. Headache 2. Serum calcium level of 5 mEq/L 3. Increased blood coagulation 4. Diarrhea 9. A nurse is assessing the patency of an arteriovenous fistula in the left arm of a client who is receiving hemodialysis for the treatment of chronic renal failure. Which finding indicates that the fistula is patent? 1. Absence of bruit on auscultation of the fistula. 2. Palpation of a thrill over the fistula 3. Presence of a radial pulse in the left wrist 4. Capillary refill time less than 3 seconds in the nail beds of the fingers on the left hand. 10. The client with chronic renal failure is at risk of developing dementia related to excessive absorption of aluminum. The nurse teaches that this is the reason that the client is being prescribed which of the following phosphate binding agents? 1. Alu-cap (aluminum hydroxide) 2. Tums (calcium carbonate)

49

3. Amphojel (aluminum hydroxide) 4. Basaljel (aluminum hydroxide) 11. The client newly diagnosed with chronic renal failure recently has begun hemodialysis. Knowing that the client is at risk for disequilibrium syndrome, the nurse assesses the client during dialysis for: 1. Hypertension, tachycardia, and fever 2. Hypotension, bradycardia, and hypothermia 3. restlessness, irritability, and generalized weakness 4. Headache, deteriorating level of consciousness, and twitching. 12. A client with chronic renal failure has completed a hemodialysis treatment. The nurse would use which of the following standard indicators to evaluate the clients status after dialysis? 1. Potassium level and weight 2. BUN and creatinine levels 3. VS and BUN 4. VS and weight. 13. The hemodialysis client with a left arm fistula is at risk for steal syndrome. The nurse assesses this client for which of the following clinical manifestations? 1. Warmth, redness, and pain in the left hand. 2. Pallor, diminished pulse, and pain in the left hand. 3. Edema and reddish discoloration of the left arm 4. Aching pain, pallor, and edema in the left arm. 14. A client is admitted to the hospital and has a diagnosis of early stage chronic renal failure. Which of the following would the nurse expect to note on assessment of the client? 1. Polyuria 2. Polydipsia 3. Oliguria 4. Anuria 15. The client with chronic renal failure returns to the nursing unit following a hemodialysis treatment. On assessment the nurse notes that the clients temperature is 100.2. Which of the following is the most appropriate nursing action? 1. Encourage fluids 2. Notify the physician 3. Monitor the site of the shunt for infection 4. Continue to monitor vital signs 16. The nurse is performing an assessment on a client who has returned from the dialysis unit following hemodialysis. The client is complaining of a headache and nausea and is extremely restless. Which of the following is the most appropriate nursing action? 1. Notify the physician 2. Monitor the client 3. Elevate the head of the bed 4. Medicate the client for nausea 17. The nurse is assisting a client on a low-potassium diet to select food items from the menu. Which of the following food items, if selected by the client, would indicate an understanding of this dietary restriction? 1. Cantaloupe 2. Spinach 3. Lima beans 4. Strawberries

50

18. The nurse is reviewing a list of components contained in the peritoneal dialysis solution with the client. The client asks the nurse about the purpose of the glucose contained in the solution. The nurse bases the response knowing that the glucose: 1. Prevents excess glucose from being removed from the client. 2. Decreases risk of peritonitis. 3. Prevents disequilibrium syndrome 4. Increases osmotic pressure to produce ultrafiltration. 19. The nurse is preparing to care for a client receiving peritoneal dialysis. Which of the following would be included in the nursing plan of care to prevent the major complication associated with peritoneal dialysis? 1. Monitor the clients level of consciousness 2. Maintain strict aseptic technique 3. Add heparin to the dialysate solution 4. Change the catheter site dressing daily 20. A client newly diagnosed with renal failure is receiving peritoneal dialysis. During the infusion of the dialysate the client complains of abdominal pain. Which action by the nurse is most appropriate? 1. Slow the infusion 2. Decrease the amount to be infused 3. Explain that the pain will subside after the first few exchanges 4. Stop the dialysis 21. The nurse is instructing a client with diabetes mellitus about peritoneal dialysis. The nurse tells the client that it is important to maintain the dwell time for the dialysis at the prescribed time because of the risk of: 1. Infection 2. Hyperglycemia 3. Fluid overload 4. Disequilibrium syndrome 22. The client with acute renal failure has a serum potassium level of 5.8 mEq/L. The nurse would plan which of the following as a priority action? 1. Allow an extra 500 ml of fluid intake to dilute the electrolyte concentration. 2. Encourage increased vegetables in the diet 3. Place the client on a cardiac monitor 4. Check the sodium level 23. The client with chronic renal failure who is scheduled for hemodialysis this morning is due to receive a daily dose of enalapril (Vasotec). The nurse should plan to administer this medication: 1. Just before dialysis 2. During dialysis 3. On return from dialysis 4. The day after dialysis 24. The client with chronic renal failure has an indwelling catheter for peritoneal dialysis in the abdomen. The client spills water on the catheter dressing while bathing. The nurse should immediately: 1. Reinforce the dressing 2. Change the dressing 3. Flush the peritoneal dialysis catheter 4. Scrub the catheter with povidone-iodine 25. The client being hemodialyzed suddenly becomes short of breath and complains of chest pain. The client is tachycardic, pale, and anxious. The nurse suspects air embolism. The nurse should: 1. Continue the dialysis at a slower rate after checking the lines for air 2. Discontinue dialysis and notify the physician

51

3. Monitor vital signs every 15 minutes for the next hour 4. Bolus the client with 500 ml of normal saline to break up the air embolism. 26. The nurse has completed client teaching with the hemodialysis client about self-monitoring between hemodialysis treatments. The nurse determines that the client best understands the information given if the client states to record the daily: 1. Pulse and respiratory rate 2. Intake, output, and weight 3. BUN and creatinine levels 4. Activity log 27. The client with an arteriovenous shunt in place for hemodialysis is at risk for bleeding. The nurse would do which of the following as a priority action to prevent this complication from occurring? 1. Check the results of the PT time as they are ordered. 2. Observe the site once per shift 3. Check the shunt for the presence of a bruit and thrill 4. Ensure that small clamps are attached to the AV shunt dressing. 28. The nurse is monitoring a client receiving peritoneal dialysis and nurse notes that a clients outflow is less than the inflow. Select actions that the nurse should take. 1. Place the client in good body alignment 2. Check the level of the drainage bag 3. Contact the physician 4. Check the peritoneal dialysis system for kinks 5. Reposition the client to his or her side. 29. The nurse assesses the client who has chronic renal failure and notes the following: crackles in the lung bases, elevated blood pressure, and weight gain of 2 pounds in one day. Based on these data, which of the following nursing diagnoses is appropriate? 1. Excess fluid volume related to the kidneys inability to maintain fluid balance. 2. Increased cardiac output related to fluid overload. 3. Ineffective tissue perfusion related to interrupted arterial blood flow. 4. Ineffective therapeutic Regimen Management related to lack of knowledge about therapy. 30. The nurse is caring for a hospitalized client who has chronic renal failure. Which of the following nursing diagnoses are most appropriate for this client? Select all that apply. 1. Excess Fluid Volume 2. Imbalanced Nutrition; Less than Body Requirements 3. Activity Intolerance 4. Impaired Gas Exchange 5. Pain. 31. What is the primary disadvantage of using peritoneal dialysis for long term management of chronic renal failure? 1. The danger of hemorrhage is high. 2. It cannot correct severe imbalances. 3. It is a time consuming method of treatment. 4. The risk of contracting hepatitis is high. 32. The dialysis solution is warmed before use in peritoneal dialysis primarily to: 1. Encourage the removal of serum urea. 2. Force potassium back into the cells. 3. Add extra warmth into the body. 4. Promote abdominal muscle relaxation.

52

33. During the clients dialysis, the nurse observes that the solution draining from the abdomen is consistently blood tinged. The client has a permanent peritoneal catheter in place. Which interpretation of this observation would be correct? 1. Bleeding is expected with a permanent peritoneal catheter 2. Bleeding indicates abdominal blood vessel damage 3. Bleeding can indicate kidney damage. 4. Bleeding is caused by too-rapid infusion of the dialysate. 34. Which of the following nursing interventions should be included in the clients care plan during dialysis therapy? 1. Limit the clients visitors 2. Monitor the clients blood pressure 3. Pad the side rails of the bed 4. Keep the client NPO. 35. Aluminum hydroxide gel (Amphojel) is prescribed for the client with chronic renal failure to take at home. What is the purpose of giving this drug to a client with chronic renal failure? 1. To relieve the pain of gastric hyperacidity 2. To prevent Curlings stress ulcers 3. To bind phosphorus in the intestine 4. To reverse metabolic acidosis. 36. The nurse teaches the client with chronic renal failure when to take the aluminum hydroxide gel. Which of the following statements would indicate that the client understands the teaching? 1. Ill take it every 4 hours around the clock. 2. Ill take it between meals and at bedtime. 3. Ill take it when I have a sour stomach. 4. Ill take it with meals and bedtime snacks. 37. The client with chronic renal failure tells the nurse he takes magnesium hydroxide (milk of magnesia) at home for constipation. The nurse suggests that the client switch to psyllium hydrophilic mucilloid (Metamucil) because: 1. MOM can cause magnesium toxicity 2. MOM is too harsh on the bowel 3. Metamucil is more palatable 4. MOM is high in sodium 38. In planning teaching strategies for the client with chronic renal failure, the nurse must keep in mind the neurologic impact of uremia. Which teaching strategy would be most appropriate? 1. Providing all needed teaching in one extended session. 2. Validating frequently the clients understanding of the material. 3. Conducting a one-on-one session with the client. 4. Using videotapes to reinforce the material as needed. 39. The nurse helps the client with chronic renal failure develop a home diet plan with the goal of helping the client maintain adequate nutritional intake. Which of the following diets would be most appropriate for a client with chronic renal failure? 1. High carbohydrate, high protein 2. High calcium, high potassium, high protein 3. Low protein, low sodium, low potassium 4. Low protein, high potassium 40. A client with chronic renal failure has asked to be evaluated for a home continuous ambulatory peritoneal dialysis (CAPD) program. The nurse should explain that the major advantage of this approach is that it: 1. Is relatively low in cost 2. Allows the client to be more independent

53

3. Is faster and more efficient than standard peritoneal dialysis 4. Has fewer potential complications than standard peritoneal dialysis 41. The client asks whether her diet would change on CAPD. Which of the following would be the nurses best response? 1. Diet restrictions are more rigid with CAPD because standard peritoneal dialysis is a more effective technique. 2. Diet restrictions are the same for both CAPD and standard peritoneal dialysis. 3. Diet restrictions with CAPD are fewer than with standard peritoneal dialysis because dialysis is constant. 4. Diet restrictions with CAPD are fewer than with standard peritoneal dialysis because CAPD works more quickly. 42. Which of the following is the most significant sign of peritoneal infection? 1. Cloudy dialysate fluid 2. Swelling in the legs 3. Poor drainage of the dialysate fluid 4. Redness at the catheter insertion site 43. The main indicator of the need for hemodialysis is: 1. Ascites 2. Acidosis 3. Hypertension 4. Hyperkalemia 44. To gain access to the vein and artery, an AV shunt was used for Mr. Roberto. The most serious problem with regards to the AV shunt is: 1. Septicemia 2. Clot formation 3. Exsanguination 4. Vessel sclerosis 45. When caring for Mr. Robertos AV shunt on his right arm, you should: 1. Cover the entire cannula with an elastic bandage 2. Notify the physician if a bruit and thrill are present 3. User surgical aseptic technique when giving shunt care 4. Take the blood pressure on the right arm instead

Answers and Rationale


1. Answer: 1. Osmosis allows for the removal of fluid from the blood by allowing it to pass through the semipermeable membrane to an area of high concentrate (dialysate), and diffusion allows for passage of particles (electrolytes, urea, and creatinine) from an area of higher concentration to an area of lower concentration. Fluid passes to an area with a higher solute concentration. The pores of a semipermeable membrane are small, thus preventing the flow of blood cells and protein molecules through it. 2. Answer: 2. To prevent life-threatening complications, the client must follow the dialysis schedule. Alcohol would further dry the clients skin more than it already is. The client should follow a low-potassium diet because potassium levels increase in chronic renal failure. The client should know hemodialysis is time-consuming and will definitely cause a change in current lifestyle. 3. Answer: 3. The first intervention should be to check for kinks and obstructions because that could be preventing drainage. After checking for kinks, have the client change position to promote drainage. Dont give the next scheduled exchange until t he dialysate is drained because abdominal distention will occur, unless the output is within parameters set by the physician. If unable to get more output despite checking for kinks and changing the clients position, the nurse should then call the physician to determine the proper intervention. 4. Answer: 1. Airway and oxygenation are always the first priority. Because the client is complaining of shortness of breath and his oxygen saturation is only 89%, the nurse needs to try to increase his levels by administering oxygen. The client is in pulmonary edema from fluid overload and will need to be dialyzed and have his fluids restricted, but the first interventions

54

should be aimed at the immediate treatment of hypoxia. The foot of the bed may be elevated to reduce edema, but this isnt th e priority. 5. Answer: 4. Assessment of the AV fistula for bruit and thrill is important because, if not present, it indicates a non-functioning fistula. No blood pressures or venipunctures should be taken in the arm with the AV fistula. When not being dialyzed, the AV fistula site may get wet. Immediately after a dialysis treatment, the access site is covered with adhesive bandages. 6. Answer: 4. Although clients with renal failure can develop stress ulcers, the nausea is usually related to the poisons of metabolic wastes that accumulate when the kidneys are unable to eliminate them. The client has electrolyte imbalances and oliguria, but these dont directly cause nausea. 7. Answer: 4. Clients with diabetes are prone to renal insufficiency and renal failure. The contrast used for heart catherization must be eliminated by the kidneys, which further stresses them and may produce acute renal failure. A teenager who has an appendectomy and a pregnant woman with a fractured femur isnt at increased risk for renal failure. A dialysis client already has end-stage renal disease and wouldnt develop acute renal failure. 8. Answer: 4. In renal failure, calcium absorption from the intestine declines, leading to increased smooth muscle contractions, causing diarrhea. CNS changes in renal failure rarely include headache. A serum calcium level of 5 mEq/L indicates hypercalcemia. As renal failure progresses, bleeding tendencies increase. 9. Answer: 2. The nurse assesses the patency of the fistula by palpating for the presence of a thrill or auscultating for a bruit. The presence of a thrill and bruit indicate patency of the fistula. Although the presence of a radial pulse in the left wrist and capillary refill time less than 3 seconds in the nail beds of the fingers on the left hand are normal findings, they do not assess fistula patency. 10. Answer: 2. Phosphate binding agents that contain aluminum include Alu-caps, Basaljel, and Amphojel. These products are made from aluminum hydroxide. Tums are made from calcium carbonate and also bind phosphorus. Tums are prescribed to avoid the occurrence of dementia related to high intake of aluminum. Phosphate binding agents are needed by the client in renal failure because the kidneys cannot eliminate phosphorus. 11. Answer: 4. Disequilibrium syndrome is characterized by headache, mental confusion, decreasing level of consciousness, nausea, and vomiting, twitching, and possible seizure activity. Disequilibrium syndrome is caused by rapid removal of solutes from the body during hemodialysis. At the same time, the blood-brain barrier interferes with the efficient removal of wastes from brain tissue. As a result, water goes into cerebral cells because of the osmotic gradient, causing brain swelling and onset of symptoms. The syndrome most often occurs in clients who are new to dialysis and is prevented by dialyzing for shorter times or at reduced blood flow rates. 12. Answer: 4. Following dialysis, the clients vital signs are monitored to determine whether the client is remaining hemodynamically stable. Weight is measured and compared with the clients predialysi s weight to determine effectiveness of fluid extraction. Laboratory studies are done as per protocol but are not necessarily done after the hemodialysis treatment has ended. 13. Answer: 2. Steal syndrome results from vascular insufficiency after creation of a fistula. The client exhibits pallor and a diminished pulse distal to the fistula. The client also complains of pain distal to the fistula, which is due to tissue ischemia. Warmth, redness, and pain more likely would characterize a problem with infection. 14. Answer: 1. Polyuria occurs early in chronic renal failure and if untreated can cause severe dehydration. Polyuria progresses to anuria, and the client loses all normal functions of the kidney. Oliguria and anuria are not early signs, and polydipsia is unrelated to chronic renal failure. 15. Answer: 4. The client may have an elevated temperature following dialysis because the dialysis machine warms the blood slightly. If the temperature is elevated excessively and remains elevated, sepsis would be suspected and a blood sample would be obtained as prescribed for culture and sensitivity purposes. 16. Answer: 1. Disequilibrium syndrome may be due to the rapid decrease in BUN levels during dialysis. These changes can cause cerebral edema that leads to increased intracranial pressure. The client is exhibiting early signs of disequilibrium syndrome and appropriate treatments with anticonvulsant medications and barbituates may be necessary to prevent a life-threatening situation. The physician must be notified.

55

17. Answer: Cantaloupe (1/4 small), spinach (1/2 cooked) and strawberries (1 cups) are high potassium foods and average 7 mEq per serving. Lima beans (1/3 c) averages 3 mEq per serving. 18. Answer: 4. Increasing the glucose concentration makes the solution increasingly more hypertonic. The more hypertonic the solution, the greater the osmotic pressure for ultrafiltration and thus the greater amount of fluid removed from the client during an exchange. 19. Answer: 2. The major complication of peritoneal dialysis is peritonitis. Strict aseptic technique is required in caring for the client receiving this treatment. Although option 4 may assist in preventing infection, this option relates to an external site. 20. Answer: 3. Pain during the inflow of dialysate is common during the first few exchanges because of peritoneal irritation; however, the pain usually disappears after 1 to 2 weeks of treatment. The infusion amount should not be decreased, and the infusion should not be slowed or stopped. 21. Answer: 2. An extended dwell time increases the risk of hyperglycemia in the client with diabetes mellitus as a result of absorption of glucose from the dialysate and electrolyte changes. Diabetic clients may require extra insulin when receiving peritoneal dialysis. 22. Answer: 3. The client with hyperkalemia is at risk for developing cardiac dysrhythmias and cardiac arrest. Because of this the client should be placed on a cardiac monitor. Fluid intake is not increased because it contributes to fluid overload and would not affect the serum potassium level significantly. Vegetables are a natural source of potassium in the diet, and their use would not be increased. The nurse may also assess the sodium level because sodium is another electrolyte commonly measured with the potassium level. However, this is not a priority action at this time. 23. Answer: 3. Antihypertensive medications such as enalapril are given to the client following hemodialysis. This prevents the client from becoming hypotensive during dialysis and also from having the medication removed from the bloodstream by dialysis. No rationale exists for waiting a full day to resume the medication. This would lead to ineffective control of the blood pressure. 24. Answer: 2. Clients with peritoneal dialysis catheters are at high risk for infection. A dressing that is wet is a conduit for bacteria for bacteria to reach the catheter insertion site. The nurse assures that the dressing is kept dry at all times. Reinforcing the dressing is not a safe practice to prevent infection in this circumstance. Flushing the catheter is not indicated. Scrubbing the catheter with povidone-iodine is done at the time of connection or disconnecting of peritoneal dialysis. 25. Answer: 2. If the client experiences air embolus during hemodialysis, the nurse should terminate dialysis immediately, notify the physician, and administer oxygen as needed. 26. Answer: 2. The client on hemodialysis should monitor fluid status between hemodialysis treatments by recording intake and output and measuring weight daily. Ideally, the hemodialysis client should not gain more than 0.5 kg of weight per day. 27. Answer: 4. An AV shunt is a less common form of access site but carries a risk for bleeding when it is used because two ends of an external cannula are tunneled subcutaneously into an artery and a vein, and the ends of the cannula are joined. If accidental connection occurs, the client could lose blood rapidly. For this reason, small clamps are attached to the dressing that covers the insertion site to use if needed. The shunt site should be assessed at least every four hours. 28. Answer: 1, 2, 4, 5. If outflow drainage is inadequate, the nurse attempts to stimulate outflow by changing the clients p osition. Turning the client to the other side or making sure that the client is in good body alignment may assist with outflow drainage. The drainage bag needs to be lower than the clients abdomen to enhance gravity drainage. The connecting tubing and the peritonea l dialysis system is also checked for kinks or twisting and the clamps on the system are checked to ensure that they are open. There is no reason to contact the physician. 29. Answer: 1. Crackles in the lungs, weight gain, and elevated blood pressure are indicators of excess fluid volume, a common complication in chronic renal failure. The clients fluid status should be monitored carefully for imbalances on an ongoing b asis. 30. Answer: 1, 2, 3. Appropriate nursing diagnoses for clients with chronic renal failure include excess fluid volume related to fluid and sodium retention; imbalanced nutrition, less than body requirements related to anorexia, nausea, and vomiting; and activity intolerance related to fatigue. The nursing diagnoses of impaired gas exchange and pain are not commonly related to chronic renal failure.

56

31. Answer: 3. The disadvantages of peritoneal dialysis in long-term management of chronic renal failure is that is requires large blocks of time. The risk of hemorrhage or hepatitis is not high with PD. PD is effective in maintaining a clients fluid and electrolyte balance. 32. Answer: 1. The main reason for warming the peritoneal dialysis solution is that the warm solution helps dilate peritoneal vessels, which increases urea clearance. Warmed dialyzing solution also contributes to client comfort by preventing chilly sensations, but this is a secondary reason for warming the solution. The warmed solution does not force potassium into the cells or promote abdominal muscle relaxation. 33. Answer: 2. Because the client has a permanent catheter in place, blood tinged drainage should not occur. Persistent blood tinged drainage could indicate damage to the abdominal vessels, and the physician should be notified. The bleeding is originating in the peritoneal cavity, not the kidneys. Too rapid infusion of the dialysate can cause pain. 34. Answer: 2. Because hypotension is a complication of peritoneal dialysis, the nurse records intake and output, monitors VS, and observes the clients behavior. The nurse also encourages visiting and other diversional activities. A client on PD does not need to be placed in bed with padded side rails or kept NPO. 35. Answer: 3. A client in renal failure develops hyperphosphatemia that causes a corresponding excretion of the bodys calci um stores, leading to renal osteodystrophy. To decrease this loss, aluminum hydroxide gel is prescribed to bind phosphates in the intestine and facilitate their excretion. Gastric hyperacidity is not necessarily a problem associated with chronic renal failure. Antacids will not prevent Curlings stress ulcers and do not affect metabolic acidosis. 36. Answer: 3. Aluminum hydroxide gel is administered to bind the phosphates in ingested foods and must be given with or immediately after meals and snacks. There is no need for the client to take it on a 24-hour schedule. It is not administered to treat hyperacidity in clients with CRF and therefore is not prescribed between meals. 37. Answer: 1. Magnesium is normally excreted by the kidneys. When the kidneys fail, magnesium can accumulate and cause severe neurologic problems. MOM is harsher than Metamucil, but magnesium toxicity is a more serious problem. A client may find both MOM and Metamucil unpalatable. MOM is not high in sodium. 38. Answer: 2. Uremia can cause decreased alertness, so the nurse needs to validate the clients comprehension frequently. Because the clients ability to concentrate is limited, short lesions are most effective. If family members are present at th e sessions, they can reinforce the material. Written materials that the client can review are superior to videotapes, because the clients may not be able to maintain alertness during the viewing of the videotape. 39. Answer: 3. Dietary management for clients with chronic renal failure is usually designed to restrict protein, sodium, and potassium intake. Protein intake is reduced because the kidney can no longer excrete the byproducts of protein metabolism. The degree of dietary restriction depends on the degree of renal impairment. The client should also receive a high carbohydrate diet along with appropriate vitamin and mineral supplements. Calcium requirements remain 1,000 to 2,000 mg/day. 40. Answer: 2. The major benefit of CAPD is that it frees the client from daily dependence on dialysis centers, home health care personnel, and machines for life-sustaining treatment. The independence is a valuable outcome for some people. CAPD is costly and must be done daily. Side effects and complications are similar to those of standard peritoneal dialysis. 41. Answer: 3. Dietary restrictions with CAPD are fewer than those with standard peritoneal dialysis because dialysis is constant, not intermittent. The constant slow diffusion of CAPD helps prevent accumulation of toxins and allows for a more liberal diet. CAPD does not work more quickly, but more consistently. Both types of peritoneal dialysis are effective. 42. Answer: 1. Cloudy drainage indicates bacterial activity in the peritoneum. Other signs and symptoms of infection are fever, hyperactive bowel sounds, and abdominal pain. Swollen legs may be indicative of congestive heart failure. Poor drainage of dialysate fluid is probably the result of a kinked catheter. Redness at the insertion site indicates local infection, not peritonitis. However, a local infection that is left untreated can progress to the peritoneum. 43. Answer: 4. Hyperkalemia 44. Answer: 3. Exsanguination 45. Answer: 3. User surgical aseptic technique when giving shunt care

Introduction
57

A 25-item NCLEX-style examination about Neurological Disorders that covers topics like: Meningitis, Level of Consciousness, Seizures and more.

Topics

Increased Intracranial Pressure Level of Consciousness Meningitis

Guidelines
Read each question carefully and choose the best answer. You are given one minute per question. Spend your time wisely! Answers and rationales are given below. Be sure to read them. If you need more clarifications, please direct them to the comments section.

Questions
1. A client admitted to the hospital with a subarachnoid hemorrhage has complaints of severe headache, nuchal rigidity, and projectile vomiting. The nurse knows lumbar puncture (LP) would be contraindicated in this client in which of the following circumstances? 1. Vomiting continues 2. Intracranial pressure (ICP) is increased 3. The client needs mechanical ventilation 4. Blood is anticipated in the cerebrospinal fluid (CSF) 2. A client with a subdural hematoma becomes restless and confused, with dilation of the ipsilateral pupil. The physician orders mannitol for which of the following reasons? 1. To reduce intraocular pressure 2. To prevent acute tubular necrosis 3. To promote osmotic diuresis to decrease ICP 4. To draw water into the vascular system to increase blood pressure 3. A client with subdural hematoma was given mannitol to decrease intracranial pressure (ICP). Which of the following results would best show the mannitol was effective? 1. Urine output increases 2. Pupils are 8 mm and nonreactive 3. Systolic blood pressure remains at 150 mm Hg 4. BUN and creatinine levels return to normal 4. Which of the following values is considered normal for ICP? 1. 0 to 15 mm Hg 2. 25 mm Hg 3. 35 to 45 mm Hg 4. 120/80 mm Hg 5. Which of the following symptoms may occur with a phenytoin level of 32 mg/dl? 1. Ataxia and confusion 2. Sodium depletion 3. Tonic-clonic seizure 4. Urinary incontinence 6. Which of the following signs and symptoms of increased ICP after head trauma would appear first? 1. Bradycardia 2. Large amounts of very dilute urine

58

3. Restlessness and confusion 4. Widened pulse pressure 7. Problems with memory and learning would relate to which of the following lobes? 1. Frontal 2. Occipital 3. Parietal 4. Temporal 8. While cooking, your client couldnt feel the temperature of a hot oven. Which lobe could be dysfunctional? 1. Frontal 2. Occipital 3. Parietal 4. Temporal 9. The nurse is assessing the motor function of an unconscious client. The nurse would plan to use which of the following to test the clients peripheral response to pain? 1. Sternal rub 2. Pressure on the orbital rim 3. Squeezing the sternocleidomastoid muscle 4. Nail bed pressure 10. The client is having a lumbar puncture performed. The nurse would plan to place the client in which position for the procedure? 1. Side-lying, with legs pulled up and head bent down onto the chest 2. Side-lying, with a pillow under the hip 3. Prone, in a slight Trendelenburgs position 4. Prone, with a pillow under the abdomen. 11. A nurse is assisting with caloric testing of the oculovestibular reflex of an unconscious client. Cold water is injected into the left auditory canal. The client exhibits eye conjugate movements toward the left followed by a rapid nystagmus toward the right. The nurse understands that this indicates the client has: 1. A cerebral lesion 2. A temporal lesion 3. An intact brainstem 4. Brain death 12. The nurse is caring for the client with increased intracranial pressure. The nurse would note which of the following trends in vital signs if the ICP is rising? 1. Increasing temperature, increasing pulse, increasing respirations, decreasing blood pressure. 2. Increasing temperature, decreasing pulse, decreasing respirations, increasing blood pressure. 3. Decreasing temperature, decreasing pulse, increasing respirations, decreasing blood pressure. 4. Decreasing temperature, increasing pulse, decreasing respirations, increasing blood pressure. 13. The nurse is evaluating the status of a client who had a craniotomy 3 days ago. The nurse would suspect the client is developing meningitis as a complication of surgery if the client exhibits: 1. A positive Brudzinskis sign 2. A negative Kernigs sign 3. Absence of nuchal rigidity 4. A Glascow Coma Scale score of 15 14. A client is arousing from a coma and keeps saying, Just stop the pain. The nurse responds based on the knowledge that the human body typically and automatically responds to pain first with attempts to: 1. Tolerate the pain 2. Decrease the perception of pain

59

3. Escape the source of pain 4. Divert attention from the source of pain. 15. During the acute stage of meningitis, a 3-year-old child is restless and irritable. Which of the following would be most appropriate to institute? 1. Limiting conversation with the child 2. Keeping extraneous noise to a minimum 3. Allowing the child to play in the bathtub 4. Performing treatments quickly 16. Which of the following would lead the nurse to suspect that a child with meningitis has developed disseminated intravascular coagulation? 1. Hemorrhagic skin rash 2. Edema 3. Cyanosis 4. Dyspnea on exertion 17. When interviewing the parents of a 2-year-old child, a history of which of the following illnesses would lead the nurse to suspect pneumococcal meningitis? 1. Bladder infection 2. Middle ear infection 3. Fractured clavicle 4. Septic arthritis 18. The nurse is assessing a child diagnosed with a brain tumor. Which of the following signs and symptoms would the nurse expect the child to demonstrate? Select all that apply. 1. Head tilt 2. Vomiting 3. Polydipsia 4. Lethargy 5. Increased appetite 6. Increased pulse 19. A lumbar puncture is performed on a child suspected of having bacterial meningitis. CSF is obtained for analysis. A nurse reviews the results of the CSF analysis and determines that which of the following results would verify the diagnosis? 1. Cloudy CSF, decreased protein, and decreased glucose 2. Cloudy CSF, elevated protein, and decreased glucose 3. Clear CSF, elevated protein, and decreased glucose 4. Clear CSF, decreased pressure, and elevated protein 20. A nurse is planning care for a child with acute bacterial meningitis. Based on the mode of transmission of this infection, which of the following would be included in the plan of care? 1. No precautions are required as long as antibiotics have been started 2. Maintain enteric precautions 3. Maintain respiratory isolation precautions for at least 24 hours after the initiation of antibiotics 4. Maintain neutropenic precautions 21. A nurse is reviewing the record of a child with increased ICP and notes that the child has exhibited signs of decerebrate posturing. On assessment of the child, the nurse would expect to note which of the following if this type of posturing was present? 1. Abnormal flexion of the upper extremities and extension of the lower extremities 2. Rigid extension and pronation of the arms and legs

60

3. Rigid pronation of all extremities 4. Flaccid paralysis of all extremities 22. Which of the following assessment data indicated nuchal rigidity? 1. Positive Kernigs sign 2. Negative Brudzinskis sign 3. Positive homans sign 4. Negative Kernigs sign 23. Meningitis occurs as an extension of a variety of bacterial infections due to which of the following conditions? 1. Congenital anatomic abnormality of the meninges 2. Lack of acquired resistance to the various etiologic organisms 3. Occlusion or narrowing of the CSF pathway 4. Natural affinity of the CNS to certain pathogens 24. Which of the following pathologic processes is often associated with aseptic meningitis? 1. Ischemic infarction of cerebral tissue 2. Childhood diseases of viral causation such as mumps 3. Brain abscesses caused by a variety of pyogenic organisms 4. Cerebral ventricular irritation from a traumatic brain injury 25. You are preparing to admit a patient with a seizure disorder. Which of the following actions can you delegate to LPN/LVN? 1. Complete admission assessment. 2. Set up oxygen and suction equipment. 3. Place a padded tongue blade at bedside. 4. Pad the side rails before patient arrives.

Answers and Rationale


1. Answer: 2. Sudden removal of CSF results in pressures lower in the lumbar area than the brain and favors herniation of the brain; therefore, LP is contraindicated with increased ICP. Vomiting may be caused by reasons other than increased ICP; therefore, LP isnt strictly contraindicated. An LP may be performed on clients needing mechanical ventilation. Blood in the CSF is diagnostic for subarachnoid hemorrhage and was obtained before signs and symptoms of ICP. 2. Answer: 3. Mannitol promotes osmotic diuresis by increasing the pressure gradient, drawing fluid from intracellular to intravascular spaces. Although mannitol is used for all the reasons described, the reduction of ICP in this client is a concern. 3. Answer: 1. Mannitol promotes osmotic diuresis by increasing the pressure gradient in the renal tubes. Fixed and dilated pupils are symptoms of increased ICP or cranial nerve damage. No information is given about abnormal BUN and creatinine levels or that mannitol is being given for renal dysfunction or blood pressure maintenance. 4. Answer: 1. Normal ICP is 0-15 mm Hg. 5. Answer: 1. A therapeutic phenytoin level is 10 to 20 mg/dl. A level of 32 mg/dl indicates toxicity. Symptoms of toxicity include confusion and ataxia. Phenytoin doesnt cause hyponatremia, seizure, or urinary incontinence. Incontinence may occur during or after a seizure. 6. Answer: 3. The earliest symptom of elevated ICP is a change in mental status. Bradycardia, widened pulse pressure, and bradypnea occur later. The client may void large amounts of very dilute urine if theres damage to the posterior p ituitary. 7. Answer: 4. The temporal lobe functions to regulate memory and learning problems because of the integration of the hippocampus. The frontal lobe primarily functions to regulate thinking, planning, and judgment. The occipital lobe functions regulate vision. The parietal lobe primarily functions with sensory function. 8. Answer: 3. The parietal lobe regulates sensory function, which would include the ability to sense hot or cold objects. The frontal lobe regulates thinking, planning, and judgment, and the occipital lobe is primarily responsible for vision function. The temporal lobe regulates memory.

61

9. Answer: 4. Motor testing on the unconscious client can be done only by testing response to painful stimuli. Nail Bed pressure tests a basic peripheral response. Cerebral responses to pain are testing using sternal rub, placing upward pressure on the orbital rim, or squeezing the clavicle or sternocleidomastoid muscle. 10. Answer: 1. The client undergoing lumbar puncture is positioned lying on the side, with the legs pulled up to the abdomen, and with the head bent down onto the chest. This position helps to open the spaces between the vertebrae. 11. Answer: 3. Caloric testing provides information about differentiating between cerebellar and brainstem lesions. After determining patency of the ear canal, cold or warm water is injected in the auditory canal. A normal response that indicates intact function of cranial nerves III, IV, and VIII is conjugate eye movements toward the side being irrigated, followed by rapid nystagmus to the opposite side. Absent or disconjugate eye movements indicate brainstem damage. 12. Answer: 2. A change in vital signs may be a late sign of increased intracranial pressure. Trends include increasing temperature and blood pressure and decreasing pulse and respirations. Respiratory irregularities also may arise. 13. Answer: 1. Signs of meningeal irritation compatible with meningitis include nuchal rigidity, positive Brudzinskis sign, and positive Kernigs sign. Nuchal rigidity is characterized by a stiff neck and soreness, which is especially noticeable when the neck is fixed. Kernigs sign is positive when the client feels pain and spasm of the hamstring muscles when the knee and thigh are extended from a flexed-right angle position. Brudzinskis sign is positive when the client flexes the hips and knees in response to the nurse gently flexing the head and neck onto the chest. A Glascow Coma Scale of 15 is a perfect score and indicates the client is awake and alert with no neurological deficits. 14. Answer: 3. The clients innate responses to pain are directed initially toward escaping from the source of pain. Variatio ns in individuals tolerance and perception of pain are apparent only in conscious clients, and only conscious cl ients are able to employ distraction to help relieve pain. 15. Answer: 2. A child in the acute stage of meningitis is irritable and hypersensitive to loud noise and light. Therefore, extraneous noise should be minimized and bright lights avoided as much as possible. There is no need to limit conversations with the child. However, the nurse should speak in a calm, gentle, reassuring voice. The child needs gentle and calm bathing. Because of the acuteness of the infection, sponge baths would be more appropriate than tub baths. Although treatments need to be completed as quickly as possible to prevent overstressing the child, any treatments should be performed carefully and at a pace that avoids sudden movements to prevent startling the child and subsequently increasing intracranial pressure. 16. Answer: 1. DIC is characterized by skin petechiae and a purpuric skin rash caused by spontaneous bleeding into the tissues. An abnormal coagulation phenomenon causes the condition. 17. Answer: 2. Organisms that cause bacterial meningitis, such as pneumococci or meningococci, are commonly spread in the body by vascular dissemination from a middle ear infection. The meningitis may also be a direct extension from the paranasal and mastoid sinuses. The causative organism is a pneumococcus. A chronically draining ear is frequently also found. 18. Answer: 1, 2, 4. Head tilt, vomiting, and lethargy are classic signs assessed in a child with a brain tumor. Clinical manifestations are the result of location and size of the tumor. 19. Answer: 2. A diagnosis of meningitis is made by testing CSF obtained by lumbar puncture. In the case of bacterial meningitis, findings usually include an elevated pressure, turbid or cloudy CSF, elevated leukocytes, elevated protein, and decreased glucose levels. 20. Answer: 3. A major priority of nursing care for a child suspected of having meningitis is to administer the prescribed antibiotic as soon as it is ordered. The child is also placed on respiratory isolation for at least 24 hours while culture results are obtained and the antibiotic is having an effect. 21. Answer: 2. Decerebrate posturing is characterized by the rigid extension and pronation of the arms and legs. 22. Answer: 1. A positive Kernigs sign indicated nuchal rigidity, caused by an irritative lesion of the subarachnoid space. Brudzinskis sign is also indicative of the condition. 23. Answer: 2. Extension of a variety of bacterial infections is a major causative factor of meningitis and occurs as a result of a lack of acquired resistance to the etiologic organisms. Preexisting CNS anomalies are factors that contribute to susceptibility. 24. Answer: 2. Aseptic meningitis is caused principally by viruses and is often associated with other diseases such as measles, mumps, herpes, and leukemia. Incidences of brain abscess are high in bacterial meningitis, and ischemic infarction of cerebral tissue can occur with tubercular meningitis. Traumatic brain injury could lead to bacterial (not viral) meningitis.

62

25. Answer: 2 The LPN/LVN can set up the equipment for oxygen and suctioning. The RN should perform the complete initial assessment. Padded side rails are controversial in terms of whether they actually provide safety and ay embarrass the patient and family. Tongue blades should not be at the bedside and should never be inserted into the patients mouth after a seizure begins. Focus: Delegation/supervision.

Introduction
This is a 50-item NCLEX style examination all about diseases affecting the Neurological System. Topics included here are: Seizure, Head Trauma, Spinal Cord Injury and more.

Topics

Seizures Head Trauma Spinal Cord Injury

Guidelines
Read each question carefully and choose the best answer. You are given one minute per question. Spend your time wisely! Answers and rationales are given below. Be sure to read them. If you need more clarifications, please direct them to the comments section.

Questions
1. An 18-year-old client is admitted with a closed head injury sustained in a MVA. His intracranial pressure (ICP) shows an upward trend. Which intervention should the nurse perform first? 1. Reposition the client to avoid neck flexion 2. Administer 1 g Mannitol IV as ordered 3. Increase the ventilators respiratory rate to 20 breaths/minute 4. Administer 100 mg of pentobarbital IV as ordered. 2. A client with a subarachnoid hemorrhage is prescribed a 1,000-mg loading dose of Dilantin IV. Which consideration is most important when administering this dose? 1. Therapeutic drug levels should be maintained between 20 to 30 mg/ml. 2. Rapid dilantin administration can cause cardiac arrhythmias. 3. Dilantin should be mixed in dextrose in water before administration. 4. Dilantin should be administered through an IV catheter in the clients hand. 3. A client with head trauma develops a urine output of 300 ml/hr, dry skin, and dry mucous membranes. Which of the following nursing interventions is the most appropriate to perform initially? 1. Evaluate urine specific gravity 2. Anticipate treatment for renal failure 3. Provide emollients to the skin to prevent breakdown 4. Slow down the IV fluids and notify the physician 4. When evaluating an ABG from a client with a subdural hematoma, the nurse notes the PaCO2 is 30 mm Hg. Which of the following responses best describes this result? 1. Appropriate; lowering carbon dioxide (CO2) reduces intracranial pressure (ICP). 2. Emergent; the client is poorly oxygenated. 3. Normal 4. Significant; the client has alveolar hypoventilation.

63

5. A client who had a transsphenoidal hypophysectomy should be watched carefully for hemorrhage, which may be shown by which of the following signs? 1. Bloody drainage from the ears 2. Frequent swallowing 3. Guaiac-positive stools 4. Hematuria 6. After a hypophysectomy, vasopressin is given IM for which of the following reasons? 1. To treat growth failure 2. To prevent syndrome of inappropriate antidiuretic hormone (SIADH) 3. To reduce cerebral edema and lower intracranial pressure 4. To replace antidiuretic hormone (ADH) normally secreted by the pituitary. 7. A client comes into the ER after hitting his head in an MVA. Hes alert and oriented. Which of the following nursing interventions should be done first? 1. Assess full ROM to determine extent of injuries 2. Call for an immediate chest x-ray 3. Immobilize the clients head and neck 4. Open the airway with the head-tilt chin-lift maneuver 8. A client with a C6 spinal injury would most likely have which of the following symptoms? 1. Aphasia 2. Hemiparesis 3. Paraplegia 4. Tetraplegia 9. A 30-year-old was admitted to the progressive care unit with a C5 fracture from a motorcycle accident. Which of the following assessments would take priority? 1. Bladder distension 2. Neurological deficit 3. Pulse ox readings 4. The clients feelings about the injury 10. While in the ER, a client with C8 tetraplegia develops a blood pressure of 80/40, pulse 48, and RR of 18. The nurse suspects which of the following conditions? 1. Autonomic dysreflexia 2. Hemorrhagic shock 3. Neurogenic shock 4. Pulmonary embolism 11. A client is admitted with a spinal cord injury at the level of T12. He has limited movement of his upper extremities. Which of the following medications would be used to control edema of the spinal cord? 1. Acetazolamide (Diamox) 2. Furosemide (Lasix) 3. Methylprednisolone (Solu-Medrol) 4. Sodium bicarbonate 12. A 22-year-old client with quadriplegia is apprehensive and flushed, with a blood pressure of 210/100 and a heart rate of 50 bpm. Which of the following nursing interventions should be done first? 1. Place the client flat in bed 2. Assess patency of the indwelling urinary catheter 3. Give one SL nitroglycerin tablet 4. Raise the head of the bed immediately to 90 degrees 13. A client with a cervical spine injury has Gardner-Wells tongs inserted for which of the following reasons?

64

1. To hasten wound healing 2. To immobilize the cervical spine 3. To prevent autonomic dysreflexia 4. To hold bony fragments of the skull together 14. Which of the following interventions describes an appropriate bladder program for a client in rehabilitation for spinal cord injury? 1. Insert an indwelling urinary catheter to straight drainage 2. Schedule intermittent catheterization every 2 to 4 hours 3. Perform a straight catheterization every 8 hours while awake 4. Perform Credes maneuver to the lower abdomen before the client voids. 15. A client is admitted to the ER for head trauma is diagnosed with an epidural hematoma. The underlying cause of epidural hematoma is usually related to which of the following conditions? 1. Laceration of the middle meningeal artery 2. Rupture of the carotid artery 3. Thromboembolism from a carotid artery 4. Venous bleeding from the arachnoid space 16. A 23-year-old client has been hit on the head with a baseball bat. The nurse notes clear fluid draining from his ears and nose. Which of the following nursing interventions should be done first? 1. Position the client flat in bed 2. Check the fluid for dextrose with a dipstick 3. Suction the nose to maintain airway patency 4. Insert nasal and ear packing with sterile gauze 17. When discharging a client from the ER after a head trauma, the nurse teaches the guardian to observe for a lucid interval. Which of the following statements best described a lucid interval? 1. An interval when the clients speech is garbled 2. An interval when the client is alert but cant recall recent events 3. An interval when the client is oriented but then becomes somnolent 4. An interval when the client has a warning symptom, such as an odor or visual disturbance. 18. Which of the following clients on the rehab unit is most likely to develop autonomic dysreflexia? 1. A client with a brain injury 2. A client with a herniated nucleus pulposus 3. A client with a high cervical spine injury 4. A client with a stroke 19. Which of the following conditions indicates that spinal shock is resolving in a client with C7 quadriplegia? 1. Absence of pain sensation in chest 2. Spasticity 3. Spontaneous respirations 4. Urinary continence 20. A nurse assesses a client who has episodes of autonomic dysreflexia. Which of the following conditions can cause autonomic dysreflexia? 1. Headache 2. Lumbar spinal cord injury 3. Neurogenic shock 4. Noxious stimuli 21. During an episode of autonomic dysreflexia in which the client becomes hypertensive, the nurse should perform which of the following interventions?

65

1. Elevate the clients legs 2. Put the client flat in bed 3. Put the client in the Trendelenburgs position 4. Put the client in the high-Fowlers position 22. A client with a T1 spinal cord injury arrives at the emergency department with a BP of 82/40, pulse 34, dry skin, and flaccid paralysis of the lower extremities. Which of the following conditions would most likely be suspected? 1. Autonomic dysreflexia 2. Hypervolemia 3. Neurogenic shock 4. Sepsis 23. A client has a cervical spine injury at the level of C5. Which of the following conditions would the nurse anticipate during the acute phase? 1. Absent corneal reflex 2. Decerebrate posturing 3. Movement of only the right or left half of the body 4. The need for mechanical ventilation 24. A client with C7 quadriplegia is flushed and anxious and complains of a pounding headache. Which of the following symptoms would also be anticipated? 1. Decreased urine output or oliguria 2. Hypertension and bradycardia 3. Respiratory depression 4. Symptoms of shock 25. A 40-year-old paraplegic must perform intermittent catheterization of the bladder. Which of the following instructions should be given? 1. Clean the meatus from back to front. 2. Measure the quantity of urine. 3. Gently rotate the catheter during removal. 4. Clean the meatus with soap and water. 26. An 18-year-old client was hit in the head with a baseball during practice. When discharging him to the care of his mother, the nurse gives which of the following instructions? 1. Watch him for keyhole pupil the next 24 hours. 2. Expect profuse vomiting for 24 hours after the injury. 3. Wake him every hour and assess his orientation to person, time, and place. 4. Notify the physician immediately if he has a headache. 27. Which neurotransmitter is responsible for may of the functions of the frontal lobe? 1. Dopamine 2. GABA 3. Histamine 4. Norepinephrine 28. The nurse is discussing the purpose of an electroencephalogram (EEG) with the family of a client with massive cerebral hemorrhage and loss of consciousness. It would be most accurate for the nurse to tell family members that the test measures which of the following conditions? 1. Extent of intracranial bleeding 2. Sites of brain injury 3. Activity of the brain 4. Percent of functional brain tissue

66

29. A client arrives at the ER after slipping on a patch of ice and hitting her head. A CT scan of the head shows a collection of blood between the skull and dura mater. Which type of head injury does this finding suggest? 1. Subdural hematoma 2. Subarachnoid hemorrhage 3. Epidural hematoma 4. Contusion 30. After falling 20, a 36-year-old man sustains a C6 fracture with spinal cord transaction. Which other findings should the nurse expect? 1. Quadriplegia with gross arm movement and diaphragmatic breathing 2. Quadriplegia and loss of respiratory function 3. Paraplegia with intercostal muscle loss 4. Loss of bowel and bladder control 31. A 20-year-old client who fell approximately 30 is unresponsive and breathless. A cervical spine injury is suspected. How should the first-responder open the clients airway for rescue breathing? 1. By inserting a nasopharyngeal airway 2. By inserting a oropharyngeal airway 3. By performing a jaw-thrust maneuver 4. By performing the head-tilt, chin-lift maneuver 32. The nurse is caring for a client with a T5 complete spinal cord injury. Upon assessment, the nurse notes flushed skin, diaphoresis above the T5, and a blood pressure of 162/96. The client reports a severe, pounding headache. Which of the following nursing interventions would be appropriate for this client? Select all that apply. 1. Elevate the HOB to 90 degrees 2. Loosen constrictive clothing 3. Use a fan to reduce diaphoresis 4. Assess for bladder distention and bowel impaction 5. Administer antihypertensive medication 6. Place the client in a supine position with legs elevated 33. The client with a head injury has been urinating copious amounts of dilute urine through the Foley catheter. The clients urine output for the previous shift was 3000 ml. The nurse implements a new physician order to administer: 1. Desmopressin (DDAVP, stimate) 2. Dexamethasone (Decadron) 3. Ethacrynic acid (Edecrin) 4. Mannitol (Osmitrol) 34. The nurse is caring for the client in the ER following a head injury. The client momentarily lost consciousness at the time of the injury and then regained it. The client now has lost consciousness again. The nurse takes quick action, knowing this is compatible with: 1. Skull fracture 2. Concussion 3. Subdural hematoma 4. Epidural hematoma 35. The nurse is caring for a client who suffered a spinal cord injury 48 hours ago. The nurse monitors for GI complications by assessing for: 1. A flattened abdomen 2. Hematest positive nasogastric tube drainage 3. Hyperactive bowel sounds 4. A history of diarrhea

67

36. A client with a spinal cord injury is prone to experiencing autonomic dysreflexia. The nurse would avoid which of the following measures to minimize the risk of recurrence? 1. Strict adherence to a bowel retraining program 2. Limiting bladder catheterization to once every 12 hours 3. Keeping the linen wrinkle-free under the client 4. Preventing unnecessary pressure on the lower limbs 37. The nurse is planning care for the client in spinal shock. Which of the following actions would be least helpful in minimizing the effects of vasodilation below the level of the injury? 1. Monitoring vital signs before and during position changes 2. Using vasopressor medications as prescribed 3. Moving the client quickly as one unit 4. Applying Teds or compression stockings. 38. The nurse is caring for a client admitted with spinal cord injury. The nurse minimizes the risk of compounding the injury most effectively by: 1. Keeping the client on a stretcher 2. Logrolling the client on a firm mattress 3. Logrolling the client on a soft mattress 4. Placing the client on a Stryker frame 39. The nurse is evaluating neurological signs of the male client in spinal shock following spinal cord injury. Which of the following observations by the nurse indicates that spinal shock persists? 1. Positive reflexes 2. Hyperreflexia 3. Inability to elicit a Babinskis reflex 4. Reflex emptying of the bladder 40. A client with a spinal cord injury suddenly experiences an episode of autonomic dysreflexia. After checking the clients vital signs, list in order of priority, the nurses actions (Number 1 being the first priority and number 5 being the last priority). 1. Check for bladder distention 2. Raise the head of the bed 3. Contact the physician 4. Loosen tight clothing on the client 5. Administer an antihypertensive medication 41. A client is at risk for increased ICP. Which of the following would be a priority for the nurse to monitor? 1. Unequal pupil size 2. Decreasing systolic blood pressure 3. Tachycardia 4. Decreasing body temperature 42. Which of the following respiratory patterns indicate increasing ICP in the brain stem? 1. Slow, irregular respirations 2. Rapid, shallow respirations 3. Asymmetric chest expansion 4. Nasal flaring 43. Which of the following nursing interventions is appropriate for a client with an ICP of 20 mm Hg? 1. Give the client a warming blanket 2. Administer low-dose barbiturate 3. Encourage the client to hyperventilate 4. Restrict fluids

68

44. A client has signs of increased ICP. Which of the following is an early indicator of deterioration in the clients condition? 1. Widening pulse pressure 2. Decrease in the pulse rate 3. Dilated, fixed pupil 4. Decrease in LOC 45. A client who is regaining consciousness after a craniotomy becomes restless and attempts to pull out her IV line. Which nursing intervention protects the client without increasing her ICP? 1. Place her in a jacket restraint 2. Wrap her hands in soft mitten restraints 3. Tuck her arms and hands under the draw sheet 4. Apply a wrist restraint to each arm 46. Which of the following describes decerebrate posturing? 1. Internal rotation and adduction of arms with flexion of elbows, wrists, and fingers 2. Back hunched over, rigid flexion of all four extremities with supination of arms and plantar flexion of the feet 3. Supination of arms, dorsiflexion of feet 4. Back arched; rigid extension of all four extremities. 47. A client receiving vent-assisted mode ventilation begins to experience cluster breathing after recent intracranial occipital bleeding. Which action would be most appropriate? 1. Count the rate to be sure the ventilations are deep enough to be sufficient 2. Call the physician while another nurse checks the vital signs and ascertains the patients Glasgow Coma score. 3. Call the physician to adjust the ventilator settings. 4. Check deep tendon reflexes to determine the best motor response 48. In planning the care for a client who has had a posterior fossa (infratentorial) craniotomy, which of the following is contraindicated when positioning the client? 1. Keeping the client flat on one side or the other 2. Elevating the head of the bed to 30 degrees 3. Log rolling or turning as a unit when turning 4. Keeping the head in neutral position 49. A client has been pronounced brain dead. Which findings would the nurse assess? Check all that apply. 1. Decerebrate posturing 2. Dilated nonreactive pupils 3. Deep tendon reflexes 4. Absent corneal reflex 50. A 23-year-old patient with a recent history of encephalitis is admitted to the medical unit with new onset generalized tonic-clonic seizures. Which nursing activities included in the patients care will be best to delegate to an LPN/LVN whom you are supervising? (Choose all that apply). 1. Document the onset time, nature of seizure activity, and postictal behaviors for all seizures. 2. Administer phenytoin (Dilantin) 200 mg PO daily. 3. Teach patient about the need for good oral hygiene. 4. Develop a discharge plan, including physician visits and referral to the Epilepsy Foundation.

Answers and Rationale


1. Answer: 1. The nurse should first attempt nursing interventions, such as repositioning the client to avoid neck flexion, which increases venous return and lowers ICP. If nursing measures prove ineffective, notify the physician, who may prescribe mannitol, pentobarbital, or hyperventilation therapy. 2. Answer: 2. Dilantin IV shouldnt be given at a rate exceeding 50 mg/minute. Rapid administration can depress the myocardium, causing arrhythmias. Therapeutic drug levels range from 10 to 20 mg/ml. Dilantin shouldnt be mixed in solution

69

for administration. However, because its compatible with normal saline solution, it can be injected through an IV line conta ining normal saline. When given through an IV catheter hand, dilantin may cause purple glove syndrome. 3. Answer: 1. Urine output of 300 ml/hr may indicate diabetes insipidus, which is a failure of the pituitary to produce anti-diuretic hormone. This may occur with increased intracranial pressure and head trauma; the nurse evaluates for low urine specific gravity, increased serum osmolarity, and dehydration. Theres no evidence that the client is experiencing renal failure. Providing emollients to prevent skin breakdown is important, but doesnt need to be performe d immediately. Slowing the rate of IV fluid would contribute to dehydration when polyuria is present. 4. Answer: 1. A normal PaCO2 value is 35 to 45 mm Hg. CO2 has vasodilating properties; therefore, lowering PaCO2 through hyperventilation will lower ICP caused by dilated cerebral vessels. Oxygenation is evaluated through PaO2 and oxygen saturation. Alveolar hypoventilation would be reflected in an increased PaCO2. 5. Answer: 2. Frequent swallowing after brain surgery may indicate fluid or blood leaking from the sinuses into the oropharynx. Blood or fluid draining from the ear may indicate a basilar skull fracture. 6. Answer: 4. After hypophysectomy, or removal of the pituitary gland, the body cant synthesize ADH. Somatropin or growth hormone, not vasopressin is used to treat growth failure. SIADH results from excessive ADH secretion. Mannitol or corticosteroids are used to decrease cerebral edema. 7. Answer: 3. All clients with a head injury are treated as if a cervical spine injury is present until x-rays confirm their absence. ROM would be contraindicated at this time. There is no indication that the client needs a chest x-ray. The airway doesnt need to be opened since the client appears alert and not in respiratory distress. In addition, the head-tilt chin-lift maneuver wouldnt be used until the cervical spine injury is ruled out. 8. Answer: 4. Tetraplegia occurs as a result of cervical spine injuries. Paraplegia occurs as a result of injury to the thoracic cord and below. 9. Answer: 3. After a spinal cord injury, ascending cord edema may cause a higher level of injury. The diaphragm is innervated at the level of C4, so assessment of adequate oxygenation and ventilation is necessary. Although the other options would be necessary at a later time, observation for respiratory failure is the priority. 10. Answer: 3. Symptoms of neurogenic shock include hypotension, bradycardia, and warm, dry skin due to the loss of adrenergic stimulation below the level of the lesion. Hypertension, bradycardia, flushing, and sweating of the skin are seen with autonomic dysreflexia. Hemorrhagic shock presents with anxiety, tachycardia, and hypotension; this wouldnt be suspected without an injury. Pulmonary embolism presents with chest pain, hypotension, hypoxemia, tachycardia, and hemoptysis; this may be a later complication of spinal cord injury due to immobility. 11. Answer: 3. High doses of Solu-Medrol are used within 24 hours of spinal injury to reduce cord swelling and limit neurological deficit. The other drugs arent indicated in this circumstance. 12. Answer: 4. Anxiety, flushing above the level of the lesion, piloerection, hypertension, and bradycardia are symptoms of autonomic dysreflexia, typically caused by such noxious stimuli such as a full bladder, fecal impaction, or decubitus ulcer. Putting the client flat will cause the blood pressure to increase even more. The indwelling urinary catheter should be assessed immediately after the HOB is raised. Nitroglycerin is given to reduce chest pain and reduce preload; it isn t used for hypertension or dysreflexia. 13. Answer: 2. Gardner-Wells, Vinke, and Crutchfield tongs immobilize the spine until surgical stabilization is accomplished. 14. Answer: 2. Intermittent catherization should begin every 2 to 4 hours early in the treatment. When residual volume is less than 400 ml, the schedule may advance to every 4 to 6 hours. Indwelling catheters may predispose the client to infection and are removed as soon as possible. Credes maneuver is not used on people with spinal cord inju ry. 15. Answer: 1. Epidural hematoma or extradural hematoma is usually caused by laceration of the middle meningeal artery. An embolic stroke is a thromboembolism from a carotid artery that ruptures. Venous bleeding from the arachnoid space is usually observed with subdural hematoma. 16. Answer: 2. Clear fluid from the nose or ear can be determined to be cerebral spinal fluid or mucous by the presence of dextrose. Placing the client flat in bed may increase ICP and promote pulmonary aspiration. The nose wou ldnt be suctioned because of the risk for suctioning brain tissue through the sinuses. Nothing is inserted into the ears or nose of a client with a skull fracture because of the risk of infection.

70

17. Answer: 3. A lucid interval is described as a brief period of unconsciousness followed by alertness; after several hours, the client again loses consciousness. Garbled speech is known as dysarthria. An interval in which the client is alert but cant r ecall recent events is known as amnesia. Warning symptoms or auras typically occur before seizures. 18. Answer: 3. Autonomic dysreflexia refers to uninhibited sympathetic outflow in clients with spinal cord injuries about the level of T10. The other clients arent prone to dysreflexia. 19. Answer: 3. Spasticity, the return of reflexes, is a sign of resolving shock. Spinal or neurogenic shock is characterized by hypotension, bradycardia, dry skin, flaccid paralysis, or the absence of reflexes below the level of injury. The absence of pain sensation in the chest doesnt apply to spinal shock. Spinal shock descends from the injury, and respiratory difficulties occur at C4 and above. 20. Answer: 4. Noxious stimuli, such as a full bladder, fecal impaction, or a decub ulcer, may cause autonomic dysreflexia. A headache is a symptom of autonomic dysreflexia, not a cause. Autonomic dysreflexia is most commonly seen with injuries at T10 or above. Neurogenic shock isnt a cause of dysreflexia. 21. Answer: 4. Putting the client in the high-Fowlers position will decrease cerebral blood flow, decreasing hypertension. Elevating the clients legs, putting the client flat in bed, or putting the bed in the Trendelenburgs position places the cl ient in positions that improve cerebral blood flow, worsening hypertension. 22. Answer: 3. Loss of sympathetic control and unopposed vagal stimulation below the level of injury typically cause hypotension, bradycardia, pallor, flaccid paralysis, and warm, dry skin in the client in neurogenic shock. Hypervolemia is indicated by rapid and bounding pulse and edema. Autonomic dysreflexia occurs after neurogenic shock abates. Signs of sepsis would include elevated temperature, increased heart rate, and increased respiratory rate. 23. Answer: 4. The diaphragm is stimulated by nerves at the level of C4. Initially, this client may need mechanical ventilation due to cord edema. This may resolve in time. Absent corneal reflexes, decerebrate posturing, and hemiplegia occur with brain injuries, not spinal cord injuries. 24. Answer: 2. Hypertension, bradycardia, anxiety, blurred vision, and flushing above the lesion occur with autonomic dysreflexia due to uninhibited sympathetic nervous system discharge. The other options are incorrect. 25. Answer: 4. Intermittent catheterization may be performed chronically with clean technique, using soap and water to clean the urinary meatus. The meatus is always cleaned from front to back in a woman, or in expanding circles working outward from the meatus in a man. It isnt necessary to measure the urine. The catheter doesnt need to be rotated during removal. 26. Answer: 3. Changes in LOC may indicate expanding lesions such as subdural hematoma; orientation and LOC are assessed frequently for 24 hours. A keyhole pupil is found after iridectomy. Profuse or projectile vomiting is a symptom of increased ICP and should be reported immediately. A slight headache may last for several days after concussion; severe or worsening headaches should be reported. 27. Answer: 1. The frontal lobe primarily functions to regulate thinking, planning, and affect. Dopamine is known to circulate widely throughout this lobe, which is why its such an important neurotransmitter in schizophrenia. 28. Answer: 3. An EEG measures the electrical activity of the brain. Extent of intracranial bleeding and location of the injury site would be determined by CT or MRI. Percent of functional brain tissue would be determined by a series of tests. 29. Answer: 3. An epidural hematoma occurs when blood collects between the skull and the dura mater. In a subdural hematoma, venous blood collects between the dura mater and the arachnoid mater. In a subarachnoid hemorrhage, blood collects between the pia mater and arachnoid membrane. A contusion is a bruise on the brains surface. 30. Answer: 1. A client with a spinal cord injury at levels C5 to C6 has quadriplegia with gross arm movement and diaphragmatic breathing. Injury levels C1 to C4 leads to quadriplegia with total loss of respiratory function. Paraplegia with intercostal muscle loss occurs with injuries at T1 to L2. Injuries below L2 cause paraplegia and loss of bowel and bladder control. 31. Answer: 3. If the client has a suspected cervical spine injury, a jaw-thrust maneuver should be used to open the airway. If the tongue or relaxed throat muscles are obstructing the airway, a nasopharyngeal or oropharyngeal airway can be inserted; however, the client must have spontaneous respirations when the airway is open. The head-tilt, chin-lift maneuver requires neck hyperextension, which can worsen the cervical spine injury. 32. Answer: 1, 2, 4, 5. The client has signs and symptoms of autonomic dysreflexia. The potentially life-threatening condition is caused by an uninhibited response from the sympathetic nervous system resulting from a lack of control over the autonomic

71

nervous system. The nurse should immediately elevate the HOB to 90 degrees and place extremities dependently to decrease venous return to the heart and increase venous return from the brain. Because tactile stimuli can trigger autonomic dysreflexia, any constrictive clothing should be loosened. The nurse should also assess for distended bladder and bowel impaction, which may trigger autonomic dysreflexia, and correct any problems. Elevated blood pressure is the most life-threatening complication of autonomic dysreflexia because it can cause stroke, MI, or seizures. If removing the triggering event doesnt reduce the clients blood pressure, IV antihypertensives should be administered. A fan shouldnt be used because cold drafts may trigger autonomi c dysreflexia. 33. Answer: 1. A complication of a head injury is diabetes insipidus, which can occur with insult to the hypothalamus, the antidiuretic storage vesicles, or the posterior pituitary gland. Urine output that exceeds 9 L per day generally requires treatment with desmopressin. Dexamethasone, a glucocorticoid, is administered to treat cerebral edema. This medication may be ordered for the head injured patient. Ethacrynic acid and mannitol are diuretics, which would be contraindicated. 34. Answer: 4. The changes in neurological signs from an epidural hematoma begin with a loss of consciousness as arterial blood collects in the epidural space and exerts pressure. The client regains consciousness as the cerebral spinal fluid is reabsorbed rapidly to compensate for the rising intracranial pressure. As the compensatory mechanisms fail, even small amounts of additional blood can cause the intracranial pressure to rise rapidly, and the clients neurological status deteriorates quick ly. 35. Answer: 2. After spinal cord injury, the client can develop paralytic ileus, which is characterized by the absence of bowel sounds and abdominal distention. Development of a stress ulcer can be detected by hematest positive NG tube aspirate or stool. A history of diarrhea is irrelevant. 36. Answer: 2. The most frequent cause of autonomic dysreflexia is a distended bladder. Straight catherization should be done every 4 to 6 hours, and Foley catheters should be checked frequently to prevent kinks in the tubing. Constipation and fecal impaction are other causes, so maintaining bowel regularity is important. Other causes include stimulation of the skin from tactile, thermal, or painful stimuli. The nurse administers care to minimize risk in these areas. 37. Answer: 3. Reflex vasodilation below the level of the spinal cord injury places the client at risk for orthostatic hypotension, which may be profound. Measures to minimize this include measuring vital signs before and during position changes, use of a tilt-table with early mobilization, and changing the clients position slowly. Venous pooling can be reduced by using Teds (compression stockings) or pneumatic boots. Vasopressor medications are administered per protocol. 38. Answer: 4. Spinal immobilization is necessary after spinal cord injury to prevent further damage and insult to the spinal cord. Whenever possible, the client is placed on a Stryker frame, which allows the nurse to turn the client to prevent complications of immobility, while maintaining alignment of the spine. If a Stryker frame is not available, a firm mattress with a bed board should be used. 39. Answer: 3. Resolution of spinal shock is occurring when there is a return of reflexes (especially flexors to noxious cutaneous stimuli), a state of hyperreflexia rather than flaccidity, reflex emptying of the bladder, and a positive Babinskis reflex. 40. Answer: 3, 1, 4, 2, 5. Autonomic dysreflexia is characterized by severe hypertension, bradycardia, severe headache, nasal stuffiness, and flushing. The cause is a noxious stimulus, most often a distended bladder or constipation. Autonomic dysreflexia is a neurological emergency and must be treated promptly to prevent a hypertensive stroke. Immediate nursing actions are to sit the client up in bed in a high-Fowlers position and remove the noxious stimulus. The nurse should loosen any tight clothing and then check for bladder distention. If the client has a foley catheter, the nurse should check for kinks in the tubing. The nurse also would check for a fecal impaction and disimpact if necessary. The physician is contacted especially if these actions do not relieve the signs and symptoms. Antihypertensive medications may be prescribed by the physician to minimize cerebral hypertension. 41. Answer: 1. Increasing ICP causes unequal pupils as a result of pressure on the third cranial nerve. Increasing ICP causes an increase in the systolic pressure, which reflects the additional pressure needed to perfuse the brain. It increases the pressure on the vagus nerve, which produces bradycardia, and it causes an increase in body temperature from hypothalamic damage. 42. Answer: 1. Neural control of respiration takes place in the brain stem. Deterioration and pressure produce irregular respiratory patterns. Rapid, shallow respirations, asymmetric chest movements, and nasal flaring are more characteristic of respiratory distress or hypoxia. 43. Answer: 3. Normal ICP is 15 mm Hg or less. Hyperventilation causes vasoconstriction, which reduces CSF and blood volume, two important factors for reducing a sustained ICP of 20 mm Hg. A cooling blanket is used to control the elevation of

72

temperature because a fever increases the metabolic rate, which in turn increases ICP. High doses of barbiturates may be used to reduce the increased cellular metabolic demands. Fluid volume and inotropic drugs are used to maintain cerebral perfusion by supporting the cardiac output and keeping the cerebral perfusion pressure greater than 80 mm Hg. 44. Answer: 4. A decrease in the clients LOC is an early indicator of deterioration of the clients neurological status. Changes in LOC, such as restlessness and irritability, may be subtle. Widening of the pulse pressure, decrease in the pulse rate, and dilated, fixed pupils occur later if the increased ICP is not treated. 45. Answer: 2. It is best for the client to wear mitts which help prevent the client from pulling on the IV without causing additional agitation. Using a jacket or wrist restraint or tucking the clients arms and hands under the draw sheet restrict movement and add to feelings of being confined, all of which would increase her agitation and increase ICP. 46. Answer: 4. Decerebrate posturing occurs in patients with damage to the upper brain stem, midbrain, or pons and is demonstrated clinically by arching of the back, rigid extension of the extremities, pronation of the arms, and plantar flexion of the feet. Internal rotation and adduction of arms with flexion of the elbows, wrists, and fingers described decorticate posturing, which indicates damage to corticospinal tracts and cerebral hemispheres. 47. Answer: 2. Cluster breathing consists of clusters of irregular breaths followed by periods of apnea on an irregular basis. A lesion in the upper medulla or lower pons is usually the cause of cluster breathing. Because the client had a bleed in the occipital lobe, which is superior and posterior to the pons and medulla, clinical manifestations that indicate a new lesion are monitored very closely in case another bleed ensues. The physician is notified immediately so that treatment can begin before respirations cease. Another nurse needs to assess vital signs and score the client according to the GCS, but time is also of the essence. Checking deep tendon reflexes is one part of the GCS analysis. 48. Answer: 2. Elevating the HOB to 30 degrees is contraindicated for infratentorial craniotomies because it could cause herniation of the brain down onto the brainstem and spinal cord, resulting in sudden death. Elevation of the head of the bed to 30 degrees with the head turned to the side opposite of the incision, if not contraindicated by the ICP; is used for supratentorial craniotomies. 49. Answer: 2, 3, 4. A client who is brain dead typically demonstrates nonreactive dilated pupils and nonreactive or absent corneal and gag reflexes. The client may still have spinal reflexes such as deep tendon and Babinski reflexes in brain death. Decerebrate or decorticate posturing would not be seen. 50. Answer: B Administration of medications is included in LPN education and scope of practice. Collection of data about the seizure activity may be accomplished by an LPN/LVN who observes initial seizure activity. An LPN/LVN would know to call the supervising RN immediately if a patient started to seize. Documentation of the seizure, patient teaching, and planning of care are complex activities that require RN level education and scope of practice.

Introduction
A 10-item examination about Neurological Disorders. The questions in this exam are a perfect for your review for the NCLEX.

Topics

Increased Intracranial Pressure Level of Consciousness Meningitis

Guidelines
Read each question carefully and choose the best answer. You are given one minute per question. Spend your time wisely! Answers and rationales are given below. Be sure to read them.

73

If you need more clarifications, please direct them to the comments section.

Questions
1. Regular oral hygiene is an essential intervention for the client who has had a stroke. Which of the following nursing measures is inappropriate when providing oral hygiene? 1. Placing the client on the back with a small pillow under the head. 2. Keeping portable suctioning equipment at the bedside. 3. Opening the clients mouth with a padded tongue blade. 4. Cleaning the clients mouth and teeth with a toothbrush . 2. A 78 year old client is admitted to the emergency department with numbness and weakness of the left arm and slurred speech. Which nursing intervention is priority? 1. Prepare to administer recombinant tissue plasminogen activator (rt-PA). 2. Discuss the precipitating factors that caused the symptoms. 3. Schedule for A STAT computer tomography (CT) scan of the head. 4. Notify the speech pathologist for an emergency consult. 3. A client arrives in the emergency department with an ischemic stroke and receives tissue plasminogen activator (t-PA) administration. Which is the priority nursing assessment? 1. Current medications. 2. Complete physical and history. 3. Time of onset of current stroke. 4. Upcoming surgical procedures. 4. During the first 24 hours after thrombolytic therapy for ischemic stroke, the primary goal is to control the clients: 1. Pulse 2. Respirations 3. Blood pressure 4. Temperature 5. What is a priority nursing assessment in the first 24 hours after admission of the client with a thrombotic stroke? 1. Cholesterol level 2. Pupil size and pupillary response 3. Vowel sounds 4. Echocardiogram 6. What is the expected outcome of thrombolytic drug therapy? 1. Increased vascular permeability. 2. Vasoconstriction. 3. Dissolved emboli. 4. Prevention of hemorrhage 7. The client diagnosed with atrial fibrillation has experienced a transient ischemic attack (TIA). Which medication would the nurse anticipate being ordered for the client on discharge? 1. An oral anticoagulant medication. 2. A beta-blocker medication. 3. An anti-hyperuricemic medication. 4. A thrombolytic medication. 8. Which client would the nurse identify as being most at risk for experiencing a CVA? 1. A 55-year-old African American male. 2. An 84-year-old Japanese female. 3. A 67-year-old Caucasian male. 4. A 39-year-old pregnant female. 9. Which assessment data would indicate to the nurse that the client would be at risk for a hemorrhagic stroke?

74

1. A blood glucose level of 480 mg/dl. 2. A right-sided carotid bruit. 3. A blood pressure of 220/120 mmHg. 4. The presence of bronchogenic carcinoma. 10. The nurse and unlicensed assistive personnel (UAP) are caring for a client with right-sided paralysis. Which action by the UAP requires the nurse to intervene? 1. The assistant places a gait belt around the clients waist prior to ambulating. 2. The assistant places the client on the back with the clients head to the side. 3. The assistant places her hand under the clients right axilla to help him/her move up in bed. 4. The assistant praises the client for attempting to perform ADLs independently.

Answers and Rationale


1. Answer: 1. A helpless client should be positioned on the side, not on the back. This lateral position helps secretions escape from the throat and mouth, minimizing the risk of aspiration. It may be necessary to suction, so having suction equipment at the bedside is necessary. Padded tongue blades are safe to use. A toothbrush is appropriate to use. 2. Answer: 3. A CT scan will determine if the client is having a stroke or has a brain tumor or another neurological disorder. This would also determine if it is a hemorrhagic or ischemic accident and guide the treatment, because only an ischemic stroke can use rt-PA. This would make (1) not the priority since if a stroke was determined to be hemorrhagic, rt-PA is contraindicated. Discuss the precipitating factors for teaching would not be a priority and slurred speech would as indicate interference for teaching. Referring the client for speech therapy would be an intervention after the CVA emergency treatment is administered according to protocol. 3. Answer: 3. The time of onset of a stroke to t-PA administration is critical. Administration within 3 hours has better outcomes. A complete history is not possible in emergency care. Upcoming surgical procedures will need to be delay if t-PA is administered. Current medications are relevant, but onset of current stroke takes priority. 4. Answer: 3. Controlling the blood pressure is critical because an intracerebral hemorrhage is the major adverse effect of thrombolytic therapy. Blood pressure should be maintained according to physician and is specific to the clients ischemic tis sue needs and risks of bleeding from treatment. Other vital signs are monitored, but the priority is blood pressure. 5. Answer: 2. It is crucial to monitor the pupil size and pupillary response to indicate changes around the cranial nerves. Cholesterol level is an assessment to be addressed for long-term healthy lifestyle rehabilitation. Bowel sounds need to be assessed because an ileus or constipation can develop, but is not a priority in the first 24 hours. An echocardiogram is not needed for the client with a thrombotic stroke. 6. Answer: 3. Thrombolytic therapy is use to dissolve emboli and reestablish cerebral perfusion. 7. Answer: 1. Thrombi form secondary to atrial fibrillation, therefore, an anticoagulant would be anticipated to prevent thrombi formation; and oral (warfarin [Coumadin]) at discharge verses intravenous. Beta blockers slow the heart rate and lower the blood pressure. Anti-hyperuricemic medication is given to clients with gout. Thrombolytic medication might have been given at initial presentation but would not be a drug prescribed at discharge. 8. Answer: 1. Africana Americans have twice the rate of CVAs as Caucasians; males are more likely to have strokes than females except in advanced years. Orientals have a lower risk, possibly due to their high omega -3 fatty acids. Pregnancy is a minimal risk factor for CVA. 9. Answer: 3. Uncontrolled hypertension is a risk factor for hemorrhagic stroke, which is a rupture blood vessel in the cranium. A bruit in the carotid artery would predispose a client to an embolic or ischemic stroke. High blood glucose levels could predispose a patient to ischemic stroke, but not hemorrhagic. Cancer is not a precursor to stroke. 10. Answer: 3. This action is inappropriate and would require intervention by the nurse because pulling on a flaccid shoulder joint could cause shoulder dislocation; as always use a lift sheet for the client and nurse safety. All the other actions are appropriate.

75

Introduction
A 60-item NCLEX-style examination all about Obstetrical Nursing. This exam has questions that will help you prepare for your boards or NCLEX.

Topics

Labor and Delivery Fetal Heart Rate Questions about oxytocin Disseminated Intravascular Coagulation (DIC) Leopolds Maneuver

Guidelines
Read each question carefully and choose the best answer. You are given one minute per question. Spend your time wisely! Answers and rationales are given below. Be sure to read them. If you need more clarifications, please direct them to the comments section.

Questions
1. A nurse is caring for a client in labor. The nurse determines that the client is beginning in the 2nd stage of labor when which of the following assessments is noted? 1. The client begins to expel clear vaginal fluid 2. The contractions are regular 3. The membranes have ruptured 4. The cervix is dilated completely 2. A nurse in the labor room is caring for a client in the active phases of labor. The nurse is assessing the fetal patterns and notes a late deceleration on the monitor strip. The most appropriate nursing action is to: 1. Place the mother in the supine position 2. Document the findings and continue to monitor the fetal patterns 3. Administer oxygen via face mask 4. Increase the rate of pitocin IV infusion 3. A nurse is performing an assessment of a client who is scheduled for a cesarean delivery. Which assessment finding would indicate a need to contact the physician? 1. Fetal heart rate of 180 beats per minute 2. White blood cell count of 12,000 3. Maternal pulse rate of 85 beats per minute 4. Hemoglobin of 11.0 g/dL 4. A client in labor is transported to the delivery room and is prepared for a cesarean delivery. The client is transferred to the delivery room table, and the nurse places the client in the: 1. Trendelenburgs position with the legs in stirrups 2. Semi-Fowler position with a pillow under the knees 3. Prone position with the legs separated and elevated 4. Supine position with a wedge under the right hip

76

5. A nurse is caring for a client in labor and prepares to auscultate the fetal heart rate by using a Doppler ultrasound device. The nurse most accurately determines that the fetal heart sounds are heard by: 1. Noting if the heart rate is greater than 140 BPM 2. Placing the diaphragm of the Doppler on the mother abdomen 3. Performing Leopolds maneuvers first to determine the location of the fetal heart 4. Palpating the maternal radial pulse while listening to the fetal heart rate 6. A nurse is caring for a client in labor who is receiving Pitocin by IV infusion to stimulate uterine contractions. Which assessment finding would indicate to the nurse that the infusion needs to be discontinued? 1. Three contractions occurring within a 10-minute period 2. A fetal heart rate of 90 beats per minute 3. Adequate resting tone of the uterus palpated between contractions 4. Increased urinary output 7. A nurse is beginning to care for a client in labor. The physician has prescribed an IV infusion of Pitocin. The nurse ensures that which of the following is implemented before initiating the infusion? 1. Placing the client on complete bed rest 2. Continuous electronic fetal monitoring 3. An IV infusion of antibiotics 4. Placing a code cart at the clients bedside 8. A nurse is monitoring a client in active labor and notes that the client is having contractions every 3 minutes that last 45 seconds. The nurse notes that the fetal heart rate between contractions is 100 BPM. Which of the following nursing actions is most appropriate? 1. Encourage the clients coach to continue to encourage breathing exercises 2. Encourage the client to continue pushing with each contraction 3. Continue monitoring the fetal heart rate 4. Notify the physician or nurse midwife 9. A nurse is caring for a client in labor and is monitoring the fetal heart rate patterns. The nurse notes the presence of episodic accelerations on the electronic fetal monitor tracing. Which of the following actions is most appropriate? 1. Document the findings and tell the mother that the monitor indicates fetal well-being 2. Take the mothers vital signs and tell the mother that bed rest is required to conserve oxygen. 3. Notify the physician or nurse midwife of the findings. 4. Reposition the mother and check the monitor for changes in the fetal tracing 10. A nurse is admitting a pregnant client to the labor room and attaches an external electronic fetal monitor to the clients abdomen. After attachment of the monitor, the initial nursing assessment is which of the following? 1. Identifying the types of accelerations 2. Assessing the baseline fetal heart rate 3. Determining the frequency of the contractions 4. Determining the intensity of the contractions 11. A nurse is reviewing the record of a client in the labor room and notes that the nurse midwife has documented that the fetus is at (-1) station. The nurse determines that the fetal presenting part is:
77

1. 1 cm above the ischial spine 2. 1 fingerbreadth below the symphysis pubis 3. 1 inch below the coccyx 4. 1 inch below the iliac crest 12. A pregnant client is admitted to the labor room. An assessment is performed, and the nurse notes that the clients hemoglobin and hematocrit levels are low, indicating anemia. The nurse determines that the client is at risk for which of the following? 1. A loud mouth 2. Low self-esteem 3. Hemorrhage 4. Postpartum infections 13. A nurse assists in the vaginal delivery of a newborn infant. After the delivery, the nurse observes the umbilical cord lengthen and a spurt of blood from the vagina. The nurse documents these observations as signs of: 1. Hematoma 2. Placenta previa 3. Uterine atony 4. Placental separation 14. A client arrives at a birthing center in active labor. Her membranes are still intact, and the nurse-midwife prepares to perform an amniotomy. A nurse who is assisting the nursemidwife explains to the client that after this procedure, she will most likely have: 1. Less pressure on her cervix 2. Increased efficiency of contractions 3. Decreased number of contractions 4. The need for increased maternal blood pressure monitoring 15. A nurse is monitoring a client in labor. The nurse suspects umbilical cord compression if which of the following is noted on the external monitor tracing during a contraction? 1. Early decelerations 2. Variable decelerations 3. Late decelerations 4. Short-term variability 16. A nurse explains the purpose of effleurage to a client in early labor. The nurse tells the client that effleurage is: 1. A form of biofeedback to enhance bearing down efforts during delivery 2. Light stroking of the abdomen to facilitate relaxation during labor and provide tactile stimulation to the fetus 3. The application of pressure to the sacrum to relieve a backache 4. Performed to stimulate uterine activity by contracting a specific muscle group while other parts of the body rest 17. A nurse is caring for a client in the second stage of labor. The client is experiencing uterine contractions every 2 minutes and cries out in pain with each contraction. The nurse recognizes this behavior as: 1. Exhaustion 2. Fear of losing control 3. Involuntary grunting 4. Valsalvas maneuver
78

18. A nurse is monitoring a client in labor who is receiving Pitocin and notes that the client is experiencing hypertonic uterine contractions. List in order of priority the actions that the nurse takes. 1. Stop of Pitocin infusion 2. Perform a vaginal examination 3. Reposition the client 4. Check the clients blood pressure and heart rate 5. Administer oxygen by face mask at 8 to 10 L/min 19. A nurse is assigned to care for a client with hypotonic uterine dysfunction and signs of a slowing labor. The nurse is reviewing the physicians orders and would expect to note which of the following prescribed treatments for this condition? 1. Medication that will provide sedation 2. Increased hydration 3. Oxytocin (Pitocin) infusion 4. Administration of a tocolytic medication 20. A nurse in the labor room is preparing to care for a client with hypertonic uterine dysfunction. The nurse is told that the client is experiencing uncoordinated contractions that are erratic in their frequency, duration, and intensity. The priority nursing intervention would be to: 1. Monitor the Pitocin infusion closely 2. Provide pain relief measures 3. Prepare the client for an amniotomy 4. Promote ambulation every 30 minutes 21. A nurse is developing a plan of care for a client experiencing dystocia and includes several nursing interventions in the plan of care. The nurse prioritizes the plan of care and selects which of the following nursing interventions as the highest priority? 1. Keeping the significant other informed of the progress of the labor 2. Providing comfort measures 3. Monitoring fetal heart rate 4. Changing the clients position frequently 22. A maternity nurse is preparing to care for a pregnant client in labor who will be delivering twins. The nurse monitors the fetal heart rates by placing the external fetal monitor: 1. Over the fetus that is most anterior to the mothers abdomen 2. Over the fetus that is most posterior to the mothers abdomen 3. So that each fetal heart rate is monitored separately 4. So that one fetus is monitored for a 15-minute period followed by a 15 minute fetal monitoring period for the second fetus 23. A nurse in the postpartum unit is caring for a client who has just delivered a newborn infant following a pregnancy with placenta previa. The nurse reviews the plan of care and prepares to monitor the client for which of the following risks associated with placenta previa? 1. Disseminated intravascular coagulation 2. Chronic hypertension 3. Infection 4. Hemorrhage
79

24. A nurse in the delivery room is assisting with the delivery of a newborn infant. After the delivery of the newborn, the nurse assists in delivering the placenta. Which observation would indicate that the placenta has separated from the uterine wall and is ready for delivery? 1. The umbilical cord shortens in length and changes in color 2. A soft and boggy uterus 3. Maternal complaints of severe uterine cramping 4. Changes in the shape of the uterus 25. A nurse in the labor room is performing a vaginal assessment on a pregnant client in labor. The nurse notes the presence of the umbilical cord protruding from the vagina. Which of the following would be the initial nursing action? 1. Place the client in Trendelenburgs position 2. Call the delivery room to notify the staff that the client will be transported immediately 3. Gently push the cord into the vagina 4. Find the closest telephone and stat page the physician 26. A maternity nurse is caring for a client with abruptio placenta and is monitoring the client for disseminated intravascular coagulopathy. Which assessment finding is least likely to be associated with disseminated intravascular coagulation? 1. Swelling of the calf in one leg 2. Prolonged clotting times 3. Decreased platelet count 4. Petechiae, oozing from injection sites, and hematuria 27. A nurse is assessing a pregnant client in the 2nd trimester of pregnancy who was admitted to the maternity unit with a suspected diagnosis of abruptio placentae. Which of the following assessment findings would the nurse expect to note if this condition is present? 1. Absence of abdominal pain 2. A soft abdomen 3. Uterine tenderness/pain 4. Painless, bright red vaginal bleeding 28. A maternity nurse is preparing for the admission of a client in the 3rd trimester of pregnancy that is experiencing vaginal bleeding and has a suspected diagnosis of placenta previa. The nurse reviews the physicians orders and would question which order? 1. Prepare the client for an ultrasound 2. Obtain equipment for external electronic fetal heart monitoring 3. Obtain equipment for a manual pelvic examination 4. Prepare to draw a Hgb and Hct blood sample 29. An ultrasound is performed on a client at term gestation that is experiencing moderate vaginal bleeding. The results of the ultrasound indicate that an abruptio placenta is present. Based on these findings, the nurse would prepare the client for: 1. Complete bed rest for the remainder of the pregnancy 2. Delivery of the fetus 3. Strict monitoring of intake and output 4. The need for weekly monitoring of coagulation studies until the time of delivery

80

30. A nurse in a labor room is assisting with the vaginal delivery of a newborn infant. The nurse would monitor the client closely for the risk of uterine rupture if which of the following occurred? 1. Hypotonic contractions 2. Forceps delivery 3. Schultz delivery 4. Weak bearing down efforts 31. A client is admitted to the birthing suite in early active labor. The priority nursing intervention on admission of this client would be: 1. Auscultating the fetal heart 2. Taking an obstetric history 3. Asking the client when she last ate 4. Ascertaining whether the membranes were ruptured 32. A client who is gravida 1, para 0 is admitted in labor. Her cervix is 100% effaced, and she is dilated to 3 cm. Her fetus is at +1 station. The nurse is aware that the fetus head is: 1. Not yet engaged 2. Entering the pelvic inlet 3. Below the ischial spines 4. Visible at the vaginal opening 33. After doing Leopolds maneuvers, the nurse determines that the fetus is in the ROP position. To best auscultate the fetal heart tones, the Doppler is placed: 1. Above the umbilicus at the midline 2. Above the umbilicus on the left side 3. Below the umbilicus on the right side 4. Below the umbilicus near the left groin 34. The physician asks the nurse the frequency of a laboring clients contractions. The nurse assesses the clients contractions by timing from the beginning of one contraction: 1. Until the time it is completely over 2. To the end of a second contraction 3. To the beginning of the next contraction 4. Until the time that the uterus becomes very firm 35. The nurse observes the clients amniotic fluid and decides that it appears normal, because it is: 1. Clear and dark amber in color 2. Milky, greenish yellow, containing shreds of mucus 3. Clear, almost colorless, and containing little white specks 4. Cloudy, greenish-yellow, and containing little white specks 36. At 38 weeks gestation, a client is having late decelerations. The fetal pulse oximeter shows 75% to 85%. The nurse should: 1. Discontinue the catheter, if the reading is not above 80% 2. Discontinue the catheter, if the reading does not go below 30% 3. Advance the catheter until the reading is above 90% and continue monitoring 4. Reposition the catheter, recheck the reading, and if it is 55%, keep monitoring 37. When examining the fetal monitor strip after rupture of the membranes in a laboring client, the nurse notes variable decelerations in the fetal heart rate. The nurse should:

81

1. Stop the oxytocin infusion 2. Change the clients position 3. Prepare for immediate delivery 4. Take the clients blood pressure 38. When monitoring the fetal heart rate of a client in labor, the nurse identifies an elevation of 15 beats above the baseline rate of 135 beats per minute lasting for 15 seconds. This should be documented as: 1. An acceleration 2. An early elevation 3. A sonographic motion 4. A tachycardic heart rate 39. A laboring client complains of low back pain. The nurse replies that this pain occurs most when the position of the fetus is: 1. Breech 2. Transverse 3. Occiput anterior 4. Occiput posterior 40. The breathing technique that the mother should be instructed to use as the fetus head is crowning is: 1. Blowing 2. Slow chest 3. Shallow 4. Accelerated-decelerated 41. During the period of induction of labor, a client should be observed carefully for signs of: 1. Severe pain 2. Uterine tetany 3. Hypoglycemia 4. Umbilical cord prolapse 42. A client arrives at the hospital in the second stage of labor. The fetus head is crowning, the client is bearing down, and the birth appears imminent. The nurse should: 1. Transfer her immediately by stretcher to the birthing unit 2. Tell her to breathe through her mouth and not to bear down 3. Instruct the client to pant during contractions and to breathe through her mouth 4. Support the perineum with the hand to prevent tearing and tell the client to pant 43. A laboring client is to have a pudendal block. The nurse plans to tell the client that once the block is working she: 1. Will not feel the episiotomy 2. May lose bladder sensation 3. May lose the ability to push 4. Will no longer feel contractions 44. Which of the following observations indicates fetal distress? 1. Fetal scalp pH of 7.14 2. Fetal heart rate of 144 beats/minute 3. Acceleration of fetal heart rate with contractions 4. Presence of long term variability
82

45. Which of the following fetal positions is most favorable for birth? 1. Vertex presentation 2. Transverse lie 3. Frank breech presentation 4. Posterior position of the fetal head 46. A laboring client has external electronic fetal monitoring in place. Which of the following assessment data can be determined by examining the fetal heart rate strip produced by the external electronic fetal monitor? 1. Gender of the fetus 2. Fetal position 3. Labor progress 4. Oxygenation 47. A laboring client is in the first stage of labor and has progressed from 4 to 7 cm in cervical dilation. In which of the following phases of the first stage does cervical dilation occur most rapidly? 1. Preparatory phase 2. Latent phase 3. Active phase 4. Transition phase 48. A multiparous client who has been in labor for 2 hours states that she feels the urge to move her bowels. How should the nurse respond? 1. Let the client get up to use the potty 2. Allow the client to use a bedpan 3. Perform a pelvic examination 4. Check the fetal heart rate 49. Labor is a series of events affected by the coordination of the five essential factors. One of these is the passenger (fetus). Which are the other four factors? 1. Contractions, passageway, placental position and function, pattern of care 2. Contractions, maternal response, placental position, psychological response 3. Passageway, contractions, placental position and function, psychological response 4. Passageway, placental position and function, paternal response, psychological response 50. Fetal presentation refers to which of the following descriptions? 1. Fetal body part that enters the maternal pelvis first 2. Relationship of the presenting part to the maternal pelvis 3. Relationship of the long axis of the fetus to the long axis of the mother 4. A classification according to the fetal part 51. A client is admitted to the L & D suite at 36 weeks gestation. She has a history of Csection and complains of severe abdominal pain that started less than 1 hour earlier. When the nurse palpates tetanic contractions, the client again complains of severe pain. After the client vomits, she states that the pain is better and then passes out. Which is the probable cause of her signs and symptoms? 1. Hysteria compounded by the flu 2. Placental abruption 3. Uterine rupture 4. Dysfunctional labor

83

52. Upon completion of a vaginal examination on a laboring woman, the nurse records: 50%, 6 cm, -1. Which of the following is a correct interpretation of the data? 1. Fetal presenting part is 1 cm above the ischial spines 2. Effacement is 4 cm from completion 3. Dilation is 50% completed 4. Fetus has achieved passage through the ischial spines 53. Which of the following findings meets the criteria of a reassuring FHR pattern? 1. FHR does not change as a result of fetal activity 2. Average baseline rate ranges between 100 140 BPM 3. Mild late deceleration patterns occur with some contractions 4. Variability averages between 6 10 BPM 54. Late deceleration patterns are noted when assessing the monitor tracing of a woman whose labor is being induced with an infusion of Pitocin. The woman is in a side-lying position, and her vital signs are stable and fall within a normal range. Contractions are intense, last 90 seconds, and occur every 1 1/2 to 2 minutes. The nurses immediate action would be to: 1. Change the womans position 2. Stop the Pitocin 3. Elevate the womans legs 4. Administer oxygen via a tight mask at 8 to 10 liters/minute 55. The nurse should realize that the most common and potentially harmful maternal complication of epidural anesthesia would be: 1. Severe postpartum headache 2. Limited perception of bladder fullness 3. Increase in respiratory rate 4. Hypotension 56. Perineal care is an important infection control measure. When evaluating a postpartum womans perineal care technique, the nurse would recognize the need for further instruction if the woman: 1. Uses soap and warm water to wash the vulva and perineum 2. Washes from symphysis pubis back to episiotomy 3. Changes her perineal pad every 2 3 hours 4. Uses the peribottle to rinse upward into her vagina 57. Which measure would be least effective in preventing postpartum hemorrhage? 1. Administer Methergine 0.2 mg every 6 hours for 4 doses as ordered 2. Encourage the woman to void every 2 hours 3. Massage the fundus every hour for the first 24 hours following birth 4. Teach the woman the importance of rest and nutrition to enhance healing 58. When making a visit to the home of a postpartum woman one week after birth, the nurse should recognize that the woman would characteristically: 1. Express a strong need to review events and her behavior during the process of labor and birth 2. Exhibit a reduced attention span, limiting readiness to learn 3. Vacillate between the desire to have her own nurturing needs met and the need to take charge of her own care and that of her newborn 4. Have reestablished her role as a spouse/partner

84

59. Four hours after a difficult labor and birth, a primiparous woman refuses to feed her baby, stating that she is too tired and just wants to sleep. The nurse should: 1. Tell the woman she can rest after she feeds her baby 2. Recognize this as a behavior of the taking-hold stage 3. Record the behavior as ineffective maternal-newborn attachment 4. Take the baby back to the nursery, reassuring the woman that her rest is a priority at this time 60. Parents can facilitate the adjustment of their other children to a new baby by: 1. Having the children choose or make a gift to give to the new baby upon its arrival home 2. Emphasizing activities that keep the new baby and other children together 3. Having the mother carry the new baby into the home so she can show the other children the new baby 4. Reducing stress on other children by limiting their involvement in the care of the new baby

Answers and Rationale


Gauge your performance by counter checking your answers to the answers below. Learn more about the question by reading the rationale. If you have any disputes or questions, please direct them to the comments section. 1. Answer: 4. The cervix is dilated completely. The second stage of labor begins when the cervix is dilated completely and ends with the birth of the neonate. 2. Answer: 3. Administer oxygen via face mask. Late decelerations are due to uteroplacental insufficiency as the result of decreased blood flow and oxygen to the fetus during the uterine contractions. This causes hypoxemia; therefore oxygen is necessary. The supine position is avoided because it decreases uterine blood flow to the fetus. The client should be turned to her side to displace pressure of the gravid uterus on the inferior vena cava. An intravenous pitocin infusion is discontinued when a late deceleration is noted. 3. Answer: 1. Fetal heart rate of 180 beats per minute. A normal fetal heart rate is 120-160 beats per minute. A count of 180 beats per minute could indicate fetal distress and would warrant physician notification. By full term, a normal maternal hemoglobin range is 11-13 g/dL as a result of the hemodilution caused by an increase in plasma volume during pregnancy. 4. Answer: 4. Supine position with a wedge under the right hip. Vena cava and descending aorta compression by the pregnant uterus impedes blood return from the lower trunk and extremities. This leads to decreasing cardiac return, cardiac output, and blood flow to the uterus and the fetus. The best position to prevent this would be side-lying with the uterus displaced off of abdominal vessels. Positioning for abdominal surgery necessitates a supine position; however, a wedge placed under the right hip provides displacement of the uterus. 5. Answer: 4. Palpating the maternal radial pulse while listening to the fetal heart rate. The nurse simultaneously should palpate the maternal radial or carotid pulse and auscultate the fetal heart rate to differentiate the two. If the fetal and maternal heart rates are similar, the nurse may mistake the maternal heart rate for the fetal heart rate. Leopolds maneuvers may help the examiner locate the position of the fetus but will not ensure a distinction between the two rates. 6. Answer: 2. A fetal heart rate of 90 beats per minute. A normal fetal heart rate is 120-160 BPM. Bradycardia or late or variable decelerations indicate fetal distress and the need to discontinue to pitocin. The goal of labor augmentation is to achieve three good-quality contractions in a 10-minute period. 7. Answer: 2. Continuous electronic fetal monitoring. Continuous electronic fetal monitoring should be implemented during an IV infusion of Pitocin.

85

8. Answer: 4. Notify the physician or nurse midwife. A normal fetal heart rate is 120-160 beats per minute. Fetal bradycardia between contractions may indicate the need for immediate medical management, and the physician or nurse midwife needs to be notified. 9. Answer: 1. Document the findings and tell the mother that the monitor indicates fetal well-being. Accelerations are transient increases in the fetal heart rate that often accompany contractions or are caused by fetal movement. Episodic accelerations are thought to be a sign of fetal-well being and adequate oxygen reserve. 10. Answer: 2. Assessing the baseline fetal heart rate. Assessing the baseline fetal heart rate is important so that abnormal variations of the baseline rate will be identified if they occur. Options 1 and 3 are important to assess, but not as the first priority. 11. Answer: 1. 1 cm above the ischial spine. Station is the relationship of the presenting part to an imaginary line drawn between the ischial spines, is measured in centimeters, and is noted as a negative number above the line and a positive number below the line. At -1 station, the fetal presenting part is 1 cm above the ischial spines. 12. Answer: 4. Postpartum infections. Anemic women have a greater likelihood of cardiac decompensation during labor, postpartum infection, and poor wound healing. Anemia does not specifically present a risk for hemorrhage. Having a loud mouth is only related to the person typing up this test. 13. Answer: 4. Placental separation. As the placenta separates, it settles downward into the lower uterine segment. The umbilical cord lengthens, and a sudden trickle or spurt of blood appears. 14. Answer: 2. Increased efficiency of contractions. Amniotomy can be used to induce labor when the condition of the cervix is favorable (ripe) or to augment labor if the process begins to slow. Rupturing of membranes allows the fetal head to contact the cervix more directly and may increase the efficiency of contractions. 15. Answer: 2. Variable decelerations. Variable decelerations occur if the umbilical cord becomes compressed, thus reducing blood flow between the placenta and the fetus. Early decelerations result from pressure on the fetal head during a contraction. Late decelerations are an ominous pattern in labor because it suggests uteroplacental insufficiency during a contraction. Short-term variability refers to the beat-to-beat range in the fetal heart rate. 16. Answer: 2. Light stroking of the abdomen to facilitate relaxation during labor and provide tactile stimulation to the fetus. Effleurage is a specific type of cutaneous stimulation involving light stroking of the abdomen and is used before transition to promote relaxation and relieve mild to moderate pain. Effleurage provides tactile stimulation to the fetus. 17. Answer: 2. Fear of losing control. Pains, helplessness, panicking, and fear of losing control are possible behaviors in the 2nd stage of labor. 18. Answer: 1, 4, 2. 5, 3. If uterine hypertonicity occurs, the nurse immediately would intervene to reduce uterine activity and increase fetal oxygenation. The nurse would stop the Pitocin infusion and increase the rate of the nonadditive solution, check maternal BP for hyper or hypotension, position the woman in a side-lying position, and administer oxygen by snug face mask at 8-10 L/min. The nurse then would attempt to determine the cause of the uterine hypertonicity and perform a vaginal exam to check for prolapsed cord. 19. Answer: 3. Oxytocin (Pitocin) infusion. Therapeutic management for hypotonic uterine dysfunction includes oxytocin augmentation and amniotomy to stimulate a labor that slows. 20. Answer: 2. Provide pain relief measures. Management of hypertonic labor depends on the cause. Relief of pain is the primary intervention to promote a normal labor pattern.
86

21. Answer: 3. Monitoring fetal heart rate. The priority is to monitor the fetal heart rate. 22. Answer: 3. So that each fetal heart rate is monitored separately. In a client with a multifetal pregnancy, each fetal heart rate is monitored separately. 23. Answer: 4. Hemorrhage. Because the placenta is implanted in the lower uterine segment, which does not contain the same intertwining musculature as the fundus of the uterus, this site is more prone to bleeding. 24. Answer: 4. Changes in the shape of the uterus. Signs of placental separation include lengthening of the umbilical cord, a sudden gush of dark blood from the introitus (vagina), a firmly contracted uterus, and the uterus changing from a discoid (like a disk) to a globular (like a globe) shape. The client may experience vaginal fullness, but not severe uterine cramping. 25. Answer: 1. Place the client in Trendelenburgs position. When cord prolapse occurs, prompt actions are taken to relieve cord compression and increase fetal oxygenation. The mother should be positioned with the hips higher than the head to shift the fetal presenting part toward the diaphragm. The nurse should push the call light to summon help, and other staff members should call the physician and notify the delivery room. No attempt should be made to replace the cord. The examiner, however, may place a gloved hand into the vagina and hold the presenting part off of the umbilical cord. Oxygen at 8 to 10 L/min by face mask is delivered to the mother to increase fetal oxygenation. 26. Answer: 1. Swelling of the calf in one leg. DIC is a state of diffuse clotting in which clotting factors are consumed, leading to widespread bleeding. Platelets are decreased because they are consumed by the process; coagulation studies show no clot formation (and are thus normal to prolonged); and fibrin plugs may clog the microvasculature diffusely, rather than in an isolated area. The presence of petechiae, oozing from injection sites, and hematuria are signs associated with DIC. Swelling and pain in the calf of one leg are more likely to be associated with thrombophlebitis. 27. 3. Uterine tenderness/pain. In abruptio placentae, acute abdominal pain is present. Uterine tenderness and pain accompanies placental abruption, especially with a central abruption and trapped blood behind the placenta. The abdomen will feel hard and boardlike on palpation as the blood penetrates the myometrium and causes uterine irritability. Observation of the fetal monitoring often reveals increased uterine resting tone, caused by failure of the uterus to relax in attempt to constrict blood vessels and control bleeding. 28. Answer: 3. Obtain equipment for a manual pelvic examination. Manual pelvic examinations are contraindicated when vaginal bleeding is apparent in the 3rd trimester until a diagnosis is made and placental previa is ruled out. Digital examination of the cervix can lead to maternal and fetal hemorrhage. A diagnosis of placenta previa is made by ultrasound. The H/H levels are monitored, and external electronic fetal heart rate monitoring is initiated. External fetal monitoring is crucial in evaluating the fetus that is at risk for severe hypoxia. 29. Answer: 2. Delivery of the fetus. The goal of management in abruptio placentae is to control the hemorrhage and deliver the fetus as soon as possible. Delivery is the treatment of choice if the fetus is at term gestation or if the bleeding is moderate to severe and the mother or fetus is in jeopardy. 30. Answer: 2. Forceps delivery. Excessive fundal pressure, forceps delivery, violent bearing down efforts, tumultuous labor, and shoulder dystocia can place a woman at risk for traumatic uterine rupture. Hypotonic contractions and weak bearing down efforts do not alone add to the risk of rupture because they do not add to the stress on the uterine wall.

87

31. Answer: 1. Auscultating the fetal heart. Determining the fetal well-being supersedes all other measures. If the FHR is absent or persistently decelerating, immediate intervention is required. 32. Answer: 3. Below the ischial spines. A station of +1 indicates that the fetal head is 1 cm below the ischial spines. 33. Answer: 3. Below the umbilicus on the right side. Fetal heart tones are best auscultated through the fetal back; because the position is ROP (right occiput presenting), the back would be below the umbilicus and on the right side. 34. Answer: 3. To the beginning of the next contraction. This is the way to determine the frequency of the contractions 35. Answer: 3. Clear, almost colorless, and containing little white specks. By 36 weeks gestation, normal amniotic fluid is colorless with small particles of vernix caseosa present. 36. Answer: 4. Reposition the catheter, recheck the reading, and if it is 55%, keep monitoring. Adjusting the catheter would be indicated. Normal fetal pulse oximetry should be between 30% and 70%. 75% to 85% would indicate maternal readings. 37. Answer: 2. Change the clients position. Variable decelerations usually are seen as a result of cord compression; a change of position will relieve pressure on the cord. 38. Answer: 1. An acceleration. An acceleration is an abrupt elevation above the baseline of 15 beats per minute for 15 seconds; if the acceleration persists for more than 10 minutes it is considered a change in baseline rate. A tachycardic FHR is above 160 beats per minute. 39. Answer: 4. Occiput posterior. A persistent occiput-posterior position causes intense back pain because of fetal compression of the sacral nerves. Occiput anterior is the most common fetal position and does not cause back pain. 40. Answer: 1. Blowing. Blowing forcefully through the mouth controls the strong urge to push and allows for a more controlled birth of the head. 41. Answer: 2. Uterine tetany. Uterine tetany could result from the use of oxytocin to induce labor. Because oxytocin promotes powerful uterine contractions, uterine tetany may occur. The oxytocin infusion must be stopped to prevent uterine rupture and fetal compromise. 42. Answer: 4. Support the perineum with the hand to prevent tearing and tell the client to pant. Gentle pressure is applied to the babys head as it emerges so it is not born too rapidly. The head is never held back, and it should be supported as it emerges so there will be no vaginal lacerations. It is impossible to push and pant at the same time. 43. Answer: 1. May lose the ability to push. A pudendal block provides anesthesia to the perineum. 44. Answer: 1. Fetal scalp pH of 7.14. A fetal scalp pH below 7.25 indicates acidosis and fetal hypoxia. 45. Answer: 1. Vertex presentation. Vertex presentation (flexion of the fetal head) is the optimal presentation for passage through the birth canal. Transverse lie is an unacceptable fetal position for vaginal birth and requires a C-section. Frank breech presentation, in which the buttocks present first, can be a difficult vaginal delivery. Posterior positioning of the fetal head can make it difficult for the fetal head to pass under the maternal symphysis pubis. 46. Answer: 4. Oxygenation. Oxygenation of the fetus may be indirectly assessed through fetal monitoring by closely examining the fetal heart rate strip. Accelerations in the fetal heart rate strip indicate good oxygenation, while decelerations in the fetal heart rate sometimes indicate poor fetal oxygenation.

88

47. Answer: 3. Active phase. Cervical dilation occurs more rapidly during the active phase than any of the previous phases. The active phase is characterized by cervical dilation that progresses from 4 to 7 cm. The preparatory, or latent, phase begins with the onset of regular uterine contractions and ends when rapid cervical dilation begins. Transition is defined as cervical dilation beginning at 8 cm and lasting until 10 cm or complete dilation. 48. Answer: 3. Perform a pelvic examination. A complaint of rectal pressure usually indicates a low presenting fetal part, signaling imminent delivery. The nurse should perform a pelvic examination to assess the dilation of the cervix and station of the presenting fetal part. 49. Answer: 3. Passageway, contractions, placental position and function, psychological response. The five essential factors (5 Ps) are passenger (fetus), passageway (pelvis), powers (contractions), placental position and function, and psyche (psychological response of the mother). 50. Answer: 1. Fetal body part that enters the maternal pelvis first. Presentation is the fetal body part that enters the pelvis first; its classified by the presenting part; the three main presentations are cephalic/occipital, breech, and shoulder. The relationship of the presenting fetal part to the maternal pelvis refers to fetal position. The relationship of the long axis to the fetus to the long axis of the mother refers to fetal lie; the three possible lies are longitudinal, transverse, and oblique. 51. Answer: 3. Uterine rupture. Uterine rupture is a medical emergency that may occur before or during labor. Signs and symptoms typically include abdominal pain that may ease after uterine rupture, vomiting, vaginal bleeding, hypovolemic shock, and fetal distress. With placental abruption, the client typically complains of vaginal bleeding and constant abdominal pain. 52. Answer: 1. Fetal presenting part is 1 cm above the ischial spines. Station of 1 indicates that the fetal presenting part is above the ischial spines and has not yet passed through the pelvic inlet. A station of zero would indicate that the presenting part has passed through the inlet and is at the level of the ischial spines or is engaged. Passage through the ischial spines with internal rotation would be indicated by a plus station, such as + 1. Progress of effacement is referred to by percentages with 100% indicating full effacement and dilation by centimeters (cm) with 10 cm indicating full dilation. 53. Answer: 4. Variability averages between 6 10 BPM. Variability indicates a well oxygenated fetus with a functioning autonomic nervous system. FHR should accelerate with fetal movement. Baseline range for the FHR is 120 to 160 beats per minute. Late deceleration patterns are never reassuring, though early and mild variable decelerations are expected, reassuring findings. 54. Answer: 2. Stop the Pitocin. Late deceleration patterns noted are most likely related to alteration in uteroplacental perfusion associated with the strong contractions described. The immediate action would be to stop the Pitocin infusion since Pitocin is an oxytocic which stimulates the uterus to contract. The woman is already in an appropriate position for uteroplacental perfusion. Elevation of her legs would be appropriate if hypotension were present. Oxygen is appropriate but not the immediate action. 55. Answer: 4. Hypotension. Epidural anesthesia can lead to vasodilation and a drop in blood pressure that could interfere with adequate placental perfusion. The woman must be well hydrated before and during epidural anesthesia to prevent this problem and maintain an adequate blood pressure. Headache is not a side effect since the spinal fluid is not disturbed by this anesthetic as it would be with a low spinal (saddle block) anesthesia; 2 is an effect of epidural

89

anesthesia but is not the most harmful. Respiratory depression is a potentially serious complication. 56. Answer: 4. Uses the peribottle to rinse upward into her vagina. Responses 1, 2, and 3 are all appropriate measures. The peri bottle should be used in a backward direction over the perineum. The flow should never be directed upward into the vagina since debris would be forced upward into the uterus through the still-open cervix. 57. Answer: 3. Massage the fundus every hour for the first 24 hours following birth. The fundus should be massaged only when boggy or soft. Massaging a firm fundus could cause it to relax. Responses 1, 2, and 4 are all effective measures to enhance and maintain contraction of the uterus and to facilitate healing. 58. Answer: 3. Vacillate between the desire to have her own nurturing needs met and the need to take charge of her own care and that of her newborn. One week after birth the woman should exhibit behaviors characteristic of the taking-hold stage as described in response 3. This stage lasts for as long as 4 to 5 weeks after birth. Responses 1 and 2 are characteristic of the taking-in stage, which lasts for the first few days after birth. Response 4 reflects the lettinggo stage, which indicates that psychosocial recovery is complete. 59. Answer: 4. Take the baby back to the nursery, reassuring the woman that her rest is a priority at this time. Response 1 does not take into consideration the need for the new mother to be nurtured and have her needs met during the taking-in stage. The behavior described is typical of this stage and not a reflection of ineffective attachment unless the behavior persists. Mothers need to reestablish their own well-being in order to effectively care for their baby. 60. Answer: 1. Having the children choose or make a gift to give to the new baby upon its arrival home. Special time should be set aside just for the other children without interruption from the newborn. Someone other than the mother should carry the baby into the home so she can give full attention to greeting her other children. Children should be actively involved in the care of the baby according to their ability without overwhelming them.

Introduction
A 55-item examination, NCLEX style, that challenges your knowledge about Postpartum Care.

Topics

Postpartum Care Lochias!

Guidelines
Read each question carefully and choose the best answer. You are given one minute per question. Spend your time wisely! Answers and rationales are given below. Be sure to read them. If you need more clarifications, please direct them to the comments section.

Questions
1. A postpartum nurse is preparing to care for a woman who has just delivered a healthy newborn infant. In the immediate postpartum period the nurse plans to take the womans vital signs: 1. Every 30 minutes during the first hour and then every hour for the next two hours. 2. Every 15 minutes during the first hour and then every 30 minutes for the next two hours.

90

3. Every hour for the first 2 hours and then every 4 hours 4. Every 5 minutes for the first 30 minutes and then every hour for the next 4 hours. 2. A postpartum nurse is taking the vital signs of a woman who delivered a healthy newborn infant 4 hours ago. The nurse notes that the mothers temperature is 100.2*F. Which of the following actions would be most appropriate? 1. Retake the temperature in 15 minutes 2. Notify the physician 3. Document the findings 4. Increase hydration by encouraging oral fluids 3. The nurse is assessing a client who is 6 hours PP after delivering a full-term healthy infant. The client complains to the nurse of feelings of faintness and dizziness. Which of the following nursing actions would be most appropriate? 1. Obtain hemoglobin and hematocrit levels 2. Instruct the mother to request help when getting out of bed 3. Elevate the mothers legs 4. Inform the nursery room nurse to avoid bringing the newborn infant to the mother until the feelings of lightheadedness and dizziness have subsided. 4. A nurse is preparing to perform a fundal assessment on a postpartum client. The initial nursing action in performing this assessment is which of the following? 1. Ask the client to turn on her side 2. Ask the client to lie flat on her back with the knees and legs flat and straight. 3. Ask the mother to urinate and empty her bladder 4. Massage the fundus gently before determining the level of the fundus. 5. The nurse is assessing the lochia on a 1 day PP patient. The nurse notes that the lochia is red and has a foul-smelling odor. The nurse determines that this assessment finding is: 1. Normal 2. Indicates the presence of infection 3. Indicates the need for increasing oral fluids 4. Indicates the need for increasing ambulation 6. When performing a PP assessment on a client, the nurse notes the presence of clots in the lochia. The nurse examines the clots and notes that they are larger than 1 cm. Which of the following nursing actions is most appropriate? 1. Document the findings 2. Notify the physician 3. Reassess the client in 2 hours 4. Encourage increased intake of fluids. 7. A nurse in a PP unit is instructing a mother regarding lochia and the amount of expected lochia drainage. The nurse instructs the mother that the normal amount of lochia may vary but should never exceed the need for: 1. One peripad per day 2. Two peripads per day 3. Three peripads per day 4. Eight peripads per day 8. A PP nurse is providing instructions to a woman after delivery of a healthy newborn infant. The nurse instructs the mother that she should expect normal bowel elimination to return: 1. One the day of the delivery 2. 3 days PP 3. 7 days PP 4. within 2 weeks PP 9. Select all of the physiological maternal changes that occur during the PP period.

91

1. Cervical involution ceases immediately 2. Vaginal distention decreases slowly 3. Fundus begins to descend into the pelvis after 24 hours 4. Cardiac output decreases with resultant tachycardia in the first 24 hours 5. Digestive processes slow immediately. 10. A nurse is caring for a PP woman who has received epidural anesthesia and is monitoring the woman for the presence of a vulva hematoma. Which of the following assessment findings would best indicate the presence of a hematoma? 1. Complaints of a tearing sensation 2. Complaints of intense pain 3. Changes in vital signs 4. Signs of heavy bruising 11. A nurse is developing a plan of care for a PP woman with a small vulvar hematoma. The nurse includes which specific intervention in the plan during the first 12 hours following the delivery of this client? 1. Assess vital signs every 4 hours 2. Inform health care provider of assessment findings 3. Measure fundal height every 4 hours 4. Prepare an ice pack for application to the area. 12. A new mother received epidural anesthesia during labor and had a forceps delivery after pushing 2 hours. At 6 hours PP, her systolic blood pressure has dropped 20 points, her diastolic BP has dropped 10 points, and her pulse is 120 beats per minute. The client is anxious and restless. On further assessment, a vulvar hematoma is verified. After notifying the health care provider, the nurse immediately plans to: 1. Monitor fundal height 2. Apply perineal pressure 3. Prepare the client for surgery. 4. Reassure the client 13. A nurse is monitoring a new mother in the PP period for signs of hemorrhage. Which of the following signs, if noted in the mother, would be an early sign of excessive blood loss? 1. A temperature of 100.4*F 2. An increase in the pulse from 88 to 102 BPM 3. An increase in the respiratory rate from 18 to 22 breaths per minute 4. A blood pressure change from 130/88 to 124/80 mm Hg 14. A nurse is preparing to assess the uterine fundus of a client in the immediate postpartum period. When the nurse locates the fundus, she notes that the uterus feels soft and boggy. Which of the following nursing interventions would be most appropriate initially? 1. Massage the fundus until it is firm 2. Elevate the mothers legs 3. Push on the uterus to assist in expressing clots 4. Encourage the mother to void 15. A PP nurse is assessing a mother who delivered a healthy newborn infant by C-section. The nurse is assessing for signs and symptoms of superficial venous thrombosis. Which of the following signs or symptoms would the nurse note if superficial venous thrombosis were present? 1. Paleness of the calf area 2. Enlarged, hardened veins 3. Coolness of the calf area 4. Palpable dorsalis pedis pulses 16. A nurse is providing instructions to a mother who has been diagnosed with mastitis. Which of the following statements if made by the mother indicates a need for further teaching?

92

1. I need to take antibiotics, and I should begin to feel better in 24 -48 hours. 2. I can use analgesics to assist in alleviating some of the discomfort. 3. I need to wear a supportive bra to relieve the discomfort. 4. I need to stop breastfeeding until this condition resolves. 17. A PP client is being treated for DVT. The nurse understands that the clients response to treatment will be evaluated by regularly assessing the client for: 1. Dysuria, ecchymosis, and vertigo 2. Epistaxis, hematuria, and dysuria 3. Hematuria, ecchymosis, and epistaxis 4. Hematuria, ecchymosis, and vertigo 18. A nurse performs an assessment on a client who is 4 hours PP. The nurse notes that the client has cool, clammy skin and is restless and excessively thirsty. The nurse prepares immediately to: 1. Assess for hypovolemia and notify the health care provider 2. Begin hourly pad counts and reassure the client 3. Begin fundal massage and start oxygen by mask 4. Elevate the head of the bed and assess vital signs 19. A nurse is assessing a client in the 4th stage if labor and notes that the fundus is firm but that bleeding is excessive. The initial nursing action would be which of the following? 1. Massage the fundus 2. Place the mother in the Trendelenburgs position 3. Notify the physician 4. Record the findings 20. A nurse is caring for a PP client with a diagnosis of DVT who is receiving a continuous intravenous infusion of heparin sodium. Which of the following laboratory results will the nurse specifically review to determine if an effective and appropriate dose of the heparin is being delivered? 1. Prothrombin time 2. International normalized ratio 3. Activated partial thromboplastin time 4. Platelet count 21. A nurse is preparing a list of self-care instructions for a PP client who was diagnosed with mastitis. Select all instructions that would be included on the list. 1. Take the prescribed antibiotics until the soreness subsides. 2. Wear supportive bra 3. Avoid decompression of the breasts by breastfeeding or breast pump 4. Rest during the acute phase 5. Continue to breastfeed if the breasts are not too sore. 22. Methergine or pitocin is prescribed for a woman to treat PP hemorrhage. Before administration of these medications, the priority nursing assessment is to check the: 1. Amount of lochia 2. Blood pressure 3. Deep tendon reflexes 4. Uterine tone 23. Methergine or pitocin are prescribed for a client with PP hemorrhage. Before administering the medication(s), the nurse contacts the health provider who prescribed the medication(s) in which of the following conditions is documented in the clients medical history? 1. Peripheral vascular disease 2. Hypothyroidism

93

3. Hypotension 4. Type 1 diabetes 24. Which of the following factors might result in a decreased supply of breastmilk in a PP mother? 1. Supplemental feedings with formula 2. Maternal diet high in vitamin C 3. An alcoholic drink 4. Frequent feedings 25. Which of the following interventions would be helpful to a breastfeeding mother who is experiencing engorged breasts? 1. Applying ice 2. Applying a breast binder 3. Teaching how to express her breasts in a warm shower 4. Administering bromocriptine (Parlodel) 26. On completing a fundal assessment, the nurse notes the fundus is situated on the clients left abdomen. Which of the following actions is appropriate? 1. Ask the client to empty her bladder 2. Straight catheterize the client immediately 3. Call the clients health provider for direction 4. Straight catheterize the client for half of her uterine volume 27. The nurse is about the give a Type 2 diabetic her insulin before breakfast on her first day postpartum. Which of the following answers best describes insulin requirements immediately postpartum? 1. Lower than during her pregnancy 2. Higher than during her pregnancy 3. Lower than before she became pregnant 4. Higher than before she became pregnant 28. Which of the following findings would be expected when assessing the postpartum client? 1. Fundus 1 cm above the umbilicus 1 hour postpartum 2. Fundus 1 cm above the umbilicus on postpartum day 3 3. Fundus palpable in the abdomen at 2 weeks postpartum 4. Fundus slightly to the right; 2 cm above umbilicus on postpartum day 2 29. A client is complaining of painful contractions, or afterpains, on postpartum day 2. Which of the following conditions could increase the severity of afterpains? 1. Bottle-feeding 2. Diabetes 3. Multiple gestation 4. Primiparity 30. On which of the postpartum days can the client expect lochia serosa? 1. Days 3 and 4 PP 2. Days 3 to 10 PP 3. Days 10-14 PP 4. Days 14 to 42 PP 31. Which of the following behaviors characterizes the PP mother in the taking inphase? 1. Passive and dependant 2. Striving for independence and autonomy 3. Curious and interested in care of the baby 4. Exhibiting maximum readiness for new learning

94

32. Which of the following complications may be indicated by continuous seepage of blood from the vagina of a PP client, when palpation of the uterus reveals a firm uterus 1 cm below the umbilicus? 1. Retained placental fragments 2. Urinary tract infection 3. Cervical laceration 4. Uterine atony 33. What type of milk is present in the breasts 7 to 10 days PP? 1. Colostrum 2. Hind milk 3. Mature milk 4. Transitional milk 34. Which of the following complications is most likely responsible for a delayed postpartum hemorrhage? 1. Cervical laceration 2. Clotting deficiency 3. Perineal laceration 4. Uterine subinvolution 35. Before giving a PP client the rubella vaccine, which of the following facts should the nurse include in client teaching? 1. The vaccine is safe in clients with egg allergies 2. Breast-feeding isnt compatible with the vaccine 3. Transient arthralgia and rash are common adverse effects 4. The client should avoid getting pregnant for 3 months after the vaccine because the vaccine has teratogenic effects 36. Which of the following changes best described the insulin needs of a client with type 1 diabetes who has just delivered an infant vaginally without complications? 1. Increase 2. Decrease 3. Remain the same as before pregnancy 4. Remain the same as during pregnancy 37. Which of the following responses is most appropriate for a mother with diabetes who wants to breastfeed her infant but is concerned about the effects of breastfeeding on her health? 1. Mothers with diabetes who breastfeed have a hard time controlling their insulin needs 2. Mothers with diabetes shouldnt breastfeed because of potential complications 3. Mothers with diabetes shouldnt breastfeed; insulin requirements are doubled. 4. Mothers with diabetes may breastfeed; insulin requirements may decrease from breastfeeding. 38. On the first PP night, a client requests that her baby be sent back to the nursery so she can get some sleep. The client is most likely in which of the following phases? 1. Depression phase 2. Letting-go phase 3. Taking-hold phase 4. Taking-in phase 39. Which of the following physiological responses is considered normal in the early postpartum period? 1. Urinary urgency and dysuria 2. Rapid diuresis 3. Decrease in blood pressure 4. Increase motility of the GI system 40. During the 3rd PP day, which of the following observations about the client would the nurse be most likely to make? 1. The client appears interested in learning about neonatal care 2. The client talks a lot about her birth experience

95

3. The client sleeps whenever the neonate isnt present 4. The client requests help in choosing a name for the neonate. 41. Which of the following circumstances is most likely to cause uterine atony and lead to PP hemorrhage? 1. Hypertension 2. Cervical and vaginal tears 3. Urine retention 4. Endometritis 42. Which type of lochia should the nurse expect to find in a client 2 days PP? 1. Foul-smelling 2. Lochia serosa 3. Lochia alba 4. Lochia rubra 43. After expulsion of the placenta in a client who has six living children, an infusion of lactated ringers solution with 10 units of pitocin is ordered. The nurse understands that this is indicated for this client because: 1. She had a precipitate birth 2. This was an extramural birth 3. Retained placental fragments must be expelled 4. Multigravidas are at increased risk for uterine atony. 44. As part of the postpartum assessment, the nurse examines the breasts of a primiparous breastfeeding woman who is one day postpartum. An expected finding would be: 1. Soft, non-tender; colostrum is present 2. Leakage of milk at let down 3. Swollen, warm, and tender upon palpation 4. A few blisters and a bruise on each areola 45. Following the birth of her baby, a woman expresses concern about the weight she gained during pregnancy and how quickly she can lose it now that the baby is born. The nurse, in describing the expected pattern of weight loss, should begin by telling this woman that: 1. Return to pre pregnant weight is usually achieved by the end of the postpartum period 2. Fluid loss from diuresis, diaphoresis, and bleeding accounts for about a 3 pound weight loss 3. The expected weight loss immediately after birth averages about 11 to 13 pounds 4. Lactation will inhibit weight loss since caloric intake must increase to support milk production 46. Which of the following findings would be a source of concern if noted during the assessment of a woman who is 12 hours postpartum? 1. Postural hypotension 2. Temperature of 100.4F 3. Bradycardia pulse rate of 55 BPM 4. Pain in left calf with dorsiflexion of left foot 47. The nurse examines a woman one hour after birth. The womans fundus is boggy, midline, and 1 cm below the umbilicus. Her lochial flow is profuse, with two plum-sized clots. The nurses initial action would be to: 1. Place her on a bedpan to empty her bladder 2. Massage her fundus 3. Call the physician 4. Administer Methergine 0.2 mg IM which has been ordered prn 48. When performing a postpartum check, the nurse should: 1. Assist the woman into a lateral position with upper leg flexed forward to facilitate the examination of her perineum 2. Assist the woman into a supine position with her arms above her head and her legs extended for the examination of her abdomen

96

3. Instruct the woman to avoid urinating just before the examination since a full bladder will facilitate fundal palpation 4. Wash hands and put on sterile gloves before beginning the check 49. Perineal care is an important infection control measure. When evaluating a postpartum womans perineal care technique, the nurse would recognize the need for further instruction if the woman: 1. Uses soap and warm water to wash the vulva and perineum 2. Washes from symphysis pubis back to episiotomy 3. Changes her perineal pad every 2 3 hours 4. Uses the peribottle to rinse upward into her vagina 50. Which measure would be least effective in preventing postpartum hemorrhage? 1. Administer Methergine 0.2 mg every 6 hours for 4 doses as ordered 2. Encourage the woman to void every 2 hours 3. Massage the fundus every hour for the first 24 hours following birth 4. Teach the woman the importance of rest and nutrition to enhance healing 51. When making a visit to the home of a postpartum woman one week after birth, the nurse should recognize that the woman would characteristically: 1. Express a strong need to review events and her behavior during the process of labor and birth 2. Exhibit a reduced attention span, limiting readiness to learn 3. Vacillate between the desire to have her own nurturing needs met and the need to take charge of her own care and that of her newborn 4. Have reestablished her role as a spouse/partner 52. Four hours after a difficult labor and birth, a primiparous woman refuses to feed her baby, stating that she is too tired and just wants to sleep. The nurse should: 1. Tell the woman she can rest after she feeds her baby 2. Recognize this as a behavior of the taking-hold stage 3. Record the behavior as ineffective maternal-newborn attachment 4. Take the baby back to the nursery, reassuring the woman that her rest is a priority at this time 53. Parents can facilitate the adjustment of their other children to a new baby by: 1. Having the children choose or make a gift to give to the new baby upon its arrival home 2. Emphasizing activities that keep the new baby and other children together 3. Having the mother carry the new baby into the home so she can show the other children the new baby 4. Reducing stress on other children by limiting their involvement in the care of the new baby 54. A primiparous woman is in the taking-in stage of psychosocial recovery and adjustment following birth. The nurse, recognizing the needs of women during this stage, should: 1. Foster an active role in the babys care 2. Provide time for the mother to reflect on the events of and her behavior during childbirth 3. Recognize the womans limited attention span by giving her written materials to read when she gets home rather than doing a teaching session now 4. Promote maternal independence by encouraging her to meet her own hygiene and comfort needs 55. All of the following are important in the immediate care of the premature neonate. Which nursing activity should have the greatest priority? 1. Instillation of antibiotic in the eyes 2. Identification by bracelet and foot prints 3. Placement in a warm environment 4. Neurological assessment to determine gestational age

Answers and Rationale


Gauge your performance by counter checking your answers to the answers below. Learn more about the question by reading the rationale. If you have any disputes or questions, please direct them to the comments section.

97

1. Answer: 2. Every 15 minutes during the first hour and then every 30 minutes for the next two hours. 2. Answer: 4. Increase hydration by encouraging oral fluids. The mothers temperature may be taken every 4 hours while she is awake. Temperatures up to 100.4 (38 C) in the first 24 hours after birth are often related to the dehydrating effects of labor. The most appropriate action is to increase hydration by encouraging oral fluids, which should bring the temperature to a normal reading. Although the nurse would document the findings, the most appropriate action would be to increase the hydration. 3. Answer: 2. Instruct the mother to request help when getting out of bed. Orthostatic hypotension may be evident during the first 8 hours after birth. Feelings of faintness or dizziness are signs that should caution the nurse to be aware of the clie nts safety. The nurse should advise the mother to get help the first few times the mother gets out of bed. Obtaining an H/H requires a physicians order. 4. Answer: 3. Ask the mother to urinate and empty her bladder. Before starting the fundal assessment, the nurse should ask the mother to empty her bladder so that an accurate assessment can be done. When the nurse is performing fundal assessment, the nurse asks the woman to lie flat on her back with the knees flexed. Massaging the fundus is not appropriate unless the fundus is boggy and soft, and then it should be massaged gently until firm. 5. Answer: 2. Indicates the presence of infection. Lochia, the discharge present after birth, is red for the first 1 to 3 days and gradually decreases in amount. Normal lochia has a fleshy odor. Foul smelling or purulent lochia usually indicates infection, and these findings are not normal. Encouraging the woman to drink fluids or increase ambulation is not an accurate nursing intervention. 6. Answer: 2. Notify the physician. Normally, one may find a few small clots in the first 1 to 2 days after birth from pooling of blood in the vagina. Clots larger than 1 cm are considered abnormal. The cause of these clots, such as uterine atony or retained placental fragments, needs to be determined and treated to prevent further blood loss. Although the findings would be documented, the most appropriate action is to notify the physician. 7. Answer: 4. Eight peripads per day. The normal amount of lochia may vary with the individual but should never exceed 4 to 8 peripads per day. The average number of peripads is 6 per day. 8. Answer: 2. 3 days PP. After birth, the nurse should auscultate the womans abdomen in all four quadrants to determine the return of bowel sounds. Normal bowel elimination usually returns 2 to 3 days PP. Surgery, anesthesia, and the use of narcotics and pain control agents also contribute to the longer period of altered bowel function. 9. Answer: 1 and 3. In the PP period, cervical healing occurs rapidly and cervical involution occurs. After 1 week the muscle begins to regenerate and the cervix feels firm and the external os is the width of a pencil. Although the vaginal mucosa heals and vaginal distention decreases, it takes the entire PP period for complete involution to occur and muscle tone is never restored to the pregravid state. The fundus begins to descent into the pelvic cavity after 24 hours, a process known as involution. Despite blood loss that occurs during delivery of the baby, a transient increase in cardiac output occurs. The increase in cardiac output, which persists about 48 hours after childbirth, is probably caused by an increase in stroke volume because Bradycardia is often noted during the PP period. Soon after childbirth, digestion begins to begin to be active and the new mother is usually hungry because of the energy expended during labor. 10. Answer: 3. Changes in vital signs. Because the woman has had epidural anesthesia and is anesthetized, she cannot feel pain, pressure, or a tearing sensation. Changes in vitals indicate hypovolemia in the anesthetized PP woman with vulvar hematoma. Heavy bruising may be visualized, but vital sign changes indicate hematoma caused by blood collection in the perineal tissues. 11. Answer: 4. Prepare an ice pack for application to the area. Application of ice will reduce swelling caused by hematoma formation in the vulvar area. The other options are not interventions that are specific to the plan of care for a client with a small vulvar hematoma. 12. Answer: 3. Prepare the client for surgery. The use of an epidural, prolonged second stage labor and forceps delivery are predisposing factors for hematoma formation, and a collection of up to 500 ml of blood can occur in the vaginal area. Although the other options may be implemented, the immediate action would be to prepare the client for surgery to stop the bleeding. 13. Answer: 2. An increase in the pulse from 88 to 102 BPM. During the 4th stage of labor, the maternal blood pressure, pulse, and respiration should be checked every 15 minutes during the first hour. A rising pulse is an early sign of excessive blood loss because the heart pumps faster to compensate for reduced blood volume. The blood pressure will fall as the blood volume

98

diminishes, but a decreased blood pressure would not be the earliest sign of hemorrhage. A slight rise in temperature is normal. The respiratory rate is increased slightly. 14. Answer: 1. Massage the fundus until it is firm. If the uterus is not contracted firmly, the first intervention is to massage the fundus until it is firm and to express clots that may have accumulated in the uterus. Pushing on an uncontracted uterus can invert the uterus and cause massive hemorrhage. Elevating the clients legs and encouraging the client to void will not assist in managing uterine atony. If the uterus does not remain contracted as a result of the uterine massage, the problem may be distended bladder and the nurse should assist the mother to urinate, but this would not be the initial action. 15. Answer: 2. Enlarged, hardened veins. Thrombosis of the superficial veins is usually accompanied by signs and symptoms of inflammation. These include swelling of the involved extremity and redness, tenderness, and warmth. 16. Answer: 4. I need to stop breastfeeding until this condition resolves. In most cases, the mother can continue to breastfeed with both breasts. If the affected breast is too sore, the mother can pump the breast gently. Regular emptying of the breast is important to prevent abscess formation. Antibiotic therapy assists in resolving the mastitis within 24-48 hours. Additional supportive measures include ice packs, breast supports, and analgesics. 17. Answer: 3. Hematuria, ecchymosis, and epistaxis. The treatment for DVT is anticoagulant therapy. The nurse assesses for bleeding, which is an adverse effect of anticoagulants. This includes hematuria, ecchymosis, and epistaxis. Dysuria and vertigo are not associated specifically with bleeding. 18. Answer: 1. Assess for hypovolemia and notify the health care provider. Symptoms of hypovolemia include cool, clammy, pale skin, sensations of anxiety or impending doom, restlessness, and thirst. When these symptoms are present, the nurse should further assess for hypovolemia and notify the health care provider. 19. Answer: 3. Notify the physician. If the bleeding is excessive, the cause may be laceration of the cervix or birth canal. Massaging the fundus if it is firm will not assist in controlling the bleeding. Trendelenburgs position is to be avoided because it may interfere with cardiac function. 20. 3. Activated partial thromboplastin time. Anticoagulation therapy may be used to prevent the extension of thrombus by delaying the clotting time of the blood. Activated partial thromboplastin time should be monitored, and a heparin dose should be adjusted to maintain a therapeutic level of 1.5 to 2.5 times the control. The prothrombin time and the INR are used to monitor coagulation time when warfarin (Coumadin) is used. 21. Answer: 2, 4, and 5. Mastitis are an infection of the lactating breast. Client instructions include resting during the acute phase, maintaining a fluid intake of at least 3 L a day, and taking analgesics to relieve discomfort. Antibiotics may be prescribed and are taken until the complete prescribed course is finished. They are not stopped when the soreness subsides. Additional supportive measures include the use of moist heat or ice packs and wearing a supportive bra. Continued decompression of the breast by breastfeeding or pumping is important to empty the breast and prevent formation of an abscess. 22. Answer: 2. Blood pressure. Methergine and pitocin are agents that are used to prevent or control postpartum hemorrhage by contracting the uterus. They cause continuous uterine contractions and may elevate blood pressure. A priority nursing intervention is to check blood pressure. The physician should be notified if hypertension is present. 23. Answer: 1. Peripheral vascular disease. These medications are avoided in clients with significant cardiovascular disease, peripheral disease, hypertension, eclampsia, or preeclampsia. These conditions are worsened by the vasoconstriction effects of these medications. 24. Answer: 1. Supplemental feedings with formula. Routine formula supplementation may interfere with establishing an adequate milk volume because decreased stimulation to the mothers nipples affects hormonal levels and milk production. 25. Answer: 3. Teaching how to express her breasts in a warm shower. Teaching the client how to express her breasts in a warm shower aids with let-down and will give temporary relief. Ice can promote comfort by vasoconstriction, numbing, and discouraging further letdown of milk. 26. Answer: 1. Ask the client to empty her bladder. A full bladder may displace the uterine fundus to the left or right side of the abdomen. Catheterization is unnecessary invasive if the woman can void on her own. 27. Answer: 3. Lower than before she became pregnant. PP insulin requirements are usually significantly lower than pre pregnancy requirements. Occasionally, clients may require little to no insulin during the first 24 to 48 hours postpartum.

99

28. Answer: 1. Fundus 1 cm above the umbilicus 1 hour postpartum. Within the first 12 hours postpartum, the fundus usually is approximately 1 cm above the umbilicus. The fundus should be below the umbilicus by PP day 3. The fundus shouldnt be palpated in the abdomen after day 10. 29. Answer: 3. Multiple gestation. Multiple gestation, breastfeeding, multiparity, and conditions that cause overdistention of the uterus will increase the intensity of after-pains. Bottle-feeding and diabetes arent directly associated with increasing severity of afterpains unless the client has delivered a macrosomic infant. 30. Answer: 2. Days 3 to 10 PP. On the third and fourth PP days, the lochia becomes a pale pink or brown and contains old blood, serum, leukocytes, and tissue debris. This type of lochia usually lasts until PP day 10. Lochia rubra usually last for the first 3 to 4 days PP. Lochia alba, which contain leukocytes, decidua, epithelial cells, mucus, and bacteria, may continue for 2 to 6 weeks PP. 31. Answer: 1. Passive and dependant. During the taking in phase, which usually lasts 1-3 days, the mother is passive and dependent and expresses her own needs rather than the neonates needs. The taking hold phase usually lasts from days 3 -10 PP. During this stage, the mother strives for independence and autonomy; she also becomes curious and interested in the care of the baby and is most ready to learn. 32. Answer: 3. Cervical laceration. Continuous seepage of blood may be due to cervical or vaginal lacerations if the uterus is firm and contracting. Retained placental fragments and uterine atony may cause subinvolution of the uterus, making it soft, boggy, and larger than expected. UTI wont cause vaginal bleeding, although hematuria may be present. 33. Answer: 4. Transitional milk. Transitional milk comes after colostrum and usually lasts until 2 weeks PP. 34. Answer: 4. Uterine subinvolution. Late postpartum bleeding is often the result of subinvolution of the uterus. Retained products of conception or infection often cause subinvolution. Cervical or perineal lacerations can cause an immediate postpartum hemorrhage. A client with a clotting deficiency may also have an immediate PP hemorrhage if the deficiency isnt corrected at the time of delivery. 35. Answer: 4. The client should avoid getting pregnant for 3 months after the vaccine because the vaccine has teratogenic effects. The client must understand that she must not become pregnant for 3 months after the vaccination because of its potential teratogenic effects. The rubella vaccine is made from duck eggs so an allergic reaction may occur in clients with egg allergies. The virus is not transmitted into the breast milk, so clients may continue to breastfeed after the vaccination. Transient arthralgia and rash are common adverse effects of the vaccine. 36. Answer: 2. Decrease. The placenta produces the hormone human placental lactogen, an insulin antagonist. After birth, the placenta, the major source of insulin resistance, is gone. Insulin needs decrease and women with type 1 diabetes may only need one-half to two-thirds of the prenatal insulin during the first few PP days. 37. Answer: 4. Mothers with diabetes may breastfeed; insulin requirements may decrease from breastfeeding. Breastfeeding has an antidiabetogenic effect. Insulin needs are decreased because carbohydrates are used in milk production. Breastfeeding mothers are at a higher risk of hypoglycemia in the first PP days after birth because the glucose levels are lower. Mothers with diabetes should be encouraged to breastfeed. 38. Answer: 4. Taking-in phase. The taking-in phase occurs in the first 24 hours after birth. The mother is concerned with her own needs and requires support from staff and relatives. The taking-hold phase occurs when the mother is ready to take responsibility for her care as well as the infants care. The letting-go phase begins several weeks later, when the mother incorporates the new infant into the family unit. 39. Answer: 2. Rapid diuresis. In the early PP period, theres an increase in the glomerular filtration rate and a drop in the progesterone levels, which result in rapid diuresis. There should be no urinary urgency, though a woman may feel anxious about voiding. Theres a minimal change in blood pressure following childbirth, and a residual decrease in GI motility. 40. Answer: 1. The client appears interested in learning about neonatal care. The third to tenth days of PP care are the taking-hold phase, in which the new mother strives for independence and is eager for her neonate. The other options describe the phase in which the mother relives her birth experience. 41. Answer: 3. Urine retention. Urine retention causes a distended bladder to displace the uterus above the umbilicus and to the side, which prevents the uterus from contracting. The uterus needs to remain contracted if bleeding is to stay within normal limits. Cervical and vaginal tears can cause PP hemorrhage but are less common occurrences in the PP period.

100

42. Answer: 4. Lochia rubra 43. Answer: 4. Multigravidas are at increased risk for uterine atony. Multiple full-term pregnancies and deliveries result in overstretched uterine muscles that do not contract efficiently and bleeding may ensue. 44. Answer: 1. Soft, non-tender; colostrum is present. Breasts are essentially unchanged for the first two to three days after birth. Colostrum is present and may leak from the nipples. 45. Answer: 3. The expected weight loss immediately after birth averages about 11 to 13 pounds. Prepregnant weight is usually achieved by 2 to 3 months after birth, not within the 6-week postpartum period. Weight loss from diuresis, diaphoresis, and bleeding is about 9 pounds. Weight loss continues during breastfeeding since fat stores developed during pregnancy and extra calories consumed are used as part of the lactation process. 46. Answer: 4. Pain in left calf with dorsiflexion of left foot. Responses 1 and 3 are expected related to circulatory changes after birth. A temperature of 100.4F in the first 24 hours is most likely indicative of dehydration which is easily corrected by increasing oral fluid intake. The findings in response 4 indicate a positive Homan sign and are suggestive of thrombophlebitis and should be investigated further. 47. Answer: 2. Massage her fundus. A boggy or soft fundus indicates that uterine atony is present. This is confirmed by the profuse lochia and passage of clots. The first action would be to massage the fundus until firm, followed by 3 and 4, especially if the fundus does not become or remain firm with massage. There is no indication of a distended bladder since the fundus is midline and below the umbilicus. 48. Answer: 1. Assist the woman into a lateral position with upper leg flexed forward to facilitate the examination of her perineum. While the supine position is best for examining the abdomen, the woman should keep her arms at her sides and slightly flex her knees in order to relax abdominal muscles and facilitate palpation of the fundus. The bladder should be emptied before the check. A full bladder alters the position of the fundus and makes the findings inaccurate. Although hands are washed before starting the check, clean (not sterile) gloves are put on just before the perineum and pad are assessed to protect from contact with blood and secretions. 49. Answer: 4. Uses the peribottle to rinse upward into her vagina. Responses 1, 2, and 3 are all appropriate measures. The peribottle should be used in a backward direction over the perineum. The flow should never be directed upward into the vagina since debris would be forced upward into the uterus through the still-open cervix. 50. Answer: 3. Massage the fundus every hour for the first 24 hours following birth. The fundus should be massaged only when boggy or soft. Massaging a firm fundus could cause it to relax. Responses 1, 2, and 4 are all effective measures to enhance and maintain contraction of the uterus and to facilitate healing. 51. Answer: 3. Express a strong need to review events and her behavior during the process of labor and birth. One week after birth the woman should exhibit behaviors characteristic of the taking-hold stage as described in response 3. This stage lasts for as long as 4 to 5 weeks after birth. Responses 1 and 2 are characteristic of the taking-in stage, which lasts for the first few days after birth. Response 4 reflects the letting-go stage, which indicates that psychosocial recovery is complete. 52. Answer: 4. Recognize this as a behavior of the taking-hold stage. Response 1 does not take into consideration the need for the new mother to be nurtured and have her needs met during the taking-in stage. The behavior described is typical of this stage and not a reflection of ineffective attachment unless the behavior persists. Mothers need to reestablish their own well-being in order to effectively care for their baby. 53. Answer: 1. Having the children choose or make a gift to give to the new baby upon its arrival home. Special time should be set aside just for the other children without interruption from the newborn. Someone other than the mother should carry the baby into the home so she can give full attention to greeting her other children. Children should be actively involved in the care of the baby according to their ability without overwhelming them. 54. Answer: 2. Provide time for the mother to reflect on the events of and her behavior during childbirth. The focus of the taking-in stage is nurturing the new mother by meeting her dependency needs for rest, comfort, hygiene, and nutrition. Once they are met, she is more able to take an active role, not only in her own care but also the care of her newborn. Women express a need to review their childbirth experience and evaluate their performance. Short teaching sessions, using written materials to reinforce the content presented, are a more effective approach. 55. Answer: 3. Placement in a warm environment

101

Introduction
A 50-item NCLEX style questionnaire that challenges your knowledge about Obstetrical Nursing. This includes nursing care for pregnant women during the antepartum period.

Topics

Obstetrical Nursing Antepartal Care GTPAL Questions Preeclampsia

Guidelines
Read each question carefully and choose the best answer. You are given one minute per question. Spend your time wisely! Answers and rationales are given below. Be sure to read them. If you need more clarifications, please direct them to the comments section.

Questions
1. A nursing instructor is conducting lecture and is reviewing the functions of the female reproductive system. She asks Mark to describe the follicle-stimulating hormone (FSH) and the luteinizing hormone (LH). Mark accurately responds by stating that: 1. FSH and LH are released from the anterior pituitary gland. 2. FSH and LH are secreted by the corpus luteum of the ovary 3. FSH and LH are secreted by the adrenal glands 4. FSH and LH stimulate the formation of milk during pregnancy. 2. A nurse is describing the process of fetal circulation to a client during a prenatal visit. The nurse accurately tells the client that fetal circulation consists of: 1. Two umbilical veins and one umbilical artery 2. Two umbilical arteries and one umbilical vein 3. Arteries carrying oxygenated blood to the fetus 4. Veins carrying deoxygenated blood to the fetus 3. During a prenatal visit at 38 weeks, a nurse assesses the fetal heart rate. The nurse determines that the fetal heart rate is normal if which of the following is noted? 1. 80 BPM 2. 100 BPM 3. 150 BPM 4. 180 BPM 4. A client arrives at a prenatal clinic for the first prenatal assessment. The client tells a nurse that the first day of her last menstrual period was September 19th, 2005. Using Naegeles rule, the nurse determines the estimated date of confinement as: 1. July 26, 2006 2. June 12, 2007 3. June 26, 2006 4. July 12, 2007

102

5. A nurse is collecting data during an admission assessment of a client who is pregnant with twins. The client has a healthy 5-year old child that was delivered at 37 weeks and tells the nurse that she doesnt have any history of abortion or fetal demise. The nurse would document the GTPAL for this client as: 1. G = 3, T = 2, P = 0, A = 0, L =1 2. G = 2, T = 0, P = 1, A = 0, L =1 3. G = 1, T = 1. P = 1, A = 0, L = 1 4. G = 2, T = 0, P = 0, A = 0, L = 1 6. A nurse is performing an assessment of a primipara who is being evaluated in a clinic during her second trimester of pregnancy. Which of the following indicates an abnormal physical finding necessitating further testing? 1. Consistent increase in fundal height 2. Fetal heart rate of 180 BPM 3. Braxton hicks contractions 4. Quickening 7. A nurse is reviewing the record of a client who has just been told that a pregnancy test is positive. The physician has documented the presence of a Goodells sign. The nurse determines this sign indicates: 1. A softening of the cervix 2. A soft blowing sound that corresponds to the maternal pulse during auscultation of the uterus. 3. The presence of hCG in the urine 4. The presence of fetal movement 8. A nursing instructor asks a nursing student who is preparing to assist with the assessment of a pregnant client to describe the process of quickening. Which of the following statements if made by the student indicates an understanding of this term? 1. It is the irregular, painless contractions that occur throughout pregnancy. 2. It is the soft blowing sound that can be heard when the uterus is auscultated. 3. It is the fetal movement that is felt by the mother. 4. It is the thinning of the lower uterine segment. 9. A nurse midwife is performing an assessment of a pregnant client and is assessing the client for the presence of ballottement. Which of the following would the nurse implement to test for the presence of ballottement? 1. Auscultating for fetal heart sounds 2. Palpating the abdomen for fetal movement 3. Assessing the cervix for thinning 4. Initiating a gentle upward tap on the cervix 10. A nurse is assisting in performing an assessment on a client who suspects that she is pregnant and is checking the client for probable signs of pregnancy. Select all probable signs of pregnancy. 1. Uterine enlargement 2. Fetal heart rate detected by nonelectric device 3. Outline of the fetus via radiography or ultrasound 4. Chadwicks sign 5. Braxton Hicks contractions 6. Ballottement 11. A pregnant client calls the clinic and tells a nurse that she is experiencing leg cramps and is awakened by the cramps at night. To provide relief from the leg cramps, the nurse tells the client to: 1. Dorsiflex the foot while extending the knee when the cramps occur 2. Dorsiflex the foot while flexing the knee when the cramps occur 3. Plantar flex the foot while flexing the knee when the cramps occur 4. Plantar flex the foot while extending the knee when the cramps occur.

103

12. A nurse is providing instructions to a client in the first trimester of pregnancy regarding measures to assist in reducing breast tenderness. The nurse tells the client to: 1. Avoid wearing a bra 2. Wash the nipples and areola area daily with soap, and massage the breasts with lotion. 3. Wear tight-fitting blouses or dresses to provide support 4. Wash the breasts with warm water and keep them dry 13. A pregnant client in the last trimester has been admitted to the hospital with a diagnosis of severe preeclampsia. A nurse monitors for complications associated with the diagnosis and assesses the client for: 1. Any bleeding, such as in the gums, petechiae, and purpura. 2. Enlargement of the breasts 3. Periods of fetal movement followed by quiet periods 4. Complaints of feeling hot when the room is cool 14. A client in the first trimester of pregnancy arrives at a health care clinic and reports that she has been experiencing vaginal bleeding. A threatened abortion is suspected, and the nurse instructs the client regarding management of care. Which statement, if made by the client, indicates a need for further education? 1. I will maintain strict bedrest throughout the remainder of pregnancy. 2. I will avoid sexual intercourse until the bleeding has stopped, and for 2 weeks following the last evidence of bleeding. 3. I will count the number of perineal pads used on a daily basis and note the amount and color of blood on the pad. 4. I will watch for the evidence of the passage of tissue. 15. A prenatal nurse is providing instructions to a group of pregnant client regarding measures to prevent toxoplasmosis. Which statement if made by one of the clients indicates a need for further instructions? 1. I need to cook meat thoroughly. 2. I need to avoid touching mucous membranes of the mouth or eyes while handling raw meat. 3. I need to drink unpasteurized milk only. 4. I need to avoid contact with materials that are possibly contaminated with cat feces. 16. A homecare nurse visits a pregnant client who has a diagnosis of mild Preeclampsia and who is being monitored for pregnancy induced hypertension (PIH). Which assessment finding indicates a worsening of the Preeclampsia and the need to notify the physician? 1. Blood pressure reading is at the prenatal baseline 2. Urinary output has increased 3. The client complains of a headache and blurred vision 4. Dependent edema has resolved 17. A nurse implements a teaching plan for a pregnant client who is newly diagnosed with gestational diabetes. Which statement if made by the client indicates a need for further education? 1. I need to stay on the diabetic diet. 2. I will perform glucose monitoring at home. 3. I need to avoid exercise because of the negative effects of insulin production. 4. I need to be aware of any infections and report signs of infection immediately to my health care provider. 18. A primigravida is receiving magnesium sulfate for the treatment of pregnancy induced hypertension (PIH). The nurse who is caring for the client is performing assessments every 30 minutes. Which assessment finding would be of most concern to the nurse? 1. Urinary output of 20 ml since the previous assessment 2. Deep tendon reflexes of 2+ 3. Respiratory rate of 10 BPM 4. Fetal heart rate of 120 BPM 19. A nurse is caring for a pregnant client with Preeclampsia. The nurse prepares a plan of care for the client and documents in the plan that if the client progresses from Preeclampsia to eclampsia, the nurses first action is to:

104

1. Administer magnesium sulfate intravenously 2. Assess the blood pressure and fetal heart rate 3. Clean and maintain an open airway 4. Administer oxygen by face mask 20. A nurse is monitoring a pregnant client with pregnancy induced hypertension who is at risk for Preeclampsia. The nurse checks the client for which specific signs of Preeclampsia (select all that apply)? 1. Elevated blood pressure 2. Negative urinary protein 3. Facial edema 4. Increased respirations 21. Rho (D) immune globulin (RhoGAM) is prescribed for a woman following delivery of a newborn infant and the nurse provides information to the woman about the purpose of the medication. The nurse determines that the woman understands the purpose of the medication if the woman states that it will protect her next baby from which of the following? 1. Being affected by Rh incompatibility 2. Having Rh positive blood 3. Developing a rubella infection 4. Developing physiological jaundice 22. A pregnant client is receiving magnesium sulfate for the management of preeclampsia. A nurse determines the client is experiencing toxicity from the medication if which of the following is noted on assessment? 1. Presence of deep tendon reflexes 2. Serum magnesium level of 6 mEq/L 3. Proteinuria of +3 4. Respirations of 10 per minute 23. A woman with preeclampsia is receiving magnesium sulfate. The nurse assigned to care for the client determines that the magnesium therapy is effective if: 1. Ankle clonus in noted 2. The blood pressure decreases 3. Seizures do not occur 4. Scotomas are present 24. A nurse is caring for a pregnant client with severe preeclampsia who is receiving IV magnesium sulfate. Select all nursing interventions that apply in the care for the client. 1. Monitor maternal vital signs every 2 hours 2. Notify the physician if respirations are less than 18 per minute. 3. Monitor renal function and cardiac function closely 4. Keep calcium gluconate on hand in case of a magnesium sulfate overdose 5. Monitor deep tendon reflexes hourly 6. Monitor I and Os hourly 7. Notify the physician if urinary output is less than 30 ml per hour. 25. In the 12th week of gestation, a client completely expels the products of conception. Because the client is Rh negative, the nurse must: 1. Administer RhoGAM within 72 hours 2. Make certain she receives RhoGAM on her first clinic visit 3. Not give RhoGAM, since it is not used with the birth of a stillborn 4. Make certain the client does not receive RhoGAM, since the gestation only lasted 12 weeks. 26. In a lecture on sexual functioning, the nurse plans to include the fact that ovulation occurs when the:

105

1. Oxytocin is too high 2. Blood level of LH is too high 3. Progesterone level is high 4. Endometrial wall is sloughed off. 27. The chief function of progesterone is the: 1. Development of the female reproductive system 2. Stimulation of the follicles for ovulation to occur 3. Preparation of the uterus to receive a fertilized egg 4. Establishment of secondary male sex characteristics 28. The developing cells are called a fetus from the: 1. Time the fetal heart is heard 2. Eighth week to the time of birth 3. Implantation of the fertilized ovum 4. End of the send week to the onset of labor 29. After the first four months of pregnancy, the chief source of estrogen and progesterone is the: 1. Placenta 2. Adrenal cortex 3. Corpus luteum 4. Anterior hypophysis 30. The nurse recognizes that an expected change in the hematologic system that occurs during the 2nd trimester of pregnancy is: 1. A decrease in WBCs 2. In increase in hematocrit 3. An increase in blood volume 4. A decrease in sedimentation rate 31. The nurse is aware than an adaptation of pregnancy is an increased blood supply to the pelvic region that results in a purplish discoloration of the vaginal mucosa, which is known as: 1. Ladins sign 2. Hegars sign 3. Goodells sign 4. Chadwicks sign 32. A pregnant client is making her first Antepartum visit. She has a two year old son born at 40 weeks, a 5 year old daughter born at 38 weeks, and 7 year old twin daughters born at 35 weeks. She had a spontaneous abortion 3 years ago at 10 weeks. Using the GTPAL format, the nurse should identify that the client is: 1. G4 T3 P2 A1 L4 2. G5 T2 P2 A1 L4 3. G5 T2 P1 A1 L4 4. G4 T3 P1 A1 L4 33. An expected cardiopulmonary adaptation experienced by most pregnant women is: 1. Tachycardia 2. Dyspnea at rest 3. Progression of dependent edema 4. Shortness of breath on exertion 34. Nutritional planning for a newly pregnant woman of average height and weighing 145 pounds should include: 1. A decrease of 200 calories a day 2. An increase of 300 calories a day

106

3. An increase of 500 calories a day 4. A maintenance of her present caloric intake per day 35. During a prenatal examination, the nurse draws blood from a young Rh negative client and explain that an indirect Coombs test will be performed to predict whether the fetus is at risk for: 1. Acute hemolytic disease 2. Respiratory distress syndrome 3. Protein metabolic deficiency 4. Physiologic hyperbilirubinemia 36. When involved in prenatal teaching, the nurse should advise the clients that an increase in vaginal secretions during pregnancy is called leukorrhea and is caused by increased: 1. Metabolic rates 2. Production of estrogen 3. Functioning of the Bartholin glands 4. Supply of sodium chloride to the cells of the vagina 37. A 26-year old multigravida is 14 weeks pregnant and is scheduled for an alpha-fetoprotein test. She asks the nurse, What does the alpha-fetoprotein test indicate? The nurse bases a response on the knowledge that this test can detect: 1. Kidney defects 2. Cardiac defects 3. Neural tube defects 4. Urinary tract defects 38. At a prenatal visit at 36 weeks gestation, a client complains of discomfort with irregularly occurring contractions. The nurse instructs the client to: 1. Lie down until they stop 2. Walk around until they subside 3. Time contraction for 30 minutes 4. Take 10 grains of aspirin for the discomfort 39. The nurse teaches a pregnant woman to avoid lying on her back. The nurse has based this statement on the knowledge that the supine position can: 1. Unduly prolong labor 2. Cause decreased placental perfusion 3. Lead to transient episodes of hypotension 4. Interfere with free movement of the coccyx 40. The pituitary hormone that stimulates the secretion of milk from the mammary glands is: 1. Prolactin 2. Oxytocin 3. Estrogen 4. Progesterone 41. Which of the following symptoms occurs with a hydatidiform mole? 1. Heavy, bright red bleeding every 21 days 2. Fetal cardiac motion after 6 weeks gestation 3. Benign tumors found in the smooth muscle of the uterus 4. Snowstorm pattern on ultrasound with no fetus or gestational sac 42. Which of the following terms applies to the tiny, blanched, slightly raised end arterioles found on the face, neck, arms, and chest during pregnancy? 1. Epulis 2. Linea nigra

107

3. Striae gravidarum 4. Telangiectasias 43. Which of the following conditions is common in pregnant women in the 2nd trimester of pregnancy? 1. Mastitis 2. Metabolic alkalosis 3. Physiologic anemia 4. Respiratory acidosis 44. A 21-year old client, 6 weeks pregnant is diagnosed with hyperemesis gravidarum. This excessive vomiting during pregnancy will often result in which of the following conditions? 1. Bowel perforation 2. Electrolyte imbalance 3. Miscarriage 4. Pregnancy induced hypertension (PIH) 45. Clients with gestational diabetes are usually managed by which of the following therapies? 1. Diet 2. NPH insulin (long-acting) 3. Oral hypoglycemic drugs 4. Oral hypoglycemic drugs and insulin 46. The antagonist for magnesium sulfate should be readily available to any client receiving IV magnesium. Which of the following drugs is the antidote for magnesium toxicity? 1. Calcium gluconate 2. Hydralazine (Apresoline) 3. Narcan 4. RhoGAM 47. Which of the following answers best describes the stage of pregnancy in which maternal and fetal blood are exchanged? 1. Conception 2. 9 weeks gestation, when the fetal heart is well developed 3. 32-34 weeks gestation 4. maternal and fetal blood are never exchanged 48. Gravida refers to which of the following descriptions? 1. A serious pregnancy 2. Number of times a female has been pregnant 3. Number of children a female has delivered 4. Number of term pregnancies a female has had. 49. A pregnant woman at 32 weeks gestation complains of feeling dizzy and lightheaded while her fundal height is being measured. Her skin is pale and moist. The nurses initial response would be to: 1. Assess the womans blood pressure and pulse 2. Have the woman breathe into a paper bag 3. Raise the womans legs 4. Turn the woman on her side. 50. A pregnant womans last menstrual period began on April 8, 2005, and ended on April 13. Using Naegeles rule her estimated date of birth would be: 1. January 15, 2006 2. January 20, 2006 3. July 1, 2006 4. November 5, 2005

108

Answers and Rationale


Gauge your performance by counter checking your answers to the answers below. Learn more about the question by reading the rationale. If you have any disputes or questions, please direct them to the comments section. 1. Answer: 1. FSH and LH are released from the anterior pituitary gland. FSH and LH, when stimulated by gonadotropinreleasing hormone from the hypothalamus, are released from the anterior pituitary gland to stimulate follicular growth and development, growth of the graafian follicle, and production of progesterone. 2. Answer: 2. Two umbilical arteries and one umbilical vein. Blood pumped by the embryos heart leaves the embryo through two umbilical arteries. Once oxygenated, the blood then is returned by one umbilical vein. Arteries carry deoxygenated blood and waste products from the fetus, and veins carry oxygenated blood and provide oxygen and nutrients to the fetus. 3. Answer: 3. 150 BPM. The fetal heart rate depends in gestational age and ranges from 160-170 BPM in the first trimester but slows with fetal growth to 120-160 BPM near or at term. At or near term, if the fetal heart rate is less than 120 or more than 160 BPM with the uterus at rest, the fetus may be in distress. 4. Answer: 3. June 26, 2006. Accurate use of Naegeles rule requires that the woman have a regular 28 -day menstrual cycle. Add 7 days to the first day of the last menstrual period, subtract three months, and then add one year to that date. 5. Answer: 2. G = 2, T = 0, P = 1, A = 0, L =1. Pregnancy outcomes can be described with the acronym GTPAL.

G is Gravidity, the number of pregnancies. T is term births, the number of born at term (38 to 41 weeks). P is preterm births, the number born before 38 weeks gestation. A is abortions or miscarriages, included in G if before 20 weeks gestation, included in parity if past 20 weeks AOE. L is live births, the number of births of living children. Therefore, a woman who is pregnant with twins and has a child has a gravida of 2. Because the child was delivered at 37 weeks, the number of preterm births is 1, and the number of term births is 0. The number of abortions is 0, and the number of live births is 1. 6. Answer: 2. Fetal heart rate of 180 BPM. The normal range of the fetal heart rate depends on gestational age. The heart rate is usually 160-170 BPM in the first trimester and slows with fetal growth, near and at term, the fetal heart rate ranges from 120-160 BPM. The other options are expected. 7. Answer: 1. A softening of the cervix. In the early weeks of pregnancy the cervix becomes softer as a result of increased vascularity and hyperplasia, which causes the Goodells sign. 8. Answer: 3. It is the fetal movement that is felt by the mother. Quickening is fetal movement and may occur as early as the 16th and 18th week of gestation, and the mother first notices subtle fetal movements that gradually increase in intensity. Braxton Hicks contractions are irregular, painless contractions that may occur throughout the pregnancy. A thinning of the lower uterine segment occurs about the 6th week of pregnancy and is called Hegars sign. 9. Answer: 4. Initiating a gentle upward tap on the cervix. Ballottement is a technique of palpating a floating structure by bouncing it gently and feeling it rebound. In the technique used to palpate the fetus, the examiner places a finger in the vagina and taps gently upward, causing the fetus to rise. The fetus then sinks, and the examiner feels a gentle tap on the finger. 10. Answers: 1, 4, 5, and 6. The probable signs of pregnancy include:

Uterine Enlargement Hegars sign or softening and thinning of the uterine segment that occurs at week 6. Goodells sign or softening of the cervix that occurs at the beginning of the 2nd month Chadwicks sign or bluish coloration of the mucous membranes of the cervix, vagina and vulva. Occurs at week 6. Ballottement or rebounding of the fetus against the examiners fingers of palpation Braxton-Hicks contractions Positive pregnancy test measuring for hCG.

109

Positive signs of pregnancy include:

Fetal Heart Rate detected by electronic device (doppler) at 10-12 weeks Fetal Heart rate detected by nonelectronic device (fetoscope) at 20 weeks AOG Active fetal movement palpable by the examiners Outline of the fetus via radiography or ultrasound 11. Answer: 1. Dorsiflex the foot while extending the knee when the cramps occur. Legs cramps occur when the pregnant woman stretches the leg and plantar flexes the foot. Dorsiflexion of the foot while extending the knee stretches the affected muscle, prevents the muscle from contracting, and stops the cramping. 12. Answer: 4. Wash the breasts with warm water and keep them dry. The pregnant woman should be instructed to wash the breasts with warm water and keep them dry. The woman should be instructed to avoid using soap on the nipples and areola area to prevent the drying of tissues. Wearing a supportive bra with wide adjustable straps can decrease breast tenderness. Tight-fitting blouses or dresses will cause discomfort. 13. Answer: 1. Any bleeding, such as in the gums, petechiae, and purpura. Severe Preeclampsia can trigger disseminated intravascular coagulation because of the widespread damage to vascular integrity. Bleeding is an early sign of DIC and should be reported to the M.D. 14. Answer: 1. I will maintain strict bedrest throughout the remainder of pregnancy. Strict bed rest throughout the remainder of pregnancy is not required. The woman is advised to curtail sexual activities until the bleeding has ceased, and for 2 weeks following the last evidence of bleeding or as recommended by the physician. The woman is instructed to count the number of perineal pads used daily and to note the quantity and color of blood on the pad. The woman also should watch for the evidence of the passage of tissue. 15. Answer: 3. I need to drink unpasteurized milk only. All pregnant women should be advised to do the following to prevent the development of toxoplasmosis. Women should be instructed to cook meats thoroughly, avoid touching mucous membranes and eyes while handling raw meat; thoroughly wash all kitchen surfaces that come into contact with uncooked meat, wash the hands thoroughly after handling raw meat; avoid uncooked eggs and unpasteurized milk; wash fruits and vegetables before consumption, and avoid contact with materials that possibly are contaminated with cat feces, such as cat litter boxes, sandboxes, and garden soil. 16. Answer: 3. The client complains of a headache and blurred vision. If the client complains of a headache and blurred vision, the physician should be notified because these are signs of worsening Preeclampsia. 17. Answer: 3. I need to avoid exercise because of the negative effects of insulin production. Exercise is safe for the client with gestational diabetes and is helpful in lowering the blood glucose level. 18. Answer: 3. Respiratory rate of 10 BPM. Magnesium sulfate depresses the respiratory rate. If the respiratory rate is less than 12 breaths per minute, the physician or other health care provider needs to be notified, and continuation of the medication needs to be reassessed. A urinary output of 20 ml in a 30 minute period is adequate; less than 30 ml in one hour needs to be reported. Deep tendon reflexes of 2+ are normal. The fetal heart rate is WNL for a resting fetus. 19. Answer: 3. Clean and maintain an open airway. The immediate care during a seizure (eclampsia) is to ensure a patent airway. The other options are actions that follow or will be implemented after the seizure has ceased. 20. Answers: 1 Elevated blood pressure and 3 Facial edema. The three classic signs of preeclampsia are hypertension, generalized edema, and proteinuria. Increased respirations are not a sign of preeclampsia. 21. Answer: 1. Being affected by Rh incompatibility. Rh incompatibility can occur when an Rh-negative mom becomes sensitized to the Rh antigen. Sensitization may develop when an Rh-negative woman becomes pregnant with a fetus who is Rh positive. During pregnancy and at delivery, some of the babys Rh positive blood can enter the maternal circulation, causing the womans immune system to form antibodies against Rh positive blood. Administration of Rho(D) immune globulin prevents the woman from developing antibodies against Rh positive blood by providing passive antibody protection against the Rh antigen. 22. Answer: 4. Respirations of 10 per minute. Magnesium toxicity can occur from magnesium sulfate therapy. Signs of toxicity relate to the central nervous system depressant effects of the medication and include respiratory depression, loss of deep tendon

110

reflexes, and a sudden drop in the fetal heart rate and maternal heart rate and blood pressure. Therapeutic levels of magnesium are 4-7 mEq/L. Proteinuria of +3 would be noted in a client with preeclampsia. 23. Answer: 3. Seizures do not occur. For a client with preeclampsia, the goal of care is directed at preventing eclampsia (seizures). Magnesium sulfate is an anticonvulsant, not an antihypertensive agent. Although a decrease in blood pressure may be noted initially, this effect is usually transient. Ankle clonus indicated hyperreflexia and may precede the onset of eclampsia. Scotomas are areas of complete or partial blindness. Visual disturbances, such as scotomas, often precede an eclamptic seizure. 24. Answers: 3, 4, 5, 6, and 7. When caring for a client receiving magnesium sulfate therapy, the nurse would monitor maternal vital signs, especially respirations, every 30-60 minutes and notify the physician if respirations are less than 12, because this would indicate respiratory depression. Calcium gluconate is kept on hand in case of magnesium sulfate overdose, because calcium gluconate is the antidote for magnesium sulfate toxicity. Deep tendon reflexes are assessed hourly. Cardiac and renal function is monitored closely. The urine output should be maintained at 30 ml per hour because the medication is eliminated through the kidneys. 25. Answer: 1. Administer RhoGAM within 72 hours. RhoGAM is given within 72 hours postpartum if the client has not been sensitized already. 26. Answer: 2. Blood level of LH is too high. It is the surge of LH secretion in mid cycle that is responsible for ovulation. 27. Answer: 3. Preparation of the uterus to receive a fertilized egg. Progesterone stimulates differentiation of the endometrium into a secretory type of tissue. 28. Answer: 2. Eighth week to the time of birth. In the first 7-14 days the ovum is known as a blastocyst; it is called an embryo until the eighth week; the developing cells are then called a fetus until birth. 29. Answer: 1. Placenta. When placental formation is complete, around the 16th week of pregnancy; it produces estrogen and progesterone. 30. Answer: 3. An increase in blood volume. The blood volume increases by approximately 40-50% during pregnancy. The peak blood volume occurs between 30 and 34 weeks of gestation. The hematocrit decreases as a result of the increased blood volume. 31. Answer: 4. Chadwicks sign. A purplish color results from the increased vascularity and blood vessel engorgement of the vagina. 32. Answer: 3. G5 T2 P1 A1 L4. 5 pregnancies; 2 term births; twins count as 1; one abortion; 4 living children. 33. Answer: 4. Shortness of breath on exertion. This is an expected cardiopulmonary adaptation during pregnancy; it is caused by an increased ventricular rate and elevated diaphragm. 34. Answer: 2. An increase of 300 calories a day. This is the recommended caloric increase for adult women to meet the increased metabolic demands of pregnancy. 35. Answer: 1. Acute hemolytic disease. When an Rh negative mother carries an Rh positive fetus there is a risk for maternal antibodies against Rh positive blood; antibodies cross the placenta and destroy the fetal RBCs. 36. Answer: 2. Production of estrogen. The increase of estrogen during pregnancy causes hyperplasia of the vaginal mucosa, which leads to increased production of mucus by the endocervical glands. The mucus contains exfoliated epithelial cells. 37. Answer: 3. Neural tube defects. The alpha-fetoprotein test detects neural tube defects and Down syndrome. 38. Answer: 2. Walk around until they subside. Ambulation relieves Braxton Hicks. 39. Answer: 2. Cause decreased placental perfusion. This is because impedance of venous return by the gravid uterus, which causes hypotension and decreased systemic perfusion. 40. Answer: 1. Prolactin. Prolactin is the hormone from the anterior pituitary gland that stimulates mammary gland secretion. Oxytocin, a posterior pituitary hormone, stimulates the uterine musculature to contract and causes the let down reflex. 41. Answer: 4. Snowstorm pattern on ultrasound with no fetus or gestational sac. The chorionic villi of a molar pregnancy resemble a snowstorm pattern on ultrasound. Bleeding with a hydatidiform mole is often dark brown and may occur erratically for weeks or months. 42. Answer: 4. Telangiectasias. The dilated arterioles that occur during pregnancy are due to the elevated level of circulating estrogen. The linea nigra is a pigmented line extending from the symphysis pubis to the top of the fundus during pregnancy.

111

43. Answer: 3. Physiologic anemia. Hemoglobin and hematocrit levels decrease during pregnancy as the increase in plasma volume exceeds the increase in red blood cell production. 44. Answer: 2. Electrolyte imbalance. Excessive vomiting in clients with hyperemesis gravidarum often causes weight loss and fluid, electrolyte, and acid-base imbalances. 45. Answer: 1. Diet. Clients with gestational diabetes are usually managed by diet alone to control their glucose intolerance. Oral hypoglycemic agents are contraindicated in pregnancy. NPH isnt usually needed for blood glucose control for GDM. 46. Answer: 1. Calcium gluconate. Calcium gluconate is the antidote for magnesium toxicity. Ten ml of 10% calcium gluconate is given IV push over 3-5 minutes. Hydralazine is given for sustained elevated blood pressures in preeclamptic clients. 47. Answer: 4. maternal and fetal blood are never exchanged. Only nutrients and waste products are transferred across the placenta. Blood exchange only occurs in complications and some medical procedures accidentally. 48. Answer: 2. Number of times a female has been pregnant. Gravida refers to the number of times a female has been pregnant, regardless of pregnancy outcome or the number of neonates delivered. 49. Answer: 4. Turn the woman on her side. During a fundal height measurement the woman is placed in a supine position. This woman is experiencing supine hypotension as a result of uterine compression of the vena cava and abdominal aorta. Turning her on her side will remove the compression and restore cardiac output and blood pressure. Then vital signs can be assessed. Raising her legs will not solve the problem since pressure will still remain on the major abdominal blood vessels, thereby continuing to impede cardiac output. Breathing into a paper bag is the solution for dizziness related to respiratory alkalosis associated with hyperventilation. 50. Answer: 1. January 15, 2006. Naegeles rule requires subtracting 3 months and adding 7 days and 1 year if appropriate to the first day of a pregnant womans last menstrual period. When this rule, is used with April 8, 2005, the estimated date of birth is January 15, 2006.

Introduction
A 60-item examination that questions your wits about Kidney Stones, Bladder Cancer and Diseases of the Prostate.

Topics

Kidney Stones Bladder Cancer Diseases of the Prostate

Guidelines
Read each question carefully and choose the best answer. You are given one minute per question. Spend your time wisely! Answers and rationales are given below. Be sure to read them. If you need more clarifications, please direct them to the comments section.

Questions
1. A client is complaining of severe flank and abdominal pain. A flat plate of the abdomen shows urolithiasis. Which of the following interventions is important? 1. Strain all urine 2. Limit fluid intake 3. Enforce strict bed rest 4. Encourage a high calcium diet 2. A client is receiving a radiation implant for the treatment of bladder cancer. Which of the following interventions is appropriate?

112

1. Flush all urine down the toilet 2. Restrict the clients fluid intake 3. Place the client in a semi-private room 4. Monitor the client for signs and symptoms of cystitis 3. A client has just received a renal transplant and has started cyclosporine therapy to prevent graft rejection. Which of the following conditions is a major complication of this drug therapy? 1. Depression 2. Hemorrhage 3. Infection 4. Peptic ulcer disease 4. A client received a kidney transplant 2 months ago. Hes admitted to the hospital with the diagnosis of acute rejection. Which of the following assessment findings would be expected? 1. Hypotension 2. Normal body temperature 3. Decreased WBC count 4. Elevated BUN and creatinine levels 5. The client is to undergo kidney transplantation with a living donor. Which of the following preoperative assessments is important? 1. Urine output 2. Signs of graft rejection 3. Signs and symptoms of rejection 4. Clients support system and understanding of lifestyle changes. 6. A client had a transurethral prostatectomy for benign prostatic hypertrophy. Hes currently being treated with a continuous bladder irrigation and is complaining of an increase in severity of bladder spasms. Which of the interventions should be done first? 1. Administer an oral analgesic 2. Stop the irrigation and call the physician 3. Administer a belladonna and opium suppository as ordered by the physician. 4. Check for the presence of clots, and make sure the catheter is draining properly. 7. A client is admitted with a diagnosis of hydronephrosis secondary to calculi. The calculi have been removed and postobstructive diuresis is occurring. Which of the following interventions should be done? 1. Take vital signs every 8 hours 2. Weigh the client every other day 3. Assess for urine output every shift 4. Monitor the clients electrolyte levels. 8. A client has passed a renal calculus. The nurse sends the specimen to the laboratory so it can be analyzed for which of the following factors? 1. Antibodies 2. Type of infection 3. Composition of calculus 4. Size and number of calculi 9. Which of the following symptoms indicate acute rejection of a transplanted kidney? 1. Edema, nausea 2. Fever, anorexia 3. Weight gain, pain at graft site 4. Increased WBC count, pain with voiding 10. Adverse reactions of prednisone therapy include which of the following conditions?

113

1. Acne and bleeding gums 2. Sodium retention and constipation 3. Mood swings and increased temperature 4. Increased blood glucose levels and decreased wound healing. 11. The nurse suspects that a client with polyuria is experiencing water diuresis. Which laboratory value suggests water diuresis? 1. High urine specific gravity 2. High urine osmolarity 3. Normal to low urine specific gravity 4. Elevated urine pH 12. A client is diagnosed with prostate cancer. Which test is used to monitor progression of this disease? 1. Serum creatinine 2. Complete blood cell count (CBC) 3. Prostate specific antigen (PSA) 4. Serum potassium 13. a 27-year old client, who became paraplegic after a swimming accident, is experiencing autonomic dysreflexia. Which condition is the most common cause of autonomic dysreflexia? 1. Upper respiratory infection 2. Incontinence 3. Bladder distention 4. Diarrhea 14. When providing discharge teaching for a client with uric acid calculi, the nurse should an instruction to avoid which type of diet? 1. Low-calcium 2. Low-oxalate 3. High-oxalate 4. High-purine 15. The client with urolithiasis has a history of chronic urinary tract infections. The nurse concludes that this client most likely has which of the following types of urinary stones? 1. Calcium oxalate 2. Uric acid 3. Struvite 4. Cystine 16. The nurse is receiving in transfer from the postanesthesia care unit a client who has had a percutaneous ultrasonic lithotripsy for calculuses in the renal pelvis. The nurse anticipates that the clients care will involve monitoring which of the following? 1. Suprapubic tube 2. Urethral stent 3. Nephrostomy tube 4. Jackson-Pratt drain 17. The client is admitted to the ER following a MVA. The client was wearing a lap seat belt when the accident occurred. The client has hematuria and lower abdominal pain. To determine further whether the pain is due to bladder trauma, the nurse asks the client if the pain is referred to which of the following areas? 1. Shoulder 2. Umbilicus 3. Costovertebral angle 4. Hip

114

18. The client complains of fever, perineal pain, and urinary urgency, frequency, and dysuria. To assess whether the clients problem is related to bacterial prostatitis, the nurse would look at the results of the prostate examination, which should reveal that the prostate gland is: 1. Tender, indurated, and warm to the touch 2. Soft and swollen 3. Tender and edematous with ecchymosis 4. Reddened, swollen, and boggy. 19. The nurse is taking the history of a client who has had benign prostatic hyperplasia in the past. To determine whether the client currently is experiencing difficulty, the nurse asks the client about the presence of which of the following early symptoms? 1. Urge incontinence 2. Nocturia 3. Decreased force in the stream of urine 4. Urinary retention 20. The client who has a cold is seen in the emergency room with inability to void. Because the client has a history of BPH, the nurse determines that the client should be questioned about the use of which of the following medications? 1. Diuretics 2. Antibiotics 3. Antitussives 4. Decongestants 21. The nurse is preparing to care for the client following a renal scan. Which of the following would the nurse include in the plan of care? 1. Place the client on radiation precautions for 18 hours 2. Save all urine in a radiation safe container for 18 hours 3. Limit contact with the client to 20 minutes per hour. 4. No special precautions except to wear gloves if in contact with the clients urine. 22. The client passes a urinary stone, and lab analysis of the stone indicates that it is composed of calcium oxalate. Based on this analysis, which of the following would the nurse specifically include in the dietary instructions? 1. Increase intake of meat, fish, plums, and cranberries 2. Avoid citrus fruits and citrus juices 3. Avoid green, leafy vegetables such as spinach. 4. Increase intake of dairy products. 23. The client returns to the nursing unit following a pyelolithotomy for removal of a kidney stone. A Penrose drain is in place. Which of the following would the nurse include on the clients postoperative care? 1. Sterile irrigation of the Penrose drain 2. Frequent dressing changes around the Penrose drain 3. Weighing the dressings 4. Maintaining the clients position on the affected side 24. The nurse is caring for a client following a kidney transplant. The client develops oliguria. Which of the following would the nurse anticipate to be prescribed as the treatment of oliguria? 1. Encourage fluid intake 2. Administration of diuretics 3. Irrigation of foley catheter 4. Restricting fluids 25. A week after kidney transplantation the client develops a temperature of 101, the blood pressure is elevated, and the kidney is tender. The x-ray results the transplanted kidney is enlarged. Based on these assessment findings, the nurse would suspect which of the following?

115

1. Acute rejection 2. Chronic rejection 3. Kidney infection 4. Kidney obstruction 26. The client with BPH undergoes a transurethral resection of the prostate. Postoperatively, the client is receiving continuous bladder irrigations. The nurse assesses the client for signs of transurethral resection syndrome. Which of the following assessment data would indicate the onset of this syndrome? 1. Bradycardia and confusion 2. Tachycardia and diarrhea 3. Decreased urinary output and bladder spasms 4. Increased urinary output and anemia 27. The client is admitted to the hospital with BPH, and a transurethral resection of the prostate is performed. Four hours after surgery the nurse takes the clients VS and empties the urinary drainage bag. Which of the following assessment findings would indicate the need to notify the physician? 1. Red bloody urine 2. Urinary output of 200 ml greater than intake 3. Blood pressure of 100/50 and pulse 130. 4. Pain related to bladder spasms. 28. Which of the following symptoms is the most common clinical finding associated with bladder cancer? 1. Suprapubic pain 2. Dysuria 3. Painless hematuria 4. Urinary retention 29. A client who has been diagnosed with bladder cancer is scheduled for an ileal conduit. Preoperatively, the nurse reinforces the clients understanding of the surgical procedure by explaining that an ileal conduit: 1. Is a temporary procedure that can be reversed later. 2. Diverts urine into the sigmoid colon, where it is expelled through the rectum. 3. Conveys urine from the ureters to a stoma opening in the abdomen. 4. Creates an opening in the bladder that allows urine to drain into an external pouch. 30. After surgery for an ileal conduit, the nurse should closely evaluate the client for the occurrence of which of the following complications related to pelvic surgery? 1. Peritonitis 2. Thrombophlebitis 3. Ascites 4. Inguinal hernia 31. The nurse is assessing the urine of a client who has had an ileal conduit and notes that the urine is yellow with a moderate amount of mucus. Based on the assessment data, which of the following nursing interventions would be most appropriate at this time? 1. Change the appliance bag 2. Notify the physician 3. Obtain a urine specimen for culture 4. Encourage a high fluid intake 32. When teaching the client to care for an ileal conduit, the nurse instructs the client to empty the appliance frequently, primarily to prevent which of the following problems? 1. Rupture of the ileal conduit 2. Interruption of urine production

116

3. Development of odor 4. Separation of the appliance from the skin 33. The client with an ileal conduit will be using a reusable appliance at home. The nurse should teach the client to clean the appliance routinely with what product? 1. Baking soda 2. Soap 3. Hydrogen peroxide 4. Alcohol 34. The nurse is evaluating the discharge teaching for a client who has an ileal conduit. Which of the following statements indicates that the client has correctly understood the teaching? Select all that apply. 1. If I limit my fluid intake I will not have to empty my ostomy pouch as often. 2. I can place an aspirin tablet in my pouch to decrease odor. 3. I can usually keep my ostomy pouch on for 3 to 7 days before changing it. 4. I must use a skin barrier to protect my skin from urine. 5. I should empty my ostomy pouch of urine when it is full. 35. A female client with a urinary diversion tells the nurse, This urinary pouch is embarrassing. Everyone will know that Im not normal. I dont see how I can go out in public anymore. The most appropriate nursing diagnosis for this patient is: 1. Anxiety related to the presence of urinary diversion. 2. Deficient Knowledge about how to care for the urinary diversion. 3. Low Self-Esteem related to feelings of worthlessness 4. Disturbed Body Image related to creation of a urinary diversion. 36. The nurse teaches the client with a urinary diversion to attach the appliance to a standard urine collection bag at night. The most important reason for doing this is to prevent: 1. Urine reflux into the stoma 2. Appliance separation 3. Urine leakage 4. The need to restrict fluids 37. The nurse teaches the client with an ileal conduit measures to prevent a UTI. Which of the following measures would be most effective? 1. Avoid people with respiratory tract infections 2. Maintain a daily fluid intake of 2,000 to 3,000 ml 3. Use sterile technique to change the appliance 4. Irrigate the stoma daily. 38. A client who has been diagnosed with calculi reports that the pain is intermittent and less colicky. Which of the following nursing actions is most important at this time? 1. Report hematuria to the physician 2. Strain the urine carefully 3. Administer meperidine (Demerol) every 3 hours 4. Apply warm compresses to the flank area 39. A client has a ureteral catheter in place after renal surgery. A priority nursing action for care of the ureteral catheter would be to: 1. Irrigate the catheter with 30 ml of normal saline every 8 hours 2. Ensure that the catheter is draining freely 3. Clamp the catheter every 2 hours for 30 minutes. 4. Ensure that the catheter drains at least 30 ml an hour

117

40. Which of the following interventions would be most appropriate for preventing the development of a paralytic ileus in a client who has undergone renal surgery? 1. Encourage the client to ambulate every 2 to 4 hours 2. Offer 3 to 4 ounces of a carbonated beverage periodically. 3. Encourage use of a stool softener 4. Continue intravenous fluid therapy 41. The nurse is conducting a postoperative assessment of a client on the first day after renal surgery. Which of the following findings would be most important for the nurse to report to the physician? 1. Temperature, 99.8 2. Urine output, 20 ml/hour 3. Absence of bowel sounds 4. A 22 inch area of serous sanguineous drainage on the flank dressing. 42. Because a clients renal stone was found to be composed to uric acid, a low-purine, alkaline-ash diet was ordered. Incorporation of which of the following food items into the home diet would indicate that the client understands the necessary diet modifications? 1. Milk, apples, tomatoes, and corn 2. Eggs, spinach, dried peas, and gravy. 3. Salmon, chicken, caviar, and asparagus 4. Grapes, corn, cereals, and liver. 43. Allopurinol (Zyloprim), 200 mg/day, is prescribed for the client with renal calculi to take home. The nurse should teach the client about which of the following side effects of this medication? 1. Retinopathy 2. Maculopapular rash 3. Nasal congestion 4. Dizziness 44. The client has a clinic appointment scheduled 10 days after discharge. Which laboratory finding at that time would indicate that allopurinol (Zyloprim) has had a therapeutic effect? 1. Decreased urinary alkaline phosphatase level 2. Increased urinary calcium excretion 3. Increased serum calcium level 4. Decreased serum uric acid level 45. When developing a plan of care for the client with stress incontinence, the nurse should take into consideration that stress incontinence is best defined as the involuntary loss of urine associated with: 1. A strong urge to urinate 2. Overdistention of the bladder 3. Activities that increase abdominal pressure 4. Obstruction of the urethra 46. Which of the following assessment data would most likely be related to a clients current complaint of stress incontinence? 1. The clients intake of 2 to 3 L of fluid p er day. 2. The clients history of three full-term pregnancies 3. The clients age of 45 years 4. The clients history of competitive swimming 47. The nurse is developing a teaching plan for a client with stress incontinence. Which of the following instructions should be included? 1. Avoid activities that are stressful and upsetting 2. Avoid caffeine and alcohol

118

3. Do not wear a girdle 4. Limit physical exertion 48. A client has urge incontinence. Which of the following signs and symptoms would the nurse expect to find in this client? 1. Inability to empty the bladder 2. Loss of urine when coughing 3. Involuntary urination with minimal warning 4. Frequent dribbling of urine 49. A 72-year old male client is brought to the emergency room by his son. The client is extremely uncomfortable and has been unable to void for the past 12 hours. He has known for some time that he has an enlarged prostate but has wanted to avoid surgery. The best method for the nurse to use when assessing for bladder distention in a male client is to check for: 1. A rounded swelling above the pubis. 2. Dullness in the lower left quadrant 3. Rebound tenderness below the symphysis 4. Urine discharge from the urethral meatus 50. During a clients urinary bladder catheterization, the bladder is emptied gradually. The best rationale for the nurses action is that completely emptying an overdistended bladder at one time tends to cause: 1. Renal failure 2. Abdominal cramping 3. Possible shock 4. Atrophy of bladder musculature 51. The primary reason for taping an indwelling catheter laterally to the thigh of a male client is to: 1. Eliminate pressure at the penoscrotal angle 2. Prevent the catheter from kinking in the urethra 3. Prevent accidental catheter removal 4. Allow the client to turn without kinking the catheter 52. The primary function of the prostate gland is: 1. To store underdeveloped sperm before ejaculation 2. To regulate the acidity and alkalinity of the environment for proper sperm development. 3. To produce a secretion that aids in the nourishment and passage of sperm 4. To secrete a hormone that stimulates the production and maturation of sperm 53. The nurse is reviewing a medication history of a client with BPH. Which medication should be recognized as likely to aggravate BPH? 1. Metformin (Glucophage) 2. Buspirone (BuSpar) 3. Inhaled ipratropium (Atrovent) 4. Ophthalmic timolol (Timoptic) 54. A client is scheduled to undergo a transurethral resection of the prostate gland (TURP). The procedure is to be done under spinal anesthesia. Postoperatively, the nurse should be particularly alert for early signs of: 1. Convulsions 2. Cardiac arrest 3. Renal shutdown 4. Respiratory paralysis 55. A client with BPH is being treated with terazosin (Hytrin) 2 mg at bedtime. The nurse should monitor the clients: 1. Urinary nitrites 2. White blood cell count

119

3. Blood pressure 4. Pulse 56. A client underwent a TURP, and a large three way catheter was inserted in the bladder with continuous bladder irrigation. In which of the following circumstances would the nurse increase the flow rate of the continuous bladder irrigation? 1. When the drainage is continuous but slow 2. When the drainage appears cloudy and dark yellow 3. When the drainage becomes bright red 4. When there is no drainage of urine and irrigating solution 57. A priority nursing diagnosis for the client who is being discharged t home 3 days after a TURP would be: 1. Deficient fluid volume 2. Imbalanced Nutrition: Less than Body Requirements 3. Impaired Tissue Integrity 4. Ineffective Airway Clearance 58. If a clients prostate enlargement is caused by a malignancy, which of the following blood examinations should the nurse anticipate to assess whether metastasis has occurred? 1. Serum creatinine level 2. Serum acid phosphatase level 3. Total nonprotein nitrogen level 4. Endogenous creatinine clearance time 59. Steroids, if used following kidney transplantation would cause which of the following side effects? 1. Alopecia 2. Increase Cholesterol Level 3. Orthostatic Hypotension 4. Increase Blood Glucose Level 60. Mr. Roberto was readmitted to the hospital with acute graft rejection. Which of the following assessment finding would be expected? 1. Hypotension 2. Normal Body Temperature 3. Decreased WBC 4. Elevated BUN and Creatinine

Answers and Rationale


1. Answer: 1. Urine should be strained for calculi and sent to the lab for analysis. Fluid intake of 3 to 4 L is encouraged to flush the urinary tract and prevent further calculi formation. A low-calcium diet is recommended to help prevent the formation of calcium calculi. Ambulation is encouraged to help pass the calculi through gravity. 2. Answer: 4. Cystitis is the most common adverse reaction of clients undergoing radiation therapy; symptoms include dysuria, frequency, urgency, and nocturia. Clients with radiation implants require a private room. Urine of clients with radiation implants for bladder cancer should be sent to the radioisotopes lab for monitoring. It is recommended that fluid intake be increased. 3. Answer: 3. Infections is the major complication to watch for in clients on cyclosporine therapy because its an immunosuppressive drug. Depression may occur posttransplantation but not because of cyclosporine. Hemorrhage is a complication associated with anticoagulant therapy. Peptic ulcer disease is a complication of steroid therapy. 4. Answer: 4. In a client with acute renal graft rejection, evidence of deteriorating renal function is expected. The nurse would see elevated WBC counts and fever because the body is recognizing the graft as foreign and is attempting to fight it. The client would most likely have acute hypertension. 5. Answer: 4. The client undergoing a renal transplantation will need vigilant follow-up care and must adhere to the medical regimen. The client is most likely anuric or oliguric preoperatively, but postoperatively will require close monitoring of urine output to make sure the transplanted kidney is functioning optimally. While the client will always need to be monitored for signs

120

and symptoms of infection, its most important post-op will require close monitoring of urine output to make sure the transplanted kidney is functioning optimally. While the client will always need to be monitored for signs and symptoms of infection, its most important postoperatively due to the immunosuppressant therapy. Rejection can occur postoperatively. 6. Answer: 4. Blood clots and blocked outflow if the urine can increase spasms. The irrigation shouldnt be stopped as long a s the catheter is draining because clots will form. A belladonna and opium suppository should be given to relieve spasms but only after assessment of the drainage. Oral analgesics should be given if the spasms are unrelieved by the belladonna and opium suppository. 7. Answer: 4. Postobstructive diuresis seen in hydronephrosis can cause electrolyte imbalances; lab values must be checked so electrolytes can be replaced as needed. VS should initially be taken every 30 minutes for the first 4 hours and then every 2 hours. Urine output needs to be assessed hourly. The clients weight should be taken daily to assess fluid status more closely. 8. Answer: 3. The calculus should be analyzed for composition to determine appropriate interventions such as dietary restrictions. Calculi dont result in infections. The size and number of calculi arent relevant, and they dont contain antibodies. 9. Answer: 3. Pain at the graft site and weight gain indicates the transplanted kidney isnt functioning and possibly is bein g rejected. Transplant clients usually have edema, anorexia, fever, and nausea before transplantation, so those symptoms may not indicate rejection. 10. Answer: 4. Steroid use tends to increase blood glucose levels, particularly in clients with diabetes and borderline diabetes. Steroids also contribute to poor wound healing and may cause acne, mood swings, and sodium and water retention. Steroids dont affect thermoregulation, bleeding tendencies, or constipation. 11. Answer: 3. Water diuresis causes low urine specific gravity, low urine osmolarity, and a normal to elevated serum sodium level. High specific gravity indicates dehydration. Hypernatremia signals acidosis and shock. Elevated urine pH can result from potassium deficiency, a high-protein diet, or uncontrolled diabetes. 12. Answer: 3. The PSA test is used to monitor prostate cancer progression; higher PSA levels indicate a greater tumor burden. Serum creatinine levels may suggest blockage from an enlarged prostate. CBC is used to diagnose anemia and polycythemia. Serum potassium levels identify hypokalemia and hyperkalemia. 13. Answer: 3. Autonomic dysreflexia is a potentially life-threatening complication of spinal cord injury, occurring from obstruction of the urinary system or bowel. Incontinence and diarrhea dont result in obstruction of the urinary system o r bowel, respectively. An URI could obstruct the respiratory system, but not the urinary or bowel system. 14. Answer: 4. To control uric acid calculi, the client should follow a low-purine diet, which excludes high-purine foods such as organ meats. A low-calcium diet decreases the risk for oxalate renal calculi. Oxalate is an essential amino acid and must be included in the diet. A low-oxalate diet is used to control calcium or oxalate calculi. 15. Answer: 3. Struvite stones commonly are referred to as infection stones because they form in urine that is alkaline and rich in ammonia, such as with a urinary tract infection. Calcium oxalate stones result from increased calcium intake or conditions that raise serum calcium concentrations. Uric acid stones occur in clients with gout. Cystine stones are rare and occur in clients with a genetic defect that results in decreased renal absorption of the amino acid cystine. 16. Answer: 3. A nephrostomy tube is put in place after a percutaneous ultrasonic lithotripsy to treat calculuses in the renal pelvis. The client may also have a foley catheter to drain urine produced by the other kidney. The nurse monitors the drainage from each of these tubes and strains the urine to detect elimination of the calculus fragments. 17. Answer: 1. Bladder trauma or injury is characterized by lower abdominal pain that may radiate to one of the shoulders. Bladder injury pain does not radiate to the umbilicus, CV angle, or hip. 18. Answer: 1. The client with prostatitis has a prostate gland that is swollen and tender but that is also warm to the touch, firm, and indurated. Systemic symptoms include fever with chills, perineal and low back pain, and signs of urinary tract infection (which often accompany the disorder). 19. Answer: 3. Decreased force in the stream of urine is an early sign of BPH. The stream later becomes weak and dribbling. The client then may develop hematuria, frequency, urgency, urge incontinence, and nocturia. If untreated, complete obstruction and urinary retention can occur. 20. Answer: 4. In the client with BPH, episodes of urinary retention can be triggered by certain medications, such as decongestants, anticholinergics, and antidepressants. The client should be questioned about the use of these medications if the

121

client has urinary retention. Retention can also be precipitated by other factors, such as alcoholic beverages, infection, bedrest, and becoming chilled. 21. Answer: 4. No specific precautions are necessary following a renal scan. Urination into a commode is acceptable without risk from the small amount of radioactive material to be excreted. The nurse wears gloves to maintain body secretion precautions. 22. Answer: 3. Oxalate is found in dark green foods such as spinach. Other foods that raise urinary oxalate are rhubarb, strawberries, chocolate, wheat bran, nuts, beets, and tea. 23. Answer: 2. Frequent dressing changes around the Penrose drain is required to protect the skin against breakdown from urinary drainage. If urinary drainage is excessive, an ostomy pouch may be placed over the drain to protect the skin. A Penrose drain is not irrigated. Weighing the dressings is not necessary. Placing the client on the affected side will prevent a free flow of urine through the drain. 24. Answer: 2. To increase urinary output, diuretics and osmotic agents are considered. The client should be monitored closely because fluid overload can cause hypertension, congestive heart failure, and pulmonary edema. Fluid intake would not be encouraged or restricted. Irrigation of the foley catheter will not assist in alleviating this oliguria. 25. Answer: 1. Acute rejection most often occurs in the first 2 weeks after transplant. Clinical manifestations include fever, malaise, elevated WBC count, acute hypertension, graft tenderness, and manifestations of deteriorating renal function. Chronic rejection occurs gradually during a period of months to years. Although kidney infection or obstruction can occur, the symptoms presented in the question do not relate specifically to these disorders. 26. Answer: 1. Transurethral resection syndrome is caused by increased absorption of nonelectrolyte irrigating fluid used during surgery. The client may show signs of cerebral edema and increased intracranial pressure such as increased blood pressure, bradycardia, confusion, disorientation, muscle twitching, visual disturbances, and nausea and vomiting. 27. Answer: 3. Frank bleeding (arterial or venous) may occur during the first few days after surgery. Some hematuria is usual for several days after surgery. A urinary output of 200 ml of greater than intake is adequate. Bladder spasms are expected to occur after surgery. A rapid pulse with a low blood pressure is a potential sign of excessive blood loss. The physician should be notified. 28. Answer: 3. Painless hematuria is the most common clinical finding in bladder cancer. Other symptoms include frequency, dysuria, and urgency, but these are not as common as the hematuria. Suprapubic pain and urinary retention do not occur in bladder cancer. 29. Answer: 3. An ileal conduit is a permanent urinary diversion in which a portion of the ileum is surgically resected and one end of the segment is closed. The ureters are surgically attached to this segment of the ileum, and the open end of the ileum is brought to the skin surface on the abdomen to form the stoma. The client must wear a pouch to collect the urine that continually flows through the conduit. The bladder is removed during the surgical procedure and the ileal conduit is not reversible. Diversion of the urine to the sigmoid colon is called a ureter ileosigmoidostomy. An opening in the bladder that allows urine to drain externally is called a cystostomy. 30. Answer: 2. After pelvic surgery, there is an increased chance of thrombophlebitis owing to the pelvic manipulation that can interfere with circulation and promote venous stasis. Peritonitis is a potential complication of any abdominal surgery, not just pelvic surgery. Ascites is most frequently an indication of liver disease. Inguinal hernia may be caused by an increase in abdominal pressure or a congenital weakness of the abdominal wall; ventral hernia occurs at the site of a previous abdominal surgery. 31. Answer: 4. Mucus is secreted by the intestinal segment used to create the conduit and is a normal occurrence. The client should be encouraged to maintain a large fluid intake to help flush the mucus out of the conduit. Because mucus in the urine is expected, it is not necessary to change the appliance bag or notify the physician. The mucus is not an indication of an infection, so a urine culture is not necessary. 32. Answer: 4. If the appliance becomes too full, it is likely to pull away from the skin completely or to leak urine onto the skin. A full appliance will not rupture the ileal conduit or interrupt urine production. Odor formation has numerous causes. 33. Answer: 2. A reusable appliance should be routinely cleaned with soap and water. 34. Answer: 3, 4. The client with an ileal conduit must learn self-care activities related to care of the stoma and ostomy appliances. The client should be taught to increase fluid intake to about 3,000 ml per day and should not limit intake. Adequate

122

fluid intake helps to flush mucus from the ileal conduit. The ostomy appliance should be changed approximately every 3 to 7 days and whenever a leak develops. A skin barrier is essential to protecting the skin from the irritation of the urine. An aspirin should not be used as a method of odor control because it can be an irritant to the stoma and lead to ulceration. The ostomy pouch should be emptied when it is one-third to one-half full to prevent the weight from pulling the appliance away from the skin. 35. Answer: 4. It is normal for clients to express fears and concerns about the body changes associated with a urinary diversion. Allowing the client time to verbalize concerns in a supportive environment and suggest that she discuss these concerns with people who have successfully adjusted to ostomy surgery can help her begin coping with these changes in a positive manner. Although the client may be anxious about this situation and self-esteem may be diminished, the underlying problem is disturbance in body image. There are no data to support a diagnosis of Deficient Knowledge. 36. Answer: 1. The most important reason for attaching the appliance to a standard urine collection bag at night is to prevent reflux into the stoma and ureters, which can result in infection. Use of a standard collection bag also keeps the appliance from separating from the skin and helps prevent urine leakage from an overly full bag, but the primary purpose is to prevent reflux of urine. A client with a urinary diversion should drink 2000-3000 ml of fluid each day; it would be inappropriate to suggest decreasing fluid intake. 37. Answer: 2. Maintaining a fluid intake of 2,000 to 3,000 ml/day is likely to be effective in preventing UTI. A high fluid intake results in high urine output, which prevents urinary stasis and bacterial growth. Avoiding people with respiratory tract infections will not prevent urinary tract infections. Clean, not sterile, technique is used to change the appliance. An ileal conduit stoma is not irrigated. 38. Answer: 2. Intermittent pain that is less colicky indicates that the calculi may be moving along the urinary tract. Fluids should be encouraged to promote movement, and the urine should be strained to detect passage of the stone. Hematuria is to be expected from the irritation of the stone. Analgesics should be administered when the client needs them, not routinely. Moist heat to the flank area is helpful when renal colic occurs, but it is less necessary as pain is lessened. 39. Answer: 2. The ureteral catheter should drain freely without bleeding at the site. The catheter is rarely irrigated, and any irrigation would be done by the physician. The catheter is never clamped. The clients total urine output (ureteral catheter plus voiding or foley catheter output) should be 30 ml/hour. 40. Answer: 1. Ambulation stimulates peristalsis. A client with paralytic ileus is kept NPO until peristalsis returns. Intravenous fluid infusion is a routine postoperative order that does not have any effect on preventing paralytic ileus. A stool softener will not stimulate peristalsis. 41. Answer: 2. The decrease in urinary output may indicate inadequate renal perfusion and should be reported immediately. Urine output of 30 ml/hour or greater is considered acceptable. A slight elevation in temperature is expected after surgery. Peristalsis returns gradually, usually the second or third day after surgery. Bowel sounds will be absent until then. A small amount of serous sanguineous drainage is to be expected. 42. Answer: 1. Because a high-purine diet contributes to the formation of uric acid, a low-purine diet is advocated. An alkalineash diet is also advocated, because uric acid crystals are more likely to develop in acid urine. Foods that may be eaten as desired in a low-purine diet include milk, all fruits, tomatoes, cereals, and corn. Food allowed on an alkaline-ash diet include milk, fruits (except cranberries, plums, and prunes), and vegetables (especially legumes and green vegetables). Gravy, chicken, and liver are high in purine. 43. Answer: 2. Allopurinol is used to treat renal calculi composed of uric acid. Side effects of allopurinol include drowsiness, maculopapular rash, anemia, abdominal pain, nausea, vomiting, and bone marrow depression. Clients should be instructed to report skin rashes and any unusual bleeding or bruising. Retinopathy, nasal congestion, and dizziness are not side effects of allopurinol. 44. Answer: 4. By inhibiting uric acid synthesis, allopurinol decreases its excretion. The drugs effectiveness is assessed b y evaluating for a decreased serum uric acid concentration. Allopurinol does not alter the level of alkaline phosphatase, not does it affect urinary calcium excretion or the serum calcium level. 45. Answer: 3. Stress incontinence is the involuntary loss of urine during such activities as coughing, sneezing, laughing, or physical exertion. These activities increase abdominal and detrusor pressure. A strong urge to urinate is associated with urge

123

incontinence. Overdistention of the bladder can lead to overflow incontinence. Obstruction of the urethra can lead to urinary retention. 46. Answer: 2. The history of three pregnancies is most likely the cause of the clients current episodes of stress incontinence. The clients fluid intake, age, or history of swimming would not create an increase in intra -abdominal pressure. 47. Answer: 2. Clients with stress incontinence are encouraged to avoid substances such as caffeine and alcohol which are bladder irritants. Emotional stressors do not cause stress incontinence. It is caused most commonly be relaxed pelvic musculature. Wearing girdles is not contraindicated. Although clients may be inclined to limit physical exertion to avoid incontinence episodes, they should be encouraged to seek treatment instead of limiting their activities. 48. Answer: 3. A characteristic of urge incontinence is involuntary urination with little or no warning. The inability to empty the bladder is urinary retention. Loss of urine when coughing occurs with stress incontinence. Frequent dribbling of urine is common in male clients after some types of prostate surgery or may occur in women after the development of vesicovaginal or urethrovaginal fistula. 49. Answer: 1. The best way to assess for a distended bladder in either a male or female client is to check for a rounded swelling above the pubis. The swelling represents the distended bladder rising above the pubis into the abdominal cavity. Dullness does not indicate a distended bladder. The client might experience tenderness or pressure above the symphysis. No urine discharge is expected; the urine flow is blocked by the enlarged prostate. 50. Answer: 3. Rapid emptying of an overdistended bladder may cause hypotension and shock due to the sudden change of pressure within the abdominal viscera. Previously, removing no more than 1,000 ml at one time was the standard of practice, but this is no longer thought to be necessary as long as the overdistended bladder is emptied slowly. 51. Answer: 1. The primary reason for taping an indwelling catheter to a male client soothe penis is held in a lateral position to prevent pressure at the penoscrotal angle. Prolonged pressure at the penoscrotal angle can cause a ureterocutaneous fistula. 52. Answer: 3. The prostate gland is located below the bladder and surrounds the urethra. It serves one primary purpose: to produce a secretion that aids in the nourishment and passage of sperm. 53. Answer: 3. Atrovent is a bronchodilator, and its anticholinergic effects can aggravate urinary retention. Glucophage and BuSpar do not affect the urinary system; timolol does not have a systemic effect. 54. Answer: 4. If paralysis of vasomotor nerves in the upper spinal cord occurs when spinal anesthesia is used, the client is likely to develop respiratory paralysis. Artificial ventilation is required until the effects of the anesthesia subside. Convulsions, cardiac arrest, and renal shutdown are not likely results of spinal anesthesia. 55. Answer: 3. Terazosin (Hytrin) is an antihypertensive drug that is also used in the treatment of BPH. Blood pressure must be monitored to ensure that the client does not develop hypotension, syncope, or postural hypotension. The client should be instructed to change positions slowly. Urinary nitrites, white blood cell count, and pulse rate are not affected by terazosin. 56. Answer: 3. The decision made by the surgeon to insert a catheter after a TURP or prostatectomy depends on the amount of bleeding that is expected after the procedure. During continuous bladder irrigation after a TURP or prostatectomy, the rate at which the solution enters the bladder should be increased when the drainage becomes brighter red. The color indicates the presence of blood. Increasing the flow of irrigating solution helps flush the catheter well so clots do not plug it. There would be no reason to increase the flow rate when the return is continuous or when the return appears cloudy and dark yellow. Increasing the flow would be contraindicated when there is no return of urine and irrigating solution. 57. Answer: 1. Deficient Fluid Volume is a priority diagnosis, because the client needs to drink a large amount of fluid to keep the urine clear. The urine should be almost without color. About 2 weeks after a TURP, when desiccated tissue is sloughed out, a secondary hemorrhage could occur. The client should be instructed to call the surgeon or go to the ED if at any time the urine turns bright red. The client is not specifically at risk for nutritional problems after a TURP. The client is not specifically at risk for nutritional problems after a TURP. The client is not specifically at risk for impaired tissue integrity because there is no external incision, and the client is not specifically at risk for airway problems because the procedure is done after spinal anesthesia. 58. Answer: 2. The most specific examination to determine whether a malignancy extends outside of the prostatic capsule is a study of the serum acid phosphatase level. The level increases when a malignancy has metastasized. The prostate specific antigen (PSA) determination and a digital rectal examination are done when screening for prostate cancer. Serum creatinine level, total

124

nonprotein nitrogen level, and endogenous creatinine clearance time give information about kidney function, not prostate malignancy. 59. Answer: 4. Increased Blood Glucose Level. 60. Answer: 4. Elevated BUN and Creatinine.

Introduction
This is a 30-item NCLEX style questionnaire about the concept of Sleep.

Topics

All about Sleep REM Sleep, Non-REM Sleep Hypnotics and Sedatives

Guidelines
Read each question carefully and choose the best answer. You are given one minute per question. Spend your time wisely! Answers and rationales are given below. Be sure to read them. If you need more clarifications, please direct them to the comments section.

Questions
1. To validate the suspicion that a married male client has sleep apnea the nurse first: 1. Asks the client if he experiences apnea in the middle of the night 2. Questions the spouse if she is awakened by her husbands snoring 3. Places the client on a continuous positive airway pressure (CPAP) device 4. Schedules the client for a sleep test 2. When analgesics are ordered for a client with obstructive sleep apnea (OSA) following surgery, the nurse is most concerned about: 1. Nonsteroidal antiinflammatory drugs (NSAIDs) 2. Opioids 3. Anticonvulsants 4. Antidepressants 5. Adjuvants 3. The nurse finds a client sleep walking down the unit hallway. An appropriate intervention the nurse implements is: 1. Asking the client what he or she is doing and call for help 2. Quietly approaching the client and then loudly calling his or her name 3. Lightly tapping the client on the shoulder and leading him or her back to bed 4. Blocking the hallway with chairs and seating the client 4. The nurse is sure to implement strategies to reduce noise on the unit particularly on the ______ night of admission, when the client is especially sensitive to hospital noises. 1. 1st 2. 2nd 3. 3rd 4. 4th 5. Which of the following medications are the safest to administer to adults needing assistance in falling asleep? 1. Sedatives 2. Hypnotics

125

3. Benzodiazepines 4. Anti-anxiety agents 6. To assist an adult client to sleep better the nurse recommends which of the following? (Select all that apply.) 1. Drinking a glass of wine just before retiring to bed 2. Eating a large meal 1 hour before bedtime 3. Consuming a small glass of warm milk at bedtime 4. Performing mild exercises 30 minutes before going to bed 7. The nurse recognizes that a client is experiencing insomnia when the client reports (select all that apply): 1. Extended time to fall asleep 2. Falling asleep at inappropriate times 3. Difficulty staying asleep 4. Feeling tired after a nights sleep 8. The nurse teaches the mother of a newborn that in order to prevent sudden infant death syndrome (SIDS) the best position to place the baby after nursing is (select all that apply): 1. Prone 2. Side-lying 3. Supine 4. Fowlers 9. When assessing a client for obstructive sleep apnea (OSA), the nurse understands the most common symptom is: 1. Headache 2. Early awakening 3. Impaired reasoning 4. Excessive daytime sleepiness 10. The nurse understands that the most vivid dreaming occurs during: 1. REM sleep 2. Stage 1 NREM 3. Stage 4 NREM 4. Transition period from NREM to REM sleep 11. A client taking a beta adrenergic blockers for HTN can experience interference with sleep patterns such as: 1. Nocturia 2. Increased daytime sleepiness 3. Increased awakening from sleep 4. Increased difficulty falling asleep 12. Narcolepsy can be best explained as: 1. A sudden muscle weakness during exercise 2. Stopping breathing for short intervals during sleep 3. Frequent awakenings during the night 4. An overwhelming wave of sleepiness and falling asleep 13. A nursing measure to promote sleep in school-age children is to: 1. Make sure the room is dark and quiet 2. Encourage evening exercise 3. Encourage television watching 4. Encourage quiet activities prior to bed time. 14. A female client verbalizes that she has been having trouble sleeping and feels wide awake as soon as getting into bed. The nurse recognizes that there are many interventions the promote sleep. Check all that apply. 1. Eat a heavy snack before bedtime 2. Read in bed before shutting out the light

126

3. Leave the bedroom if you are unable to sleep 4. Drink a cup of warm tea with milk at bedtime 5. Exercise in the afternoon rather than the evening 6. Count backwards from 100 to 0 when your mind is racing. 15. A client has a diagnosis of primary insomnia. Before assessing this client, the nurse recalls the numerous causes of this disorder. Select all that apply: 1. Chronic stress 2. Severe anxiety 3. Generalized pain 4. Excessive caffeine 5. Chronic depression 6. Environmental noise 16. A hospitalized client is prescribed chloral hydrate (Noctec). The nurse includes which action in the plan of care? 1. Monitor apical heart rate every 2 hours 2. Monitor blood pressure every 4 hours 3. Instruct the client to call for ambulation assistance 4. Clear a path to the bathroom at bedtime. 17. Select all that apply to the use of barbiturates in treating insomnia: 1. Barbiturates deprive people of NREM sleep 2. Barbiturates deprive people of REM sleep 3. When the barbiturates are discontinued, the NREM sleep increases. 4. When the barbiturates are discontinued, the REM sleep increases. 5. Nightmares are often an adverse effect when discontinuing barbiturates. 18. Select all that apply that is appropriate when there is a benzodiazepine overdose: 1. Administration of syrup of ipecac 2. Gastric lavage 3. Activated charcoal and a saline cathartic 4. Hemodialysis 5. Administration of Flumazenil 19. A patient is admitted to the emergency department with an overdose of a benzodiazepine. The nurse immediately prepares to administer which of the following antidotes from the emergency drug cart? 1. naloxone (Narcan) 2. naltrexone (ReVia) 3. nalmefene (Revex) 4. flumazenil (Romazicon) 20. Older adults who take long-acting sedatives or hypnotics are likely to experience: 1. Hallucinations 2. Ataxia 3. Alertness 4. Dyspnea 21. Which nursing diagnosis is appropriate for a patient who has received a sedative-hypnotic agent? 1. Alteration in tissue perfusion 2. Fluid volume excess 3. Risk for injury 4. Risk for infection 22. A patient is admitted to the emergency department with an overdose of a barbiturate. The nurse immediately prepares to administer which of the following from the emergency drug cart?

127

1. naloxone HCl (Narcan 2. activated charcoal 3. flumazenil (Romazicon) 4. ipecac syrup 23. During patient teaching, the nurse explains the difference between a sedative and hypnotic by stating: 1. Sedatives are much stronger than hypnotic drugs and should only be used for short periods of time. 2. Sedative drugs induce sleep, whereas hypnotic drugs induce a state of hypnosis. 3. Most drugs produce sedation at low doses and sleep (the hypnotic effect) at higher doses. 4. There really is no difference; the terms are used interchangeably. 24. The patients chart notes the administration of dantrolene (Dantrium) immediately postoperatively. The nurse suspects that the patient experienced: 1. Delirium tremens 2. Malignant hyperthermia 3. A tonic-clonic seizure 4. Respiratory arrest 25. Which of the following is an important nursing action for the administration of a benzodiazepine as a sedativehypnotic agent? 1. Use IM dosage forms for longer duration 2. Administer safely with other CNS depressants for insomnia 3. Monitor geriatric patients for the common occurrence of paradoxical reactions. 4. Evaluate for physical dependence that occurs within 48 hours of beginning the drug. 26. Pediatric and geriatric patients often react with more sensitivity to CNS depressants. This type of sensitivity manifests itself in the development of which type of reaction? 1. Idiopathic 2. Teratogenic 3. Paradoxical 4. Psychogenic 27. Which of the following is an appropriate nursing intervention for patients who are receiving CNS depressants? 1. Prevent any activity within the hospital setting while on oral muscle relaxants 2. Make sure that the patient knows that sedation should be minimal with these agents. 3. Cardiovascular stimulation, a common side effect, would lead to hypertension 4. Make sure the patients call light is close by in case of the need for assistance with activities. 28. Which of the following conditions characterizes rapid eye movement (REM) sleep? 1. Disorientation and disorganized thinking 2. Jerky limb movements and position changes 3. Pulse rate slowed by 5 to 10 beats/minute 4. Highly active brain and physiological activity levels. 29. Which of the following sleep disorders is the most prevalent? 1. Hypersomnia 2. Insomnia 3. Parasomnia 4. Sleep-awake schedule disturbance. 30. Which of the following substances is a natural hormone produced by the pineal gland that induces sleep? 1. Amphetamine 2. Melatonin 3. Methylphenidate 4. Pemoline

128

Answers and Rationale


1. Answer: 4. (2- Although this is a diagnostic tool, the first thing the nurse would do is question the spouse. This may lead to determining whether more tests are needed). 2. Answer: 2. Clients with obstructive sleep apnea are particularly sensitive to opioids. Thus the risk of respiratory depression is increased. The nurse must recognize that clients with OSA should start out receiving very low doses of opioids. 3. Answer: 3. The nurse should not startle the client but should gently awaken the client and lead him or her back to bed. 4. Answer: 1. The client is most sensitive to noise in the hospital setting the first night because everything is new. This represents sensory overload, which interferes with sleep and decreases rapid eye movement (REM) as well as total sleep time. 5. Answer: 3. The group of drugs that are the safest are the benzodiazepines. They facilitate the action of the neurons in the central nervous system (CNS) that suppress responsiveness to stimulation, therefore decreasing levels of arousal. 6. Answer: 3. A small glass of milk relaxes the body and promotes sleep. 7. Answer: 1, 3, and 4. These symptoms are often reported by clients with insomnia. Clients report nonrestorative sleep. Arising once at night to urinate (nocturia) is not in and of itself insomnia. 8. Answer: 2 and 3. Research demonstrate that the occurrence of SIDS is reduced with these two positions. 9. Answer: 4. Excessive daytime sleepiness is the most common complaint of people with OSA. Persons with severe OSA may report taking daytime naps and experiencing a disruption in their daily activities because of sleepiness. 10. Answer: 1. Although dreams occur during both NREM and REM sleep, the dreams of REM sleep are more vivid and elaborate and are believed to be functionally important to learning, memory processing, and adaptation to stress. 11. Answer: 2. Beta Blockers can cause nightmares, insomnia, and awakenings from sleep. 12. Answer: 4. Narcolepsy is a dysfunction of mechanisms that regulate the sleep and wake states. Excessive daytime sleepiness is the most common complaint associated with this disorder. During the day a person may suddenly feel an overwhelming wave of sleepiness and fall asleep; REM sleep can occur within 15 minutes of falling asleep. 13. Answer: 4. The amount of sleep needed during the school years is individualized because of varying states of activities and levels of health. A 6-year old averages 11-12 hours of sleep nightly, whereas an 11-year old sleeps about 9-10 hours. The 6- or 7year old can usually be persuaded to go to bed by encouraging quiet activities. 14. Answer: 3, 5, and 6. Lying in bed when one is unable to sleep increases frustration and anxiety which further impede sleep; other activities, such as reading or watching television, should not be conducted in bed. Counting backwards requires minimal concentration but it is enough to interfere with thoughts that distract a person from falling asleep. 15. Answer: 1, 4, and 6. Acute or primary insomnia is caused by emotional or physical discomfort not caused by the direct physiologic effects of a substance or a medical condition. Excessive caffeine intake is an example of disruptive sleep hygiene; caffeine is a stimulant that inhibits sleep. Environmental noise causes physical and/or emotional and therefore is related to primary insomnia. 16. Answer: 3. Chloral hydrate is a sedative. This medication does not affect cardiac function. Blood pressure changes are not significant with the use of this medication. A client should call for assistance to the bathroom at night. Additionally, the client may experience residual daytime sedation; therefore, the nurse should instruct the client to call for ambulation assistance during the daytime hours. 17. Answer: 2, 4, and 5. Barbiturates deprive people of REM sleep. When the barbiturate is stopped and REM sleep once again occurs, a rebound phenomenon occurs. During this phenomenon, the persons dream time constitutes a larger percentage of the total sleep pattern, and the dreams are often nightmares. 18. Answer: 2, 3, and 5. If ingestion is recent, decontamination of the GI system is indicated. The administration of syrup of ipecac is contraindicated because of aspiration risks related to sedation. Gastric lavage is generally the best and most effective means of gastric decontamination. Activated charcoal and a saline cathartic may be administered to remove any remaining drug. Hemodialysis is not useful in the treatment of benzodiazepine overdose. Flumazenil can be used to acutely reverse the sedative effects of benzodiazepines, though this is normally done only in cases of extreme overdose or sedation. 19. Answer: 4. Flumazenil is the antidote for benzodiazepine overdoses. 20. Answer: 2. If longer-acting barbiturates are used in older adults, these clients may experience daytime sedation, ataxia, and memory deficits.

129

21. Answer: 3. Sedative-hypnotics cause CNS depression, putting the patient at risk for injury. 22. Answer: 2. There is no antidote for barbiturates. The use of activated charcoal absorbs any drug in the GI tract, preventing absorption. 23. Answer: 3. Many drugs have both sedative and hypnotic properties, with the sedative properties evident at low doses and the hypnotic properties demonstrated at larger doses. 24. Answer: 2. Dantrolene is a direct-acting musculoskeletal muscle relaxant and is the drug of choice to treat malignant hyperthermia, a complication of generalized anesthesia (remember intraoperative nursing???) 25. Answer: 3 26. Answer: 3 27. Answer: 4 28. Answer: 4.Highly active brain and physiological activity levels characterize REM stage. Stages 3 and 4 of NREM sleep are characterized by disorientation and disorganization, During REM sleep, the body movement ceases except for the eyes. The pulse rate slows by 5-10 beats/minute during NREM sleep, not REM sleep. 29. Answer: 2. Approximately 1/3 of American adults have some type of sleep disorder, and insomnia is the most common. 30. Answer: 2. Melatonin is a natural hormone that induces sleep. All the others are medications classified as stimulants.

Introduction
Take the last part of our Nursing Pharmacology exam series with this 30-item exam. This exam focuses more on providing interventions when adverse effects occurs.

Topics

Drug administration Interventions for adverse effects

Guidelines
Read each question carefully. Choose the best answer. You are given 1 minute and 20 seconds for each question. Answers & Rationales are given below. Be sure to read them!

Questions
1. Walter, a teenage patient is admitted to the hospital because of acetaminophen (Tylenol) overdose. Overdoses of acetaminophen can precipitate life-threatening abnormalities in which of the following organs? a. Lungs b. Liver c. Kidney d. Adrenal Glands 2. A contraindication for topical corticosteroid usage in a male patient with atopic dermatitis (eczema) is: a. Parasite infection. b. Viral infection. c. Bacterial infection. d. Spirochete infection. 3. In infants and children, the side effects of first generation over-the-counter (OTC) antihistamines, such as diphenhydramine (Benadryl) and hydroxyzine (Atarax) include: a. Reyes syndrome. b. Cholinergic effects.

130

c. Paradoxical CNS stimulation. d. Nausea and diarrhea. 4. Reyes syndrome, a potentially fatal illness associated with liver failure and encephalopathy is associ ated with the administration of which over-the-counter (OTC) medication? a. acetaminophen (Tylenol) b. ibuprofen (Motrin) c. aspirin d. brompheniramine/psudoephedrine (Dimetapp) 5. The nurse is aware that the patients who are allergic to intravenous contrast media are usually also allergic to which of the following products? a. Eggs b. Shellfish c. Soy d. acidic fruits 6. A 13-month-old child recently arrived in the United States from a foreign country with his parents and needs childhood immunizations. His mother reports that he is allergic to eggs. Upon further questioning, you determine that the allergy to eggs is anaphylaxis. Which of the following vaccines should he not receive? a. Hepatitis B b. inactivated polio c. diphtheria, acellular pertussis, tetanus (DTaP) d. mumps, measles, rubella (MMR) 7. The cell and Coombs classification system categorizes allergic reactions and is useful in describing and classifying patient reactions to drugs. Type I reactions are immediate hypersensitivity reactions and are mediated by: a. immunoglobulin E (IgE). b. immunoglobulin G (IgG). c. immunoglobulin A (IgA). d. immunoglobulin M (IgM). 8. Drugs can cause adverse events in a patient. Bone marrow toxicity is one of the most frequent types of drug-induced toxicity. The most serious form of bone marrow toxicity is: a. aplastic anemia. b. thrombocytosis. c. leukocytosis. d. granulocytosis. 9. Serious adverse effects of oral contraceptives include: a. Increase in skin oil followed by acne. b. Headache and dizziness. c. Early or mid-cycle bleeding. d. Thromboembolic complications. 10. The most serious adverse effect of Alprostadil (Prostin VR pediatric injection) administration in neonates is: a. Apnea. b. Bleeding tendencies. c. Hypotension. d. Pyrexia. 11. Mandy, a patient calls the clinic today because he is taking atrovastatin (Lipitor) to treat his high cholesterol and is having pain in both of his legs. You instruct him to: a. Stop taking the drug and make an appointment to be seen next week. b. Continue taking the drug and make an appointment to be seen next week.

131

c. Stop taking the drug and come to the clinic to be seen today. d. Walk for at least 30 minutes and call if symptoms continue. 12. Which of the following adverse effects is associated with levothyroxine (Synthroid) therapy? a. Tachycardia b. Bradycardia c. Hypotension d. Constipation 13. Which of the following adverse effects is specific to the biguanide diabetic drug metformin (Glucophage) therapy? a. Hypoglycemia b. GI distress c. Lactic acidosis d. Somulence 14. The most serious adverse effect of tricyclic antidepressant (TCA) overdose is: a. Seizures. b. Hyperpyrexia. c. Metabolic acidosis. d. Cardiac arrhythmias. 15. The nurse is aware that the following solutions is routinely used to flush an IV device before and after the administration of blood to a patient is: a. 0.9 percent sodium chloride b. 5 percent dextrose in water solution c. Sterile water d. Heparin sodium 16. Cris asks the nurse whether all donor blood products are cross-matched with the recipient to prevent a transfusion reaction. Which of the following always require cross-matching? a. packed red blood cells b. platelets c. plasma d. granulocytes 17. A month after receiving a blood transfusion an immunocompromised male patient develops fever, liver abnormalities, a rash, and diarrhea. The nurse would suspect this patient has: a. Nothing related to the blood transfusion. b. Graft-versus-host disease (GVHD). c. Myelosuppression. d. An allergic response to a recent medication. 18. Jonas comes into the local blood donation center. He says he is here to donate platelets only today. The nurse knows this process is called: a. Directed donation. b. Autologous donation. c. Allogenic donation. d. Apheresis. 19. Nurse Bryan knows that the age group that uses the most units of blood and blood products is: a. Premature infants. b. Children ages 1-20 years. c. Adults ages 21-64 years. d. The elderly above age 65 years.

132

20. A child is admitted with a serious infection. After two days of antibiotics, he is severely neutropenic. The physician orders granulocyte transfusions for the next four days. The mother asks the nurse why? The nurse responds: a. This is the only treatment left to offer the child. b. This therapy is fast and reliable in treating infections in children. c. The physician will have to explain his rationale to you. d. Granulocyte transfusions replenish the low white blood cells until the body can produce its own. 21. A neighbor tells nurse Maureen he has to have surgery and is reluctant to have any blood product transfusions because of a fear of contracting an infection. He asks the nurse what are his options. The nurse teaches the person that the safest blood product is: a. An allogenic product. b. A directed donation product. c. An autologous product. d. A cross-matched product. 22. A severely immunocompromised female patient requires a blood transfusion. To prevent GVHD, the physician will order: a. Diphenhydramine hydrochloride (Benadryl). b. The transfusion to be administered slowly over several hours. c. Irradiation of the donor blood. d. Acetaminophen (Tylenol). 23. Louie who is to receive a blood transfusion asks the nurse what is the most common type of infection he could receive from the transfusion. The nurse teaches him that approximately 1 in 250,000 patients contract: a. Human immunodeficiency disease (HIV). b. Hepatitis C infection. c. Hepatitis B infection. d. West Nile viral disease. 24. A male patient with blood type AB, Rh factor positive needs a blood transfusion. The Transfusion Service (blood bank) sends type O, Rh factor negative blood to the unit for the nurse to infuse into this patient. The nurse knows that: a. This donor blood is incompatible with the patients blood. b. Premedicating the patient with diphenhydramine hydrochloride (Benadryl) and acetaminophen (Tylenol) will prevent any transfusion reactions or side effects. c. This is a compatible match. d. The patient is at minimal risk receiving this product since it is the first time he has been transfused with type O, Rh negative blood. 25. Dr. Rodriguez orders 250 milliliters of packed red blood cells (RBC) for a patient. This therapy is administered for treatment of: a. Thrombocytopenia. b. Anemia. c. Leukopenia. d. Hypoalbuminemia. 26. A female patient needs a whole blood transfusion. In order for transfusion services (the blood bank) to prepare the correct product a sample of the patients blood must be obtained for: a. A complete blood count and differential. b. A blood type and cross-match. c. A blood culture and sensitivity. d. A blood type and antibody screen. 27. A male patient needs to receive a unit of whole blood. What type of intravenous (IV) device should the nurse consider starting?

133

a. A small catheter to decrease patient discomfort b. The type of IV device the patient has had in the past, which worked well c. A large bore catheter d. The type of device the physician prefers 28. Dr. Smith orders a gram of human salt poor albumin product for a patient. The product is available in a 50 milliliter vial with a concentration of 25 percent. What dosage will the nurse administer? a. The nurse should use the entire 50 milliliter vial. b. The nurse should determine the volume to administer from the physician. c. This concentration of product should not be used. d. The nurse will administer 4 milliliters. 29. Central venous access devices (CVADs) are frequently utilized to administer chemotherapy. What is a distinct advantage of using the CVAD for chemotherapeutic agent administration? a. CVADs are less expensive than a peripheral IV. b. Once a week administration is possible. c. Caustic agents in small veins can be avoided. d. The patient or his family can administer the drug at home. 30. A female patients central venous access device (CVAD) becomes infected. Why would the physician order antibiotics to be given through the line rather than through a peripheral IV line? a. To prevent infiltration of the peripheral line b. To reduce the pain and discomfort associated with antibiotic administration in a small vein c. To lessen the chance of an allergic reaction to the antibiotic d. To attempt to sterilize the catheter and prevent having to remove it

Answers & Rationale


Here are the answers and rationale for this exam. Counter check your answers to those below and tell us your scores. If you have any disputes or need more clarification to a certain question, please direct them to the comments section. 1. Answer B. Acetaminophen is extensively metabolized by pathways in the liver. Toxic doses of acetaminophen deplete hepatic glutathione, resulting in accumulation of the intermediate agent, quinine, which leads to hepatic necrosis. Prolonged use of acetaminophen may result in an increased risk of renal dysfunction, but a single overdose does not precipitate lifethreatening problems in the respiratory system, renal system, or adrenal glands. 2. Answer B. Topical agents produce a localized, rather than systemic effect. When treating atopic dermatitis with a steroidal preparation, the site is vulnerable to invasion by organisms. Viruses, such as herpes simplex or varicella-zoster, present a risk of disseminated infection. Educate the patient using topical corticosteroids to avoid crowds or people known to have infections and to report even minor signs of an infection. Topical corticosteroid usage results in little danger of concurrent infection with these agents. 3. Answer C. Typically, first generation OTC antihistamines have a sedating effect because of passage into the CNS. However, in some individuals, especially infants and children, paradoxical CNS stimulation occurs and is manifested by excitement, euphoria, restlessness, and confusion. For this reason, use of first generation OTC antihistamines has declined, and second generation product usage has increased. Reyes syndrome is a systemic response to a virus. First generation OTC antihistamines do not exhibit a cholinergic effect. Nausea and diarrhea are uncommon when first generation OTC antihistamines are taken. 4. Answer C. Virus-infected children who are given aspirin to manage pain, fever, and inflammation are at an increased risk of developing Reyes syndrome. Use of acetaminophen has not been associated with Reyes syndrome and can be safely given to patients with fever due to viral illnesses. Ibuprofen adverse effects include GI irritation and bleeding, and in toxic doses, both renal and hepatic failure are reported. However, ibuprofen has not been associated with the onset of Reyes disease. Brompheniramine/psudoephedrine contains a first generation OTC antihistamine and a decongestant. Neither agent has been associated with the development of Reyes syndrome.

134

5.

Answer B. Some types of contrast media contain iodine as an ingredient. Shellfish also contain significant amounts of iodine. Therefore, a patient who is allergic to iodine will exhibit an allergic response to both iodine containing contrast media and shellfish. These products do not contain iodine.

6.

Answer D. The measles portion of the MMR vaccine is grown in chick embryo cells. The current MMR vaccine does not contain a significant amount of egg proteins, and even children with dramatic egg allergies are extremely unlikely to have an anaphylactic reaction. However, patients that do respond to egg contact with anaphylaxis should be in a medically controlled setting where full resuscitation efforts can be administered if anaphylaxis results. The vaccines in options a,b and c do not contain egg protein.

7.

Answer A. IgE, the least common serum immunoglobulin (Ig) binds very tightly to receptors on basophils and mast cells and is involved in allergic reactions. Binding of the allergen to the IgE on the cells results in the release of various pharmacological mediators that result in allergic symptoms. IgG is the major Ig (75 percent of serum Ig is IgG). Most versatile Ig because it is capable of carrying out all of the functions of Ig molecules. IgG is the only class of Ig that crosses the placenta. It is an opsonin, a substance that enhances phagocytosis. IgA, the second most common serum Ig is found in secretions (tears, saliva, colostrum, and mucus). It is important in local (mucosal) immunity. IgM, the third most common serum Ig, is the first Ig to be made by the fetus and the first Ig to be made by a virgin B cell when it is stimulated by antigen. IgM antibodies are very efficient in leading to the lysis of microorganisms.

8.

Answer A. Aplastic anemia is the result of a hypersensitivity reaction and is often irreversible. It leads to pancytopenia, a severe decrease in all cell types: red blood cells, white blood cells, and platelets. A reduced number of red blood cells causes hemoglobin to drop. A reduced number of white blood cells make the patient susceptible to infection. And, a reduced number of platelets cause the blood not to clot as easily. Treatment for mild cases is supportive. Transfusions may be necessary. Severe cases require a bone marrow transplant. Option 2 is an elevated platelet count. Option 3 is an elevated white count. Option 4 is an elevated granulocyte count. A granulocyte is a type of white blood cell.

9.

Answer D. Oral contraceptives have been associated with an increased risk of stroke, myocardial infarction, and deep vein thrombosis. These risks are increased in women who smoke. Increased skin oil and acne are effects of progestin excess. Headache and dizziness are effects of estrogen excess. Early or mid-cycle bleeding are effects of estrogen deficiency.

10. Answer A. All items are adverse reactions of the drug. However, apnea appearing during the first hour of drug infusion occurs in 10-12 percent of neonates with congenital heart defects. Clinicians deciding to utilize alprostadil must be prepared to intubate and mechanically ventilate the infant. Careful monitoring for apnea or respiratory depression is mandatory. In some institutions, elective intubation occurs prior to initiation of the medication. 11. Answer C. Muscle aches, soreness, and weakness may be early signs of myopathy such as rhabdomyolysis associated with the HMG-CoA reducatase class of antilipemic agents. This patient will need an immediate evaluation to rule out myopathy. Additional doses may exacerbate the problem. Exercise will not reverse myopathy and delays diagnosis. 12. Answer A. Levothyroxine, especially in higher doses, can induce hyperthyroid-like symptoms including tachycardia. An agent that increases the basal metabolic rate would not be expected to induce a slow heart rate. Hypotension would be a side effect of bradycardia. Constipation is a symptom of hypothyroid disease. 13. Answer C. Lactic acidosis is the most dangerous adverse effect of metformin administration with death resulting in approximately 50 percent of individuals who develop lactic acidosis while on this drug. Metformin does not induce insulin production; thus, administration does not result in hypoglycemic events. Some nausea, vomiting, and diarrhea may develop but is usually not severe. NVD is not specific for metformin. Metformin does not induce sleepiness. 14. Answer D. Excessive ingestion of TCAs result in life-threatening wide QRS complex tachycardia. TCA overdose can induce seizures, but they are typically not life-threatening. TCAs do not cause an elevation in body temperature. TCAs do not cause metabolic acidosis. 15. Answer A. 0.9 percent sodium chloride is normal saline. This solution has the same osmolarity as blood. Its use prevents red cell lysis. The solutions given in options 2 and 3 are hypotonic solutions and can cause red cell lysis. The solution in option 4 may anticoagulate the patient and result in bleeding. 16. Answer A. Red blood cells contain antigens and antibodies that must be matched between donor and recipient. The blood products in options 2-4 do not contain red cells. Thus, they require no cross-match.

135

17. Answer B. GVHD occurs when white blood cells in donor blood attack the tissues of an immunocompromised recipient. This process can occur within a month of the transfusion. Options 1 and 4 may be a thought, but the nurse must remember that immunocompromised transfusion recipients are at risk for GVHD. 18. Answer D. The process of apheresis involves removal of whole blood from a donor. Within an instrument that is essentially designed as a centrifuge, the components of whole blood are separated. One of the separated portions is then withdrawn, and the remaining components are retransfused into the donor. Directed donation is collected from a blood donor other than the recipient, but the donor is known to the recipient and is usually a family member or friend. Autologous donation is the collection and reinfusion of the patients own blood. Allogenic donation is collected from a blood donor other than the recipient. 19. Answer D. People older than 65 years use 43 percent of donated blood. This number is expected to increase as the population ages. 20. Answer D. Granulocyte (neutrophil) replacement therapy is given until the patients blood values are normal and he is able to fight the infection himself. Options 1 and 3 are not therapeutic responses. The treatment in option 2 takes days and is not always able to prevent morbidity and mortality. 21. Answer C. This process is the collection and reinfusion of the patients own blood. It is rec ommended by the American Medical Associations Council on Scientific Affairs as the safest product since it eliminates recipient incompatibility and infection. The product in option 1 is collected from a blood donor other than the recipient. The process in option 2 is also collected from a blood donor other than the recipient, but the donor is known to the recipient and is usually a family member or friend. Cross-matching significantly enhances compatibility. It does not detect infection. 22. Answer C. This process eliminates white blood cell functioning, thus, preventing GVHD. Diphenhydramine HCl is an antihistamine. Its use prior to a blood transfusion decreases the likelihood of a transfusion reaction. Option 2 will not pr event GVHD. Use of acetaminophen prevents and treats the common side effects of blood administration caused by the presence of white blood cells in the transfusion product: fever, headache, and chills. 23. Answer C. Hepatitis B is the most common infection spread via blood transfusion. Donors are screened by a questionnaire that includes symptoms. The donated blood is also tested for infection. The risk of infection with the agents in options 2 and 3 has decreased to approximately 1 in 2 million secondary to donor questioning and donor blood testing. The incidence of West Nile viral transmission is unknown, but donor infection is still relatively rare. 24. Answer C. Type O, Rh negative blood has none of the major antigens and is safely administered to patients of all blood types. It is also known as the universal donor. Premedicating with these agents will not prevent a major transfusion reaction if the blood type and Rh factors of the donor blood are incompatible with the recipients blood. 25. Answer B. A red blood cell transfusion is used to correct anemia in patients in which the low red blood cell count must be rapidly corrected. RBC transfusion will not correct a low platelet count. RBC transfusion will not correct a low white blood cell count. Packed RBCs contain very little plasma and, thus, only a small amount of albumin. This amount will not correct low albumin levels. 26. Answer B. This is needed to utilize the correct type of donor blood and to match the donor product with the patient. Incompatible matches would result in severe adverse events and possible death. The tests in options 1 and 3 are unnecessary. The test in option 4 is utilized to determine the patients blood type and presence of antibodies to blood antigens. It does not determine donor blood compatibility with the patient. 27. Answer C. Large bore catheters prevent damage to blood components and are less likely to develop clotting problems than a small bore catheter. The nurse should determine the correct device without asking the patient what type has been used before or asking the physician which type he prefers and start the IV. 28. Answer D. A 25 percent solution contains one quarter of a gram per milliliter. Thus, the nurse will administer 4 milliliters to provide a complete gram of albumin. The volume in option 1 would provide 12.5 grams of albumin. The nurse should determine the volume. It is unnecessary to seek the answer from the physician. A 25 percent solution is an acceptable product and can safely be used. 29. Answer C. Many chemotherapeutic drugs are vesicants (highly active corrosive materials that can produce tissue damage even in low concentrations). Extravasations of a vesicant can result in significant tissue necrosis. Administration into a large

136

vein is optimal. CVADs are more expensive than a peripheral IV. Dosing depends on the drug. IV chemotherapeutic agents are not administered at home. They are given in an outpatient or clinic setting if not given during hospitalization. 30. Answer D. Microorganisms that infect CVADs are often coagulase-negative staphylococci, which can be eliminated by antibiotic administration through the catheter. If unsuccessful in eliminating the microorganism, the CVAD must be removed. CVAD use lessens the need for peripheral IV lines and, thus, the risk of infiltration. In this case however, the antibiotics are given to eradicate microorganisms from the CVAD. CVAD use has this effect, but in this case, the antibiotics are given through the CVAD to eliminate the infective agent. The third option would not occur.

Introduction
This is the third part of our Nursing Pharmacology exam series which focuses more on administration of various drugs. Take our 30-item exam that will review you on the different concepts of drug administration.

Topics

Drug administration Interventions for adverse effects

Guidelines
Read each question carefully. Choose the best answer. You are given 1 minute and 20 seconds for each question. Answers & Rationales are given below. Be sure to read them!

Questions
1. An infection in a central venous access device is not eliminated by giving antibiotics through the catheter. How would bacterial glycocalyx contribute to this? a. It protects the bacteria from antibiotic and immunologic destruction. b. Glycocalyx neutralizes the antibiotic rendering it ineffective. c. It competes with the antibiotic for binding sites on the microbe. d. Glycocalyx provides nutrients for microbial growth. 2. Central venous access devices are beneficial in pediatric therapy because: a. They dont frighten children. b. Use of the arms is not restricted. c. They cannot be dislodged. d. They are difficult to see. 3. How can central venous access devices (CVADs) be of value in a patient receiving chemotherapy who has stomatitis and severe diarrhea? a. The chemotherapy can be rapidly completed allowing the stomatitis and diarrhea to resolve. b. Crystalloid can be administered to prevent dehydration. c. Concentrated hyperalimentation fluid can be administered through the CVAD. d. The chemotherapy dose can be reduced. 4. Some central venous access devices (CVAD) have more than one lumen. These multi lumen catheters: a. Have an increased risk of infiltration. b. Only work a short while because the small bore clots off. c. Are beneficial to patient care but are prohibitively expensive. d. Allow different medications or solutions to be administered simultaneously. 5. Some institutions will not infuse a fat emulsion, such as Intralipid, into central venous access devices (CVAD) because:

137

a. Lipid residue may accumulate in the CVAD and occlude the catheter. b. If the catheter clogs, there is no treatment other than removal and replacement. c. Lipids are necessary only in the most extreme cases to prevent essential fatty acid (EFA) deficiency. d. Fat emulsions are very caustic. 6. A male patient needs a percutaneously inserted central catheter (PICC) for prolonged IV therapy. He knows it can be inserted without going to the operating room. He mentions that, at least the doctor wont be wearing surgical garb, will he? How wil l the nurse answer the patient? a. You are correct. It is a minor procedure performed on the unit and does not necessitate surgical attire. b. To decrease the risk of infection, the doctor inserting the PICC will wear a cap, mask, and sterile gown and gloves. c. It depends on the doctors preference. d. Most doctors only wear sterile gloves, not a cap, mask, or sterile gown. 7. A male patient is to receive a percutaneously inserted central catheter (PICC). He asks the nurse whether the insertion will hurt. How will the nurse reply? a. You will have general anesthesia so you wont feel anything. b. It will be inserted rapidly, and any discomfort is fleeting. c. The insertion site will be anesthetized. Threading the catheter through the vein is not painful. d. You will receive sedation prior to the procedure. 8. What volume of air can safely be infused into a patient with a central venous access device (CVAD)? a. It is dependent on the patients weight and height. b. Air entering the patient through a CVAD will follow circulation to the lungs where it will be absorbed and cause no problems. c. It is dependent on comorbidities such as asthma or chronic obstructive lung disease. d. None. 9. Kent a new staff nurse asks her preceptor nurse how to obtain a blood sample from a patient with a portacath device. The preceptor nurse teaches the new staff nurse: a. The sample will be withdrawn into a syringe attached to the portacath needle and then placed into a vacutainer. b. Portacath devices are not used to obtain blood samples because of the risk of clot formation. c. The vacutainer will be attached to the portacath needle to obtain a direct sample. d. Any needle and syringe may be utilized to obtain the sample. 10. What is the purpose of tunneling (inserting the catheter 2 -4 inches under the skin) when the surgeon inserts a Hickman central catheter device? Tunneling: a. Increases the patients comfort level. b. Decreases the risk of infection. c. Prevents the patients clothes from having contact with the catheter d. Makes the catheter less visible to other people. 11. The primary complication of a central venous access device (CVAD) is: a. Thrombus formation in the vein. b. Pain and discomfort. c. Infection. d. Occlusion of the catheter as the result of an intra-lumen clot. 12. Nurse Blessy is doing some patient education related to a patients central venous access device. Which of the following statements will the nurse make to the patient? a. These type of devices are essentially risk free. b. These devices seldom work for more than a week or two necessitating replacement. c. The dressing should only the changed by your doctor. d. Heparin in instilled into the lumen of the catheter to decrease the risk of clotting. 13. The chemotherapeutic DNA alkylating agents such as nitrogen mustards are effective because they:

138

a. Cross-link DNA strands with covalent bonds between alkyl groups on the drug and guanine bases on DNA. b. Have few, if any, side effects. c. Are used to treat multiple types of cancer. d. Are cell cycle-specific agents. 14. Hormonal agents are used to treat some cancers. An example would be: a. Thyroxine to treat thyroid cancer. b. ACTH to treat adrenal carcinoma. c. Estrogen antagonists to treat breast cancer. d. Glucagon to treat pancreatic carcinoma. 15. Chemotherapeutic agents often produce a certain degree of myelosuppression including leukopenia. Leukopenia does not present immediately but is delayed several days to weeks because: a. The patients hemoglobin and hematocrit are normal. b. Red blood cells are affected first. c. Folic acid levels are normal. d. The current white cell count is not affected by chemotherapy. 16. Currently, there is no way to prevent myelosuppression. However, there are medications available to elicit a more rapid bone marrow recovery. An example is: a. Epoetin alfa (Epogen, Procrit). b. Glucagon. c. Fenofibrate (Tricor). d. Lamotrigine (Lamictal). 17. Estrogen antagonists are used to treat estrogen hormone-dependent cancer, such as breast carcinoma. Androgen antagonists block testosterone stimulation of androgen-dependent cancers. An example of an androgen-dependent cancer would be: a. Prostate cancer. b. Thyroid cancer. c. Renal carcinoma. d. neuroblastoma. 18. Serotonin release stimulates vomiting following chemotherapy. Therefore, serotonin antagonists are effective in preventing and treating nausea and vomiting related to chemotherapy. An example of an effective serotonin antagonist antiemetic is: a. ondansetron (Zofran). b. fluoxetine (Prozac). c. paroxetine (Paxil). d. sertraline (Zoloft). 19. Methotrexate, the most widely used antimetabolite in cancer chemotherapy does not penetrate the central nervous system (CNS). To treat CNS disease this drug must be administered: a. Intravenously. b. Subcutaneously. c. Intrathecally. d. By inhalation. 20. Methotrexate is a folate antagonist. It inhibits enzymes required for DNA base synthesis. To prevent harm to normal cells, a fully activated form of folic acid known as leucovorin (folinic acid; citrovorum factor) can be administered. Administration of leucovorin is known as: a. Induction therapy. b. Consolidation therapy. c. Pulse therapy. d. Rescue therapy.

139

21. A male Patient is undergoing chemotherapy may also be given the drug allopurinol (Zyloprim, Aloprim). Allopurinol inhibits the synthesis of uric acid. Concomitant administration of allopurinol prevents: a. Myelosuppression. b. Gout and hyperuricemia. c. Pancytopenia. d. Cancer cell growth and replication 22. Superficial bladder cancer can be treated by direct instillation of the antineoplastic antibiotic agent mitomycin (Mutamycin). This process is termed: a. Intraventricular administration. b. Intravesical administration. c. Intravascular administration. d. Intrathecal administration. 23. The most common dose-limiting toxicity of chemotherapy is: a. Nausea and vomiting. b. Bloody stools. c. Myelosuppression. d. Inability to ingest food orally due to stomatitis and mucositis. 24. Chemotherapy induces vomiting by: a. Stimulating neuroreceptors in the medulla. b. Inhibiting the release of catecholamines. c. Autonomic instability. d. Irritating the gastric mucosa. 25. Myeloablation using chemotherapeutic agents is useful in cancer treatment because: a. It destroys the myelocytes (muscle cells). b. It reduces the size of the cancer tumor. c. After surgery, it reduces the amount of chemotherapy needed. d. It destroys the bone marrow prior to transplant. 26. Anticipatory nausea and vomiting associated with chemotherapy occurs: a. Within the first 24 hours after chemotherapy. b. 1-5 days after chemotherapy. c. Before chemotherapy administration. d. While chemotherapy is being administered. 27. Medications bound to protein have the following effect: a. Enhancement of drug availability. b. Rapid distribution of the drug to receptor sites. c. The more drug bound to protein, the less available for desired effect. d. Increased metabolism of the drug by the liver. 28. Some drugs are excreted into bile and delivered to the intestines. Prior to elimination from the body, the drug may be absorbed. This process is known as: a. Hepatic clearance. b. Total clearance. c. Enterohepatic cycling. d. First-pass effect. 29. An adult patient has been taking a drug (Drug A) that is highly metabolized by the cytochrome p-450 system. He has been on this medication for 6 months. At this time, he is started on a second medication (Drug B) that is an inducer of the cytochrome p450 system. You should monitor this patient for:

140

a. Increased therapeutic effects of Drug A. b. Increased adverse effects of Drug B. c. Decreased therapeutic effects of Drug A. d. Decreased therapeutic effects of Drug B. 30. Epinephrine is administered to a female patient. The nurse should expect this agent to rapidly affect: a. Adrenergic receptors. b. Muscarinic receptors. c. Cholinergic receptors. d. Nicotinic receptors.

Answers & Rationale


Here are the answers and rationale for this exam. Counter check your answers to those below and tell us your scores. If you have any disputes or need more clarification to a certain question, please direct them to the comments section. 1. Answer C. Glycocalyx is a viscous polysaccharide or polypeptide slime that covers microbes. It enhances adherence to surfaces, resists phagocytic engulfment by the white blood cells, and prevents antibiotics from contacting the microbe. Glycocalyx does not have the effects in options B-D. 2. Answer B. The child can move his extremities and function in a normal fashion. This lessens stress associated with position restriction and promotes normal activity. Fear may not be eliminated. All lines can be dislodged. Even small catheters can be readily seen. 3. Answer C. In patients unable to take oral nutrition, parenteral hyperalimentation is an option for providing nutritional support. High concentrations of dextrose, protein, minerals, vitamins, and trace elements can be provided. Dosing is not affected with options a and d. Crystalloid can provide free water but has very little nutritional benefits. Hyperalimentation can provide free water and considerable nutritional benefits. 4. Answer D. A multilumen catheter contains separate ports and means to administer agents. An agent infusing in one port cannot mix with an agent infusing into another port. Thus, agents that would be incompatible if given together can be given in separate ports simultaneously. 5. Answer A. Occlusion occurs with slow infusion rates and concurrent administration of some medications. Lipid occlusions may be treated with 70 percent ethanol or with 0.1 mmol/mL NaOH. Lipids provide essential fatty acids. It is recommended that approximately 4 percent of daily calories be EFAs. A deficiency can quickly develop. Daily essential fatty acids are necessary for constant prostaglandin production. Lipids are almost isotonic with blood. 6. 7. Answer B. Strict aseptic technique including the use of cap, mask, and sterile gown and gloves is require when placing a central venous line including a PICC. Options A, C, and D are incorrect statements. They increase the risk of infection. Answer C. Pain related to PICC insertion occurs with puncture of the skin. When inserting PICC lines, the insertion site is anesthetized so no pain is felt. The patient will not receive general anesthesia or sedation. Statement 2 is false. Unnecessary pain should be prevented. 8. 9. Answer D. Any air entering the right heart can lead to a pulmonary embolus. All air should be purged from central venous lines; none should enter the patient. Answer A. A special portacath needle is used to access the portacath device. A syringe is attached and the sample is obtained. One of the primary reasons for insertion of a portacath device is the need for frequent or long-term blood sampling. A vacutainer will exert too much suction on the central line resulting in collapse of the line. Only special portacath needles should be used to access the portacath device. 10. Answer B. The actual access to the subclavian vein is still just under the clavicle, but by tunneling the distal portion of the catheter several inches under the skin the risk of migratory infection is reduces compared to a catheter that enters the subclavian vein directly and is not tunneled. The catheter is tunneled to prevent infection. 11. Answer C. A foreign body in a blood vessel increases the risk of infection. Catheters that come outside the body have an even higher risk of infection. Most infections are caused by skin bacteria. Other infective organisms include yeasts and fungi.

141

Options 1 and 4 are complications of a CVAD but are not the primary problem. Once placed, these lines do not cause pain and discomfort. 12. Answer D. A solution containing heparin is used to reduce catheter clotting and maintain patency. The concentration of heparin used depends on the patients age, comorbidities, and the frequency of catheter access/flushing. Although patients have few complications, the device is not risk free. Patients may develop infection, catheter clots, vascular obstruction, pneumothorax, hemothorax, or mechanical problems (catheter breakage). Strict adherence to protocol enhances the longevity of central access devices. They routinely last weeks to months and sometimes years. The patient will be taught how to perform dressing changes at home. 13. Answer A. Alkylating agents are highly reactive chemicals that introduce alkyl radicals into biologically active molecules and thereby prevent their proper functioning, replication, and transcription. Alkylating agents have numerous side effects including alopecia, nausea, vomiting, and myelosuppression. Nitrogen mustards have a broad spectrum of activity against chronic lymphocytic leukemia, non-Hodgkins lymphoma, and breast and ovarian cancer, but they are effective chemotherapeutic agents because of DNA cross-linkage. Alkylating agents are noncell cycle-specific agents. 14. Answer C. Estrogen antagonists are used to treat estrogen hormone-dependent cancer, such as breast carcinoma. A wellknown estrogen antagonist used in breast cancer therapy is tamoxifen (Nolvadex). This drug, in combination with surgery and other chemotherapeutic drugs reduces breast cancer recurrence by 30 percent. Estrogen antagonists can also be administered to prevent breast cancer in women who have a strong family history of the disease. Thyroxine is a natural thyroid hormone. It does not treat thyroid cancer. ACTH is an anterior pituitary hormone, which stimulates the adrenal glands to release glucocorticoids. It does not treat adrenal cancer. Glucagon is a pancreatic alpha cell hormone, which stimulates glycogenolysis and gluconeogenesis. It does not treat pancreatic cancer. 15. Answer D. The time required to clear circulating cells before the effect that chemotherapeutic drugs have on precursor cell maturation in the bone marrow becomes evident. Leukopenia is an abnormally low white blood cell count. Answers A-C pertain to red blood cells. 16. Answer A. Epoetin alfa (Epogen, Procrit) is a recombinant form of endogenous erythropoietin, a hematopoietic growth factor normally produced by the kidney that is used to induce red blood cell production in the bone marrow and reduce the need for blood transfusion. Glucagon is a pancreatic alpha cell hormone, which cause glycogenolysis and gluconeogenesis. Fenofibrate (Tricor) is an antihyperlipidemic agent that lowers plasma triglycerides. Lamotrigine (Lamictal) is an anticonvulsant. 17. Answer A. Prostate tissue is stimulated by androgens and suppressed by estrogens. Androgen antagonists will block testosterone stimulation of prostate carcinoma cells. The types of cancer in options 2-4 are not androgen dependent. 18. Answer A. Chemotherapy often induces vomiting centrally by stimulating the chemoreceptor trigger zone (CTZ) and peripherally by stimulating visceral afferent nerves in the GI tract. Ondansetron (Zofran) is a serotonin antagonist that bocks the effects of serotonin and prevents and treats nausea and vomiting. It is especially useful in single-day highly emetogenic cancer chemotherapy (for example, cisplatin). The agents in options 2-4 are selective serotonin reuptake inhibitors. They increase the available levels of serotonin. 19. Answer C. With intrathecal administration chemotherapy is injected through the theca of the spinal cord and into the subarachnoid space entering into the cerebrospinal fluid surrounding the brain and spinal cord. The methods in options A, B, and D are ineffective because the medication cannot enter the CNS. 20. Answer B. Leucovorin is used to save or rescue normal cells from the damaging effects of chemotherapy allowing them to survive while the cancer cells die. Therapy to rapidly reduce the number of cancerous cells is the induction phase. Consolidation therapy seeks to complete or extend the initial remission and often uses a different combination of drugs than that used for induction. Chemotherapy is often administered in intermittent courses called pulse therapy. Pulse therapy allows the bone marrow to recover function before another course of chemotherapy is given. 21. Answer B. Prevent uric acid nephropathy, uric acid lithiasis, and gout during cancer therapy since chemotherapy causes the rapid destruction of cancer cells leading to excessive purine catabolism and uric acid formation. Allopurinol can induce myelosuppression and pancytopenia. Allopurinol does not have this function.

142

22. Answer B. Medications administered intravesically are instilled into the bladder. Intraventricular administration involves the ventricles of the brain. Intravascular administration involves blood vessels. Intrathecal administration involves the fluid surrounding the brain and spinal cord. 23. Answer C. The overall goal of cancer chemotherapy is to give a dose large enough to be lethal to the cancer cells, but small enough to be tolerable for normal cells. Unfortunately, some normal cells are affected including the bone marrow. Myelosuppression limits the bodys ability to prevent and fight infection, produce platelets for clotting, and manufacture re d blood cells for oxygen portage. Even though the effects in options a, b, and d are uncomfortable and distressing to the patient, they do not have the potential for lethal outcomes that myelosuppression has. 24. Answer A. Vomiting (emesis) is initiated by a nucleus of cells located in the medulla called the vomiting center. This center coordinates a complex series of events involving pharyngeal, gastrointestinal, and abdominal wall contractions that lead to expulsion of gastric contents. Catecholamine inhibition does not induce vomiting. Chemotherapy does not induce vomiting from autonomic instability. Chemotherapy, especially oral agents, may have an irritating effect on the gastric mucosa, which could result in afferent messages to the solitary tract nucleus, but these pathways do not project to the vomiting center. 25. Answer A. Myelo comes from the Greek word myelos, which means marrow. Ablation comes from the Latin word ablatio, which means removal. Thus, myeloablative chemotherapeurtic agents destroy the bone marrow. This procedure destroys normal bone marrow as well as the cancerous marrow. The patients bone marrow will be replaced with a bone marrow transplant. Myelocytes are not muscle cells Tumors are solid masses typically located in organs. Surgery may be performed to reduce tumor burden and require less chemotherapy afterward. 26. Answer C. Nausea and vomiting (N&V) are common side effects of chemotherapy. Some patients are able to trigger these events prior to actually receiving chemotherapy by anticipating, or expecting, to have these effects. N&V occurring postchemotherapeutic administration is not an anticipatory event but rather an effect of the drug. N&V occurring during the administration of chemotherapy is an effect of the drug. 27. Answer C. Only an unbound drug can be distributed to active receptor sites. Therefore, the more of a drug that is bound to protein, the less it is available for the desired drug effect. Less drug is available if bound to protein. Distribution to receptor sites is irrelevant since the drug bound to protein cannot bind with a receptor site. Metabolism would not be increased. The liver will first have to remove the drug from the protein molecule before metabolism can occur. The protein is then free to return to circulation and be used again. 28. Answer C. Drugs and drug metabolites with molecular weights higher than 300 may be excreted via the bile, stored in the gallbladder, delivered to the intestines by the bile duct, and then reabsorbed into the circulation. This process reduces the elimination of drugs and prolongs their half-life and duration of action in the body. Hepatic clearance is the amount of drug eliminated by the liver. Total clearance is the sum of all types of clearance including renal, hepatic, and respiratory. First-pass effect is the amount of drug absorbed from the GI tract and then metabolized by the liver; thus, reducing the amount of drug making it into circulation. 29. Answer C. Drug B will induce the cytochrome p-450 enzyme system of the liver; thus, increasing the metabolism of Drug A. Therefore, Drug A will be broken down faster and exert decreased therapeutic effects. Drug A will be metabolized faster, thus reducing, not increasing its therapeutic effect. Inducing the cytochrome p-450 system will not increase the adverse effects of Drug B. Drug B induces the cytochrome p-450 system but is not metabolized faster. Thus, the therapeutic effects of Drug B will not be decreased. 30. Answer A. Epinephrine (adrenaline) rapidly affects both alpha and beta adrenergic receptors eliciting a sympathetic (fight or flight) response. Muscarinic receptors are cholinergic receptors and are primarily located at parasympathetic junctions. Cholinergic receptors respond to acetylcholine stimulation. Cholinergic receptors include muscarinic and nicotinic receptors. Nicotinic receptors are cholinergic receptors activated by nicotine and found in autonomic ganglia and somatic neuromuscular junctions.

143

As a nurse, we need to have an extensive knowledge about the different drugs and their uses. But how vast is your knowledge about the concepts covering Pharmacology? Take our 20-item exam about this topic.

Topics

Pharmacology nursing Drug computation and dosages Administration of medicine

Guidelines
Read each question carefully. Choose the best answer. You are given 1 minute and 20 seconds for each question. Answers & Rationales are given below. Be sure to read them!

Questions
1. A 2 year-old child is receiving temporary total parental nutrition (TPN) through a central venous line. This is the first day of TPN therapy. Although all of the following nursing actions must be included in the plan of care of this child, which one would be a priority at this time? a. Use aseptic technique during dressing changes b. Maintain central line catheter integrity c. Monitor serum glucose levels d. Check results of liver function tests 2. Nurse Jamie is administering the initial total parenteral nutrition solution to a client. Which of the following assessments requires the nurses immediate attention? a. Temperature of 37.5 degrees Celsius b. Urine output of 300 cc in 4 hours c. Poor skin turgor d. Blood glucose of 350 mg/dl 3. Nurse Susan administered intravenous gamma globulin to an 18 month-old child with AIDS. The parent asks why this medication is being given. What is the nurses best response? a. It will slow down the replication of the virus. b. This medication will improve your childs overall health status. c. This medication is used to prevent bacterial infections. d. It will increase the effectiveness of the other medications your child receives. 4. When caring for a client with total parenteral nutrition (TPN), what is the most important action on the part of the nurse? a. Record the number of stools per day b. Maintain strict intake and output records c. Sterile technique for dressing change at IV site d. Monitor for cardiac arrhythmias 5. The nurse is administering an intravenous vesicant chemotherapeutic agent to a client. Which assessment would require the nurses immediate action? a. Stomatitis lesion in the mouth b. Severe nausea and vomiting
144

c. Complaints of pain at site of infusion d. A rash on the clients extremities 6. Nurse Celine is caring for a client with clinical depression who is receiving a MAO inhibitor. When providing instructions about precautions with this medication, the nurse should instruct the client to: a. Avoid chocolate and cheese b. Take frequent naps c. Take the medication with milk d. Avoid walking without assistance 7. While providing home care to a client with congestive heart failure, the nurse is asked how long diuretics must be taken. The BEST response to this client should be: a. As you urinate more, you will need less medication to control fluid. b. You will have to take this medication for about a year. c. The medication must be continued so the fluid problem is controlled. d. Please talk to your physician about medications and treatments. 8. George, age 8, is admitted with rheumatic fever. Which clinical finding indicates to the nurse that George needs to continue taking the salicylates he had received at home? a. Chorea. b. Polyarthritis. c. Subcutaneous nodules. d. Erythema marginatum. 9. An order is written to start an IV on a 74-year-old client who is getting ready to go to the operating room for a total hip replacement. What gauge of catheter would best meet the needs of this client? a. 18 b. 20 c. 21 butterfly d. 25 10. A client with an acute exacerbation of rheumatoid arthritis is admitted to the hospital for treatment. Which drug, used to treat clients with rheumatoid arthritis, has both an anti-inflammatory and immunosuppressive effect? a. Gold sodium thiomalate (Myochrysine) b. Azathioprine (Imuran) c. Prednisone (Deltasone) d. Naproxen (Naprosyn) 11. Which of the following is least likely to influence the potential for a client to comply with lithium therapy after discharge? a. The impact of lithium on the clients energy level and life-style. b. The need for consistent blood level monitoring. c. The potential side effects of lithium. d. What the clients friends think of his need to take medication 12. Which of the following is least likely to influence the potential for a client to comply with lithium therapy after discharge? a. The impact of lithium on the clients energy level and life-style. b. The need for consistent blood level monitoring.

145

c. The potential side effects of lithium. d. What the clients friends think of his need to take medication. 13. The nurse is caring for an elderly client who has been diagnosed as having sundown syndrome. He is alert and oriented during the day but becomes disoriented and disruptive around dinnertime. He is hospitalized for evaluation. The nurse asks the client and his family to list all of the medications, prescription and nonprescription, he is currently taking. What is the primary reason for this action? a. Multiple medications can lead to dementia b. The medications can provide clues regarding his medical background c. Ability to recall medications is a good assessment of the clients level of orientation. d. Medications taken by a client are part of every nursing assessment. 14. A 25-year-old woman is in her fifth month of pregnancy. She has been taking 20 units of NPH insulin for diabetes mellitus daily for six years. Her diabetes has been well controlled with this dosage. She has been coming for routine prenatal visits, during which diabetic teaching has been implemented. Which of the following statements indicates that the woman understands the teaching regarding her insulin needs during her pregnancy? a. Are you sure all this insulin wont hurt my baby? b. Ill probably need my daily insulin dose raised. c. I will continue to take my regular dose of insulin. d. These finger sticks make my hand sore. Can I do them less frequently? 15. Mrs. Johansons physician has prescribed tetracycline 500 mg po q6h. While assessing Mrs. Johansons nursing history for allergies, the nurse notes that Mrs. Johansons is also taking oral contraceptives. What is the most appropriate initial nursing intervention? a. Administer the dose of tetracycline. b. Notify the physician that Mrs. Johanson is taking oral contraceptives. c. Tell Mrs. Johanson, she should stop taking oral contraceptives since they are inactivated by tetracycline. d. Tell Mrs. Johanson, to use another form of birth control for at least two months. 16. An adult clients insulin dosage is 10 units of regular insulin and 15 units of NPH insulin in the morning. The client should be taught to expect the first insulin peak: a. as soon as food is ingested. b. in two to four hours. c. in six hours. d. in ten to twelve hours. 17. An adult is hospitalized for treatment of deep electrical burns. Burn wound sepsis develops and mafenide acetate 10% (Sulfamylon) is ordered bid. While applying the Sulfamylon to the wound, it is important for the nurse to prepare the client for expected responses to the topical application, which include: a. severe burning pain for a few minutes following application. b. possible severe metabolic alkalosis with continued use. c. black discoloration of everything that comes in contact with this drug. d. chilling due to evaporation of solution from the moistened dressings. 18. Ms.Clark has hyperthyroidism and is scheduled for a thyroidectomy. The physician has ordered Lugols solution for the client. The nurse understands that the primary reason for giving Lugols solution preoperatively is to:

146

a. decrease the risk of agranulocytosis postoperatively. b. prevent tetany while the client is under general anesthesia. c. reduce the size and vascularity of the thyroid and prevent hemorrhage. d. potentiate the effect of the other preoperative medication so less medicine can be given while the client is under anesthesia. 19. A two-year-old child with congestive heart failure has been receiving digoxin for one week. The nurse needs to recognize that an early sign of digitalis toxicity is: a. bradypnea. b. failure to thrive. c. tachycardia. d. vomiting. 20. Mr. Bates is admitted to the surgical ICU following a left adrenalectomy. He is sleepy but easily aroused. An IV containing hydrocortisone is running. The nurse planning care for Mr. Bates knows it is essential to include which of the following nursing interventions at this time? a. Monitor blood glucose levels every shift to detect development of hypo- or hyperglycemia. b. Keep flat on back with minimal movement to reduce risk of hemorrhage following surgery. c. Administer hydrocortisone until vital signs stabilize, then discontinue the IV. d. Teach Mr. Bates how to care for his wound since he is at high risk for developing postoperative infection.

Answers & Rationale


Here are the answers and rationale for this exam. Counter check your answers to those below and tell us your scores. If you have any disputes or need more clarification to a certain question, please direct them to the comments section. 1. Answer C. Monitor serum glucose levels. Hyperglycemia may occur during the first day or 2 as the child adapts to the high-glucose load of the TPN solution. Thus, a chief nursing responsibility is blood glucose testing. 2. Answer D. Total parenteral nutrition formulas contain dextrose in concentrations of 10% or greater to supply 20% to 50% of the total calories. Blood glucose levels should be checked every 4 to 6 hours. A sliding scale dose of insulin may be ordered to maintain the blood glucose level below 200mg/dl. 3. Answer C. Intravenous gamma globulin is given to help prevent as well as to fight bacterial infections in young children with AIDS. 4. Answer C. Clients receiving TPN are very susceptible to infection. The concentrated glucose solutions are a good medium for bacterial growth. Strict sterile technique is crucial in preventing infection at IV infusion site. 5. Answer C. A vesicant is a chemotherapeutic agent capable of causing blistering of tissues and possible tissue necrosis if there is extravasation. These agents are irritants which cause pain along the vein wall, with or without inflammation.

147

6. Answer A. Foods high in tryptophan, tyramine and caffeine, such as chocolate and cheese may precipitate hypertensive crisis. 7. Answer C. This is the most therapeutic response and gives the client accurate information. 8. Answer B. Chorea is the restless and sudden aimless and irregular movements of the extremities suddenly seen in persons with rheumatic fever, especially girls. Polyarthritis is characterized by swollen, painful, hot joints that respond to salicylates. Subcutaneous nodules are nontender swellings over bony prominences sometimes seen in persons with rheumatic fever. Erythema marginatum is a skin condition characterized by nonpruritic rash, affecting trunk and proximal extremities, seen in persons with rheumatic fever. 9. Answer A. Clients going to the operating room ideally should have an 18- gauge catheter. This is large enough to handle blood products safely and to allow rapid administration of large amounts of fluid if indicated during the perioperative period. An 18-gauge catheter is recommended. A 20gauge catheter is a second choice. A 21-gauge needle is too small and a butterfly too unstable for a client going to surgery. A 25-gauge needle is too small. 10. Answer C. Gold sodium thiomalate is usually used in combination with aspirin and nonsteroidal antiinflammatory drugs to relieve pain. Gold has an immunosuppressive affect. Azathioprine is used for clients with life-threatening rheumatoid arthritis for its immunosuppressive effects. Prednisone is used to treat persons with acute exacerbations of rheumatoid arthritis. This medication is given for its anti-inflammatory and immunosuppressive effects. Naproxen is a nonsteroidal anti-inflammatory drug. Immunosuppression does not occur. 11. Answer D. The impact of lithium on the clients energy level and life style are great determinants to compliance. The frequent blood level monitoring required is difficult for clients to follow for a long period of time. Potential side effects such as fine tremor, drowsiness, diarrhea, polyuria, thirst, weight gain, and fatigue can be disturbing to the client. While the clients social network can influence the client in terms of compliance, the influence is typically secondary to that of the other factors listed. 12. Answer D. The impact of lithium on the clients energy level and life style are great determinants to compliance. The frequent blood level monitoring required is difficult for clients to follow for a long period of time. Potential side effects such as fine tremor, drowsiness, diarrhea, polyuria, thirst, weight gain, and fatigue can be disturbing to the client. While the clients social network can influence the client in terms of compliance, the influence is typically secondary to that of the other factors listed. 13. Answer A. Drugs commonly used by elderly people, especially in combination, can lead to dementia. Assessment of the medication taken may or may not provide information on the clients medical background. However, this is not the primary reason for assessing medications in a client who is exhibiting sundown syndrome. Ability to recall medications may indicate short-term memory and recall. However, that is not the primary reason for assessing medications in a client with

148

sundown syndrome. Medication history should be a part of the nursing assessment. In this client there is an even more important reason for evaluating the medications taken. 14. Answer B. The client starts to need increased insulin in the second trimester. This statement indicates a lack of understanding. As a result of placental maturation and placental production of lactogen, insulin requirements begin increasing in the second trimester and may double or quadruple by the end of pregnancy. The client starts to need increased insulin in the second trimester. This statement indicates a lack of understanding. Insulin doses depend on blood glucose levels. Finger sticks for glucose levels must be continued. 15. Answer B. The nurse should be aware that tetracyclines decrease the effectiveness of oral contraceptives. The physician should be notified. The physician should be notified. Tetracycline decreases the effectiveness of oral contraceptives. There may be an equally effective antibiotic available that can be prescribed. Note on the clients chart that the physician was notified. The nurse should be aware that tetracyclines decrease the effectiveness of oral contraceptives. The nurse should not tell the client to stop taking oral contraceptives unless the physician orders this. The nurse should be aware that tetracyclines decrease the effectiveness of oral contraceptives. If the physician chooses to keep the client on tetracycline, the client should be encouraged to use another form of birth control. The first intervention is to notify the physician. 16. Answer B. The first insulin peak will occur two to four hours after administration of regular insulin. Regular insulin is classified as rapid acting and will peak two to four hours after administration. The second peak will be eight to twelve hours after the administration of NPH insulin. This is why a snack must be eaten mid-morning and also three to four hours after the evening meal. The first insulin peak will occur two to four hours after administration of regular insulin. The first insulin peak will occur two to four hours after administration of regular insulin. The second peak will occur eight to twelve hours after the administration of NPH insulin. 17. Answer A. Mafenide acetate 10% (Sulfamylon) does cause burning on application. An analgesic may be required before the ointment is applied. Mafenide acetate 10% (Sulfamylon) is a strong carbonic anhydrase inhibitor that affects the renal tubular buffering system, resulting in metabolic acidosis. Mafenide acetate 10% (Sulfamylon) does not cause discoloration. Silver nitrate solution, another topical antibiotic used to treat burn sepsis, has the disadvantage of turning everything it touches black. Mafenide acetate 10% (Sulfamylon) is an ointment that is applied directly to the wound. It has the ability to diffuse rapidly through the eschar. The wound may be left open or dry dressing may be applied. Silver nitrate solution is applied by soaking the wound dressings and keeping them constantly wet, which may cause chilling and hypotension. 18. Answer C. Doses of over 30 mg/day may increase the risk of agranulocytosis. Lugols solution does not act to prevent tetany. Calcium is used to treat tetany. The client may receive iodine solution (Lugols solution) for 10 to 14 days before surgery to decrease vascularity of the thyroid and thus prevent excess bleeding. Lugols solution does not potentiate any other preoperative medication. 19. Answer D. Bradypnea (slow breathing) is not associated with digitalis toxicity. Bradycardia is associated with digitalis toxicity. Although children with congestive heart failure often have a related condition of failure to thrive, it is not directly related to digitalis administration. It is more related
149

to chronic hypoxia. Tachycardia is not a sign of digitalis toxicity. Bradycardia is a sign of digitalis toxicity. The earliest sign of digitalis toxicity is vomiting, although one episode does not warrant discontinuing medication. 20. Answer A. Hydrocortisone promotes gluconeogenesis and elevates blood glucose levels. Following adrenalectomy the normal supply of hydrocortisone is interrupted and must be replaced to maintain the blood glucose at normal levels. Care for the client following adrenalectomy is similar to that for any abdominal operation. The client is encouraged to change position, cough, and deep breathe to prevent postoperative complications such as pneumonia or thrombophlebitis. Maintenance doses of hydrocortisone will be administered IV until the client is able to take it by mouth and will be necessary for six months to two years or until the remaining gland recovers. The client undergoing an adrenalectomy is at increased risk for infection and delayed wound healing and will need to learn about wound care, but not at this time while he is in the ICU.

Introduction
As a nurse, we need to have an extensive knowledge about the different drugs and their uses. But how vast is your knowledge about the concepts covering Pharmacology? Take our part two exam for this topic.

Topics

Different drugs and their effects

Guidelines
Read each question carefully. Choose the best answer. You are given 1 minute and 20 seconds for each question. Answers & Rationales are given below. Be sure to read them!

Questions
1. The nursery nurse is putting erythromycin ointment in the newborns eyes to prevent infection. She places it in the following area of the eye: A. under the eyelid B. on the cornea. C. in the lower conjunctival sac D. by the optic disc. 2. The physician orders penicillin for a patient with streptococcal pharyngitis. The nurse administers the drug as ordered, and the patient has an allergic reaction. The nurse checks the medication order sheet and finds that the patient is allergic to penicillin. Legal responsibility for the error is: A. only the nursesshe should have checked the allergies before administering the medication. B. only the physiciansshe gave the order, the nurse is obligated to follow it. C. only the pharmacistshe should alert the floor to possible allergic reactions. D. the pharmacist, physician, and nurse are all liable for the mistake 3. James Perez, a nurse on a geriatric floor, is administering a dose of digoxin to one of his patients. The woman asks why she takes a different pill than her niece, who also has heart trouble. James replies that as people get older, liver and kidney function decline, and if the dose is as high as her nieces, the drug will tend to: A. have a shorter half-life. B. accumulate.

150

C. have decreased distribution. D. have increased absorption. 4. The nurse is administering augmentin to her patient with a sinus infection. Which is the best way for her to insure that she is giving it to the right patient? A. Call the patient by name B. Read the name of the patient on the patients door C. Check the patients wristband D. Check the patients room number on the unit census list 5. The most important instructions a nurse can give a patient regarding the use of the antibiotic ampicillin prescribed for her are to A. call the physician if she has any breathing difficulties. B. take it with meals so it doesnt cause an upset stomach. C. take all of the medication prescribed even if the symptoms stop sooner. D. not share the pills with anyone else. 6. Mr. Jessie Ray, a newly admitted patient, has a seizure disorder which is being treated with medication. Which of the following drugs would the nurse question if ordered for him? A. Phenobarbitol, 150 mg hs B. Amitriptylene (Elavil), 10 mg QID. C. Valproic acid (Depakote), 150 mg BID D. Phenytoin (Dilantin), 100 mg TID 7. Mrs. Jane Gately has been dealing with uterine cancer for several months. Pain management is the primary focus of her current admission to your oncology unit. Her vital signs on admission are BP 110/64, pulse 78, respirations 18, and temperature 99.2 F. Morphine sulfate 6mg IV, q 4 hours, prn has been ordered. During your assessment after lunch, your findings are: BP 92/60, pulse 66, respirations 10, and temperature 98.8. Mrs. Gately is crying and tells you she is still experiencing severe pain. Your action should be to A. give her the next ordered dose of MS. B. give her a back rub, put on some light music, and dim the lights in the room. C. report your findings to the MD, requesting an alternate medication order D. be obtained from the physician. E. call her daughter to come and sit with her. 8. When counseling a patient who is starting to take MAO (monoamine oxidase) inhibitors such as Nardil for depression, it is essential that they be warned not to eat foods containing tyramine, such as: A. Roquefort, cheddar, or Camembert cheese. B. grape juice, orange juice, or raisins. C. onions, garlic, or scallions. D. ground beef, turkey, or pork. 9. The physician orders an intramuscular injection of Demerol for the postoperativepatients pain. When preparing to draw up the medication, the nurse is careful to remove the correct vial from the narcotics cabinet. It is labeled A. simethicone. B. albuterol. C. meperidine. D. ibuprofen. 10. The nurse is administering an antibiotic to her pediatric patient. She checks the patients armband and verifies the corr ect medication by checking the physicians order, medication kardex, and vial. Which of the following is not considered one of the five rights of drug administration? A. Right dose B. Right route

151

C. Right frequency D. Right time 11. A nurse is preparing the clients morning NPH insulin dose and notices a clumpy precipitate inside the insulin vial. The nurse should: A. draw up and administer the dose B. shake the vial in an attempt to disperse the clumps C. draw the dose from a new vial D. warm the bottle under running water to dissolve the clump 12. A client with histoplasmosis has an order for ketoconazole (Nizoral). The nurse teaches the client to do which of the following while taking this medication? A. take the medication on an empty stomach B. take the medication with an antacid C. avoid exposure to sunlight D. limit alcohol to 2 ounces per day 13. A nurse has taught a client taking a xanthine bronchodilator about beverages to avoid. The nurse determines that the client understands the information if the client chooses which of the following beverages from the dietary menu? A. chocolate milk B. cranberry juice C. coffee D. cola 14. A client is taking famotidine (Pepcid) asks the home care nurse what would be the best medication to take for a headache. The nurse tells the client that it would be best to take: A. aspirin (acetylsalicylic acid, ASA) B. ibuprofen (Motrin) C. acetaminophen (Tylenol) D. naproxen (Naprosyn) 15. A nurse is planning dietary counseling for the client taking triamterene (Dyrenium). The nurse plans to include which of the following in a list of foods that are acceptable? A. baked potato B. bananas C. oranges D. pears canned in water 16. A client with advanced cirrhosis of the liver is not tolerating protein well, as eveidenced by abnormal laboratory values. The nurse anticipates that which of the following medications will be prescribed for the client? A. lactulose (Chronulac) B. ethacrynic acid (Edecrin) C. folic acid (Folvite) D. thiamine (Vitamin B1) 17. A female client tells the clinic nurse that her skin is very dry and irritated. Which product would the nurse suggest that the client apply to the dry skin? A. glycerin emollient B. aspercreame C. myoflex D. acetic acid solution 18. A nurse is providing instructions to a client regarding quinapril hydrochloride (Accupril). The nurse tells the client: A. to take the medication with food only B. to rise slowly from a lying to a sitting position

152

C. to discontinue the medication if nausea occurs D. that a therapeutic effect will be noted immediately 19. Auranofin (Ridaura) is prescribed for a client with rheumatoid arthritis, and the nurse monitors the client for signs of an adverse effect related to the medication. Which of the following indicates an adverse effect? A. nausea B. diarrhea C. anorexia D. proteinuria 20. A client has been taking benzonatate (Tessalon) as ordered. The nurse tells the client that this medication should do which of the following? A. take away nausea and vomiting B. calm the persistent cough C. decrease anxiety level D. increase comfort level

Answers & Rationale


Here are the answers and rationale for this exam. Counter check your answers to those below and tell us your scores. If you have any disputes or need more clarification to a certain question, please direct them to the comments section. 1. 2. 3. 4. C. The ointment is placed in the lower conjunctival sac so it will not scratch the eye itself and will get well distributed. D. The physician, nurse, and pharmacist all are licensed professionals and share responsibility for errors. B. The decreased circulation to the kidney and reduced liver function tend to allow drugs to accumulate and have toxic effects. C. The correct way to identify a patient before giving a medication is to check the name on the medication administration record with the patients identification band. The nurse should also ask the patient to state their name. The name on the door or the census list are not sufficient proof of identification. Calling the patient by name is not as effective as having the patient state their name; patients may not hear well or understand what the nurse is saying, and may respond to a name which is not their own. 5. 6. 7. 8. 9. C. Frequently patients do not complete an entire course of antibiotic therapy, and the bacteria are not destroyed. B. Elavil is an antidepressant that lowers the seizure threshold, so would not be appropriate for this patient. The other medications are anti-seizure drugs. C. Morphine sulfate depresses the respiratory center. When the rate is less than 10, the MD should be notified. A. Monoamine oxidase inhibitors react with foods high in the amino acid tyramine to cause dangerously high blood pressure. Aged cheeses are all high in this amino acid; the other foods are not. C. The generic name for Demerol is meperidine. not included. 11. C. The nurse should always inspect the vial of insulin before use for solution changes that may signify loss of potency. NPH insulin is normally uniformly cloudy. Clumping, frosting, and precipitates are signs of insulin damage. In this situation, because potency is questionable, it is safer to discard the vial and draw up the dose from a new vial. 12. C. The client should be taught that ketoconazole is an antifungal medication. It should be taken with food or milk. Antacids should be avoided for 2 hours after it is taken because gastric acid is needed to activate the medication. The client should avoid concurrent use of alcohol, because the medication is hepatotoxic. The client should also avoid exposure to sunlight, because the medication increases photosensitivity. 13. B. Cola, coffee, and chocolate contain xanthine and should be avoided by the client taking a xanthine bronchodilator. This could lead to an increased incidence of cardiovascular and central nervous system side effects that can occur with the use of these types of bronchodilators. 10. C. The five rights of medication administration are right drug, right dose, right route, right time, right patient. Frequency is

153

14. C. The client is taking famotidine, a histamine receptor antagonist. This implies that the client has a disorder characterized by gastrointestinal (GI) irritation. The only medication of the ones listed in the options that is not irritating to the GI tract is acetaminophen. The other medications could aggravate an already existing GI problem. 15. D. Triamterene is a potassium-sparing diuretic, and clients taking this medication should be cautioned against eating foods that are high in potassium, including many vegetables, fruits, and fresh meats. Because potassium is very water-soluble, foods that are prepared in water are often lower in potassium. 16. A. The client with cirrhosis has impaired ability to metabolize protein because of liver dysfunction. Administration of lactulose aids in the clearance of ammonia via the gastrointestinal (GI) tract. Ethacrynic acid is a diuretic. Folic acid and thiamine are vitamins, which may be used in clients with liver disease as supplemental therapy. 17. A. Glycerin is an emollient that is used for dry, cracked, and irritated skin. Aspercreme and Myoflex are used to treat muscular aches. Acetic acid solution is used for irrigating, cleansing, and packing wounds infected by Pseudomonas aeruginosa. 18. B. Accupril is an angiotensin-converting enzyme (ACE) inhibitor. It is used in the treatment of hypertension. The client should be instructed to rise slowly from a lying to sitting position and to permit the legs to dangle from the bed momentarily before standing to reduce the hypotensive effect. The medication does not need to be taken with meals. It may be given without regard to food. If nausea occurs, the client should be instructed to take a non cola carbonated beverage and salted crackers or dry toast. A full therapeutic effect may be noted in 1 to 2 weeks. 19. D. Auranofin (Ridaura) is a gold preparation that is used as an antirheumatic. Gold toxicity is an adverse effect and is evidenced by decreased hemoglobin, leukopenia, reduced granulocyte counts, proteinuria, hematuria, stomatitis, glomerulonephritis, nephrotic syndrome, or cholestatic jaundice. Anorexia, nausea, and diarrhea are frequent side effects of the medication. 20. B. Benzonatate is a locally acting antitussive. Its effectiveness is measured by the degree to which it decreases the intensity and frequency of cough, without eliminating the cough reflex.

12 Effective Test-Taking Skills & Strategies for Nurses


Before stepping into the real world of nursing, one must pass their licensure examinations first! Reviewing board exam questions are just a way of further enhancing your knowledge and confidence for one of the most important test in your life. What you must remember is that you are already knowledgeable about cores of nursing, but this knowledge would be futile if you cannot practice effective test-taking skills during your exams. Researches suggest that an examiner who fully understands test construction and familiar with appropriate test-taking strategies score higher than those with similar level of knowledge but with inadequate test-taking skills. Multiple choice tests are different from tests such as essays, identifications and true/false. In multiple choice exams, the question is called the stem. The stem is followed by four alternative answers. One answer is correct, and the other three are called distracterbecause they distract your attention from identifying the correct answer. Note also that these distracter are not necessarily incorrect answers but rather they may not be as correct as as the one you are required to choose. Here we have 12 effective test-taking skills and strategies that can help you during your examinations: 1. Do not read extra meaning into the question

Tests questions are made to be direct and to the point. The question asks for one particular response and you should not read or add other information into the question.

154

Often you will find questions that require common sense answers and that reading into these questions may give you an other interpretation. You should not search for subtle meaning about the questions or answers. Ask yourself What is the question asking?. Look for keywords and phrases to help you understand. Interpret the question correctly first before reading into the choices.

2. Understand exactly what the stem is asking before considering the distracter

Make sure you read the stem correctly. Notice particularly the way the question is phrase. Is it asking for the best response or the initial response? Understand what the question is asking.

3. Rephrase the question in your own words

This technique requires you to interpret or translate the question into your own words so that it is very clear in your own mind. Rephrasing the stem of the question can assist your read the question correctly and in turn choose the appropriate response. Placing the question into your own words would help you in removing extraneous data and get into the core of the stem.

4. When analyzing the distracter, isolate what is important in the answer alternatives from what is not important relative to the question.

In a good test construction, all of the distracter should be feasible and reasonable and should apply directly to the stem. There should be a commonality in all of the distracter. Also, all of the distracter may be correct but not the right chose for the specific question that is being asked. The technique here is to ask yourself whether each possible alternative is true or false in relation to the stem.

5. After choosing the correct answer alternative and separating it form the distracter, go back to the stem and make sure your choice does, in fact, answer the question.

Many test-takers fail to recheck the answer with the stem, and they answer the question incorrectly. An effective strategy is to judge all four alternative choices against the stem and not against one another. Read the stem, then check alternative 1 against the stem, then check alternative 2 against the stem and so on. This process will eliminate choosing an alternative that does not fit with the question.

6. When a question contains multiple variables as alternative choices, use the elimination of variable technique.

Each question may pose different alternatives with several variables. Use the process of elimination. Study the question first and ask yourself what variable fits with this condition, or after examining the distracter underline the symptom that you know is correct. Now ask yourself what variable is not present with this condition. Again examine the distracter and cross out those variables that are incorrect. By this process your probably eliminated at least two distracter even without taking the time to consider the other two.

7. When answering a difficult question, utilize your body of knowledge.


When you come across a difficult question and you cannot immediately identify the answer, go back to your body of knowledge and draw all the information that you do know about the condition. If you are unfamiliar with the disease or disorder and cannot choose the right nursing action, try to generalize to other situations. For example, if the question asks about dog bites, and youve never learned the course of the disorder, go back to an area of k nowledge that you do know, for example, circulation and body response to toxic substances. Even though you do not know exactly what to d, you might know what not to do. Eliminate distracter to increase your chances of arriving at the correct answer.

8. The ability to guess correctly is both a skill and an art.

The board exams is not a right minus wrong type. It is important for you to answer every question even if you have to guess. Guessing gives you only a 25% chance of getting the correct answer. 155

Try to eliminate at least one (or more) distracter as this will increase the percentage margin of chance for guessing correctly. Examine the distracter and if one is the exact opposite of another (e.g. complete bed rest is different from activity as tolerated; both cannot be correct since they are of opposites), choose the one that seems to be most logical. Try to identify the underlying principle that supports the question. If you can answer the question, you might then be able to guess the correct answer. This strategy is especially true with a psychosocial question. Look at the way the alternatives are presented. Are there two answers that are very close? often when this occurs, the ability to discriminate will show evidence of judgement. Check to see if one, more than the other, is the best choice for the question. Are there distracter that are presented not logical (which are correct in themselves but do not have anything to do with the question)? Eliminate these and focus on other alternatives. Use your intuition. If you cannot choose an alternative from a logical point, allow yourself to feel which one might be right. Often your subconscious mind will choose correctly (based on all the conscious knowledge you have of course) so simply let yourself feel which alternative might be right. Remember, its better to choose one answer than none at all.

9. Choosing answer from a hunch

There comes a time when you are faced with a certain question and you have a hunch that this particular choice is correct. Do we depend on this hunch?. Current studies supports that hunches are often correct, for they are based on rapid subconscious connections in the brain. Your stored knowledge, recall, and experience can combine to assist you in arriving at the correct answer. So, if you have an initial hunch, go with it! Do not chance the answer if and only if, upon reflection, it just doesnt seem right. On the other hand, if later in the test you find relevant information or make new connection of information and you feel that your answer was incorrect, do go back and change it.

10. Choosing the best answer from a strategy point-of-view.

Frequently, the most comprehensive answer is the best choice (Longest the best!). For example, if two alternatives seems reasonable but one answer includes the other (i.e., it is more detailed, more comprehensive), then this answer would be the best choice. If an answer focuses on medical knowledge, be wary, for this alternative might be just a good distractor. Remember, this is a nursing test and questions are designed to test your nursing competency and safety. It is unlikely that a question would require a medical action for the correct answer; it may, however offer these actions as distracter. Beware of answers that contain specific qualifiers, such as always and never. they rarely fit within a logical framework.

11. Time Management is important

Time allotment for each question in the NLE is 2 hours/100 questions is equal to 1.2 minutes. It is important to use your time wisely and program yourself not to spend too much time on one question. Since your are not penalized for incorrect answers, it is important to finish each portion of the test. Prepare yourself to practice time management by timing yourself for each question during your sample exams.

12. Let the Divine Intervention Intervene

If you cannot answer the question. Have faith in the Lord and thus youll be confident to answer each question. If you do not know, He knows. Ask for Gods help! Remember, you are doing this for His glory!

156

Reference: Smith S. (1989). Review questions for NCLEX-RN 5th Edition.

Introduction
Another set of the ultimate reviewer for Fundamentals of Nursing. The Nursing Bullets are compilation of bite-sized information that are extremely useful during your review for the NCLEX or the board exams.

Topics
Topics included are:

Nursing Assessment Diets Various topics about Fundamentals of Nursing

Bullets
1. After turning a patient, the nurse should document the position used, the time that the patient was turned, and the findings of skin assessment. 2. PERRLA is an abbreviation for normal pupil assessment findings: pupils equal, round, and reactive to light with accommodation. 3. When percussing a patients chest for postural drainage, the nurses hands should be cupped. 4. When measuring a patients pulse, the nurse should assess its rate, rhythm, quality, and strength. 5. Before transferring a patient from a bed to a wheelchair, the nurse should push the wheelchair footrests to the sides and lock its wheels. 6. When assessing respirations, the nurse should document their rate, rhythm, depth, and quality. 7. For a subcutaneous injection, the nurse should use a 5/8 25G needle. 8. The notation AA & O 3 indicates that the patient is awake, alert, and oriented to person (knows who he is), place (kno ws where he is), and time (knows the date and time). 9. Fluid intake includes all fluids taken by mouth, including foods that are liquid at room temperature, such as gelatin, custard, and ice cream; I.V. fluids; and fluids administered in feeding tubes. Fluid output includes urine, vomitus, and drainage (such as from a nasogastric tube or from a wound) as well as blood loss, diarrhea or feces, and perspiration. 10. After administering an intradermal injection, the nurse shouldnt massage the area because massage can irritate the site and interfere with results. 11. When administering an intradermal injection, the nurse should hold the syringe almost flat against the patients skin (at about a 15-degree angle), with the bevel up. 12. To obtain an accurate blood pressure, the nurse should inflate the manometer to 20 to 30 mm Hg above the disappearance of the radial pulse before releasing the cuff pressure. 13. The nurse should count an irregular pulse for 1 full minute. 14. A patient who is vomiting while lying down should be placed in a lateral position to prevent aspiration of vomitus. 15. Prophylaxis is disease prevention. 16. Body alignment is achieved when body parts are in proper relation to their natural position. 17. Trust is the foundation of a nurse-patient relationship. 18. Blood pressure is the force exerted by the circulating volume of blood on the arterial walls. 19. Malpractice is a professionals wrongful conduct, improper discharge of duties, or failure to meet standards of care that causes harm to another. 20. As a general rule, nurses cant refuse a patient care assignment; ho wever, in most states, they may refuse to participate in abortions.

157

21. A nurse can be found negligent if a patient is injured because the nurse failed to perform a duty that a reasonable and prudent person would perform or because the nurse performed an act that a reasonable and prudent person wouldnt perform. 22. States have enacted Good Samaritan laws to encourage professionals to provide medical assistance at the scene of an accident without fear of a lawsuit arising from the assistance. These laws don t apply to care provided in a health care facility. 23. A physician should sign verbal and telephone orders within the time established by facility policy, usually 24 hours. 24. A competent adult has the right to refuse lifesaving medical treatment; however, the individual should be fully informed of the consequences of his refusal. 25. Although a patients health record, or chart, is the health care facilitys physical property, its contents belong to the patient. 26. Before a patients health record can be released to a third party, the patient or the patients legal guardian must give written consent. 27. Under the Controlled Substances Act, every dose of a controlled drug thats dispensed by the pharmacy must be accounted for, whether the dose was administered to a patient or discarded accidentally. 28. A nurse cant perform duties that violate a rule or regulation established by a state licensing board, even if they are a uthorized by a health care facility or physician. 29. To minimize interruptions during a patient interview, the nurse should select a private room, preferably one with a door that can be closed. 30. In categorizing nursing diagnoses, the nurse addresses life-threatening problems first, followed by potentially life-threatening concerns. 31. The major components of a nursing care plan are outcome criteria (patient goals) and nursing interventions. 32. Standing orders, or protocols, establish guidelines for treating a specific disease or set of symptoms. 33. In assessing a patients heart, the nurse normally finds the point of maximal impulse at the fifth intercostal space, near the apex. 34. The S1 heard on auscultation is caused by closure of the mitral and tricuspid valves. 35. To maintain package sterility, the nurse should open a wrappers top flap away from the body, open each side flap by touching only the outer part of the wrapper, and open the final flap by grasping the turned-down corner and pulling it toward the body. 36. The nurse shouldnt dry a patients ear canal or remove wax with a cotton-tipped applicator because it may force cerumen against the tympanic membrane. 37. A patients identification bracelet should remain in place until the patient has been discharged from the health care fac ility and has left the premises. 38. The Controlled Substances Act designated five categories, or schedules, that classify controlled drugs according to their abuse potential. 39. Schedule I drugs, such as heroin, have a high abuse potential and have no currently accepted medical use in the United States. 40. Schedule II drugs, such as morphine, opium, and meperidine (Demerol), have a high abuse potential, but currently have accepted medical uses. Their use may lead to physical or psychological dependence. 41. Schedule III drugs, such as paregoric and butabarbital (Butisol), have a lower abuse potential than Schedule I or II drugs. Abuse of Schedule III drugs may lead to moderate or low physical or psychological dependence, or both. 42. Schedule IV drugs, such as chloral hydrate, have a low abuse potential compared with Schedule III drugs. 43. Schedule V drugs, such as cough syrups that contain codeine, have the lowest abuse potential of the controlled substances. 44. Activities of daily living are actions that the patient must perform every day to provide self-care and to interact with society. 45. Testing of the six cardinal fields of gaze evaluates the function of all extraocular muscles and cranial nerves III, IV, and VI. 46. The six types of heart murmurs are graded from 1 to 6. A grade 6 heart murmur can be heard with the stethoscope slightly raised from the chest. 47. The most important goal to include in a care plan is the patients goal. 48. Fruits are high in fiber and low in protein, and should be omitted from a low-residue diet. 49. The nurse should use an objective scale to assess and quantify pain. Postoperative pain varies greatly among individuals.

158

50. Postmortem care includes cleaning and preparing the deceased patient for family viewing, arranging transportation to the morgue or funeral home, and determining the disposition of belongings. 51. The nurse should provide honest answers to the patients questions. 52. Milk shouldnt be included in a clear liquid diet. 53. When caring for an infant, a child, or a confused patient, consistency in nursing personnel is paramount. 54. The hypothalamus secretes vasopressin and oxytocin, which are stored in the pituitary gland. 55. The three membranes that enclose the brain and spinal cord are the dura mater, pia mater, and arachnoid. 56. A nasogastric tube is used to remove fluid and gas from the small intestine preoperatively or postoperatively. 57. Psychologists, physical therapists, and chiropractors arent authorized to write prescriptions for drugs. 58. The area around a stoma is cleaned with mild soap and water. 59. Vegetables have a high fiber content. 60. The nurse should use a tuberculin syringe to administer a subcutaneous injection of less than 1 ml. 61. For adults, subcutaneous injections require a 25G 1 needle; for infants, children, elderly, or very thin patients, they require a 25G to 27G needle. 62. Before administering a drug, the nurse should identify the patient by checking the identification band and asking the patient to state his name. 63. To clean the skin before an injection, the nurse uses a sterile alcohol swab to wipe from the center of the site outward in a circular motion. 64. The nurse should inject heparin deep into subcutaneous tissue at a 90-degree angle (perpendicular to the skin) to prevent skin irritation. 65. If blood is aspirated into the syringe before an I.M. injection, the nurse should withdraw the needle, prepare another syringe, and repeat the procedure. 66. The nurse shouldnt cut the patients hair without written consent from the patient or an appropr iate relative. 67. If bleeding occurs after an injection, the nurse should apply pressure until the bleeding stops. If bruising occurs, the nurse should monitor the site for an enlarging hematoma. 68. When providing hair and scalp care, the nurse should begin combing at the end of the hair and work toward the head. 69. The frequency of patient hair care depends on the length and texture of the hair, the duration of hospitalization, and the patients condition. 70. Proper function of a hearing aid requires careful handling during insertion and removal, regular cleaning of the ear piece to prevent wax buildup, and prompt replacement of dead batteries. 71. The hearing aid thats marked with a blue dot is for the left ear; the one with a red dot is for the right ear. 72. A hearing aid shouldnt be exposed to heat or humidity and shouldnt be immersed in water. 73. The nurse should instruct the patient to avoid using hair spray while wearing a hearing aid. 74. The five branches of pharmacology are pharmacokinetics, pharmacodynamics, pharmacotherapeutics, toxicology, and pharmacognosy. 75. The nurse should remove heel protectors every 8 hours to inspect the foot for signs of skin breakdown. 76. Heat is applied to promote vasodilation, which reduces pain caused by inflammation. 77. A sutured surgical incision is an example of healing by first intention (healing directly, without granulation). 78. Healing by secondary intention (healing by granulation) is closure of the wound when granulation tissue fills the defect and allows reepithelialization to occur, beginning at the wound edges and continuing to the center, until the entire wound is covered. 79. Keloid formation is an abnormality in healing thats characterized by overgrowth of scar tissue at the wound site. 80. The nurse should administer procaine penicillin by deep I.M. injection in the upper outer portion of the buttocks in the adult or in the midlateral thigh in the child. The nurse shouldnt massage the injection site. 81. An ascending colostomy drains fluid feces. A descending colostomy drains solid fecal matter. 82. A folded towel (scrotal bridge) can provide scrotal support for the patient with scrotal edema caused by vasectomy, epididymitis, or orchitis.

159

83. When giving an injection to a patient who has a bleeding disorder, the nurse should use a small-gauge needle and apply pressure to the site for 5 minutes after the injection. 84. Platelets are the smallest and most fragile formed element of the blood and are essential for coagulation. 85. To insert a nasogastric tube, the nurse instructs the patient to tilt the head back slightly and then inserts the tube. When the nurse feels the tube curving at the pharynx, the nurse should tell the patient to tilt the head forward to close the trachea and open the esophagus by swallowing. (Sips of water can facilitate this action.) 86. Families with loved ones in intensive care units report that their four most important needs are to have their questions answered honestly, to be assured that the best possible care is b eing provided, to know the patients prognosis, and to feel that there is hope of recovery. 87. Double-bind communication occurs when the verbal message contradicts the nonverbal message and the receiver is unsure of which message to respond to. 88. A nonjudgmental attitude displayed by a nurse shows that she neither approves nor disapproves of the patient. 89. Target symptoms are those that the patient finds most distressing. 90. A patient should be advised to take aspirin on an empty stomach, with a full glass of water, and should avoid acidic foods such as coffee, citrus fruits, and cola. 91. For every patient problem, there is a nursing diagnosis; for every nursing diagnosis, there is a goal; and for every goal, there are interventions designed to make the goal a reality. The keys to answering examination questions correctly are identifying the problem presented, formulating a goal for the problem, and selecting the intervention from the choices provided that will enable the patient to reach that goal. 92. Fidelity means loyalty and can be shown as a commitment to the profession of nursing and to the patient. 93. Administering an I.M. injection against the patients will and without legal authority is battery. 94. An example of a third-party payer is an insurance company. 95. The formula for calculating the drops per minute for an I.V. infusion is as follows: (volume to be infused drip factor) time in minutes = drops/minute 96. On-call medication should be given within 5 minutes of the call. 97. Usually, the best method to determine a patients cultural or spiritual needs is to ask him. 98. An incident report or unusual occurrence report isnt part of a patients record, but is an in -house document thats used for the purpose of correcting the problem. 99. Critical pathways are a multidisciplinary guideline for patient care. 100. When prioritizing nursing diagnoses, the following hierarchy should be used: Problems associated with the airway, those concerning breathing, and those related to circulation. 101. The two nursing diagnoses that have the highest priority that the nurse can assign are Ineffective airway clearance and Ineffective breathing pattern. 102. A subjective sign that a sitz bath has been effective is the patients expression of decreased pa in or discomfort. 103. For the nursing diagnosis Deficient diversional activity to be valid, the patient must state that hes bored, that he has nothing to do, or words to that effect. 104. The most appropriate nursing diagnosis for an individual who d oesnt speak English is Impaired verbal communication related to inability to speak dominant language (English). 105. The family of a patient who has been diagnosed as hearing impaired should be instructed to face the individual when they speak to him. 106. Before instilling medication into the ear of a patient who is up to age 3, the nurse should pull the pinna down and back to straighten the eustachian tube. 107. To prevent injury to the cornea when administering eyedrops, the nurse should waste the first drop and instill the drug in the lower conjunctival sac. 108. After administering eye ointment, the nurse should twist the medication tube to detach the ointment. 109. When the nurse removes gloves and a mask, she should remove the gloves first. They are soiled and are likely to contain pathogens.

160

110. Crutches should be placed 6 (15.2 cm) in front of the patient and 6 to the side to form a tripod arrangement. 111. Listening is the most effective communication technique. 112. Before teaching any procedure to a patient, the nurse must assess the patients current knowledge and willingness to learn. 113. Process recording is a method of evaluating ones communication effectiveness. 114. When feeding an elderly patient, the nurse should limit high-carbohydrate foods because of the risk of glucose intolerance. 115. When feeding an elderly patient, essential foods should be given first. 116. Passive range of motion maintains joint mobility. Resistive exercises increase muscle mass. 117. Isometric exercises are performed on an extremity thats in a cast. 118. A back rub is an example of the gate-control theory of pain. 119. Anything thats located below the waist is considered unsterile; a sterile field becomes unsterile when it comes in cont act with any unsterile item; a sterile field must be monitored continuously; and a border of 1 (2.5 cm) around a sterile field is considered unsterile. 120. A shift to the left is evident when the number of immature cells (bands) in the blood increases to fight an infection . 121. A shift to the right is evident when the number of mature cells in the blood increases, as seen in advanced liver dise ase and pernicious anemia. 122. Before administering preoperative medication, the nurse should ensure that an informed consent form has been signed and attached to the patients record. 123. A nurse should spend no more than 30 minutes per 8-hour shift providing care to a patient who has a radiation implant. 124. A nurse shouldnt be assigned to care for more than one patient who ha s a radiation implant. 125. Long-handled forceps and a lead-lined container should be available in the room of a patient who has a radiation implant. 126. Usually, patients who have the same infection and are in strict isolation can share a room. 127. Diseases that require strict isolation include chickenpox, diphtheria, and viral hemorrhagic fevers such as Marburg disease. 128. For the patient who abides by Jewish custom, milk and meat shouldnt be served at the same meal. 129. Whether the patient can perform a procedure (psychomotor domain of learning) is a better indicator of the effectiveness of patient teaching than whether the patient can simply state the steps involved in the procedure (cognitive domain of learning). 130. According to Erik Erikson, developmental stages are trust versus mistrust (birth to 18 months), autonomy versus shame and doubt (18 months to age 3), initiative versus guilt (ages 3 to 5), industry versus inferiority (ages 5 to 12), identity versus identity diffusion (ages 12 to 18), intimacy versus isolation (ages 18 to 25), generativity versus stagnation (ages 25 to 60), and ego integrity versus despair (older than age 60). 131. When communicating with a hearing impaired patient, the nurse should face him. 132. An appropriate nursing intervention for the spouse of a patient who has a serious incapacitating disease is to help him to mobilize a support system. 133. Hyperpyrexia is extreme elevation in temperature above 106 F (41.1 C). 134. Milk is high in sodium and low in iron. 135. When a patient expresses concern about a health-related issue, before addressing the concern, the nurse should assess the patients level of knowledge. 136. The most effective way to reduce a fever is to administer an antipyretic, which lowers the temperature set point. 137. When a patient is ill, its essential for the members of his family to maintain communication about his health needs. 138. Ethnocentrism is the universal belief that ones way of life is superior to others. 139. When a nurse is communicating with a patient through an interpreter, the nurse should speak to the patient and the interpreter. 140. In accordance with the hot-cold system used by some Mexicans, Puerto Ricans, and other Hispanic and Latino groups, most foods, beverages, herbs, and drugs are described as cold. 141. Prejudice is a hostile attitude toward individuals of a particular group. 142. Discrimination is preferential treatment of individuals of a particular group. Its usually discussed in a negative sens e. 143. Increased gastric motility interferes with the absorption of oral drugs. 144. The three phases of the therapeutic relationship are orientation, working, and termination.

161

145. Patients often exhibit resistive and challenging behaviors in the orientation phase of the therapeutic relationship. 146. Abdominal assessment is performed in the following order: inspection, auscultation, percussion & palpation. 147. When measuring blood pressure in a neonate, the nurse should select a cuff thats no less than one -half and no more than two-thirds the length of the extremity thats used. 148. When administering a drug by Z-track, the nurse shouldnt use the same needle that was used to draw the drug into the syringe because doing so could stain the skin. 149. Sites for intradermal injection include the inner arm, the upper chest, and on the back, under the scapula. 150. When evaluating whether an answer on an examination is correct, the nurse should consider whether the action thats described promotes autonomy (independence), safety, self-esteem, and a sense of belonging. 151. When answering a question on the NCLEX examination, the student should consider the cue (the stimulus for a thought) and the inference (the thought) to determine whether the inference is correct. When in doubt, the nurse should select an answer that indicates the need for further information to eliminate ambiguity. For example, the patient complains of chest pain (the stimulus for the thought) and the nurse infers that the patient is having cardiac pain (the thought). In this case, the nurse hasnt confirmed whether the pain is cardiac. It would be more appropriate to make further assessments. 152. Veracity is truth and is an essential component of a therapeutic relationship between a health care provider and his patient. 153. Beneficence is the duty to do no harm and the duty to do good. Theres an obligation in patient care to do no harm and a n equal obligation to assist the patient. 154. Nonmaleficence is the duty to do no harm. 155. Fryes ABCDE cascade provides a framework for prioritizing care by identifying the most important treatment concerns. 156. A = Airway. This category includes everything that affects a patent airway, including a foreign object, fluid from an upper respiratory infection, and edema from trauma or an allergic reaction. 157. B = Breathing. This category includes everything that affects the breathing pattern, including hyperventilation or hypoventilation and abnormal breathing patterns, such as Korsakoffs, Biots, or Cheyne -Stokes respiration. 158. C = Circulation. This category includes everything that affects the circulation, including fluid and electrolyte disturbances and disease processes that affect cardiac output. 159. D = Disease processes. If the patient has no problem with the airway, breathing, or circulation, then the nurse should evaluate the disease processes, giving priority to the disease process that poses the greatest immediate risk. For example, if a patient has terminal cancer and hypoglycemia, hypoglycemia is a more immediate concern. 160. E = Everything else. This category includes such issues as writing an incident report and completing the patient chart. When evaluating needs, this category is never the highest priority. 161. When answering a question on an NCLEX examination, the basic rule is assess before action. The student should evaluate each possible answer carefully. Usually, several answers reflect the implementation phase of nursing and one or two reflect the assessment phase. In this case, the best choice is an assessment response unless a specific course of action is clearly indicated. 162. Rule utilitarianism is known as the greatest good for the greatest number of people theory. 163. Egalitarian theory emphasizes that equal access to goods and services must be provided to the less fortunate by an affluent society. 164. Active euthanasia is actively helping a person to die. 165. Brain death is irreversible cessation of all brain function. 166. Passive euthanasia is stopping the therapy thats sustaining life. 167. A third-party payer is an insurance company. 168. Utilization review is performed to determine whether the care provided to a patient was appropriate and cost-effective. 169. A value cohort is a group of people who experienced an out-of-the-ordinary event that shaped their values. 170. Voluntary euthanasia is actively helping a patient to die at the patients request. 171. Bananas, citrus fruits, and potatoes are good sources of potassium. 172. Good sources of magnesium include fish, nuts, and grains. 173. Beef, oysters, shrimp, scallops, spinach, beets, and greens are good sources of iron. 174. Intrathecal injection is administering a drug through the spine.

162

175. When a patient asks a question or makes a statement thats emotionally charged, t he nurse should respond to the emotion behind the statement or question rather than to whats being said or asked. 176. The steps of the trajectory-nursing model are as follows: 177. Step 1: Identifying the trajectory phase 178. Step 2: Identifying the problems and establishing goals 179. Step 3: Establishing a plan to meet the goals 180. Step 4: Identifying factors that facilitate or hinder attainment of the goals 181. Step 5: Implementing interventions 182. Step 6: Evaluating the effectiveness of the interventions 183. A Hindu patient is likely to request a vegetarian diet. 184. Pain threshold, or pain sensation, is the initial point at which a patient feels pain. 185. The difference between acute pain and chronic pain is its duration. 186. Referred pain is pain thats felt at a site other than its origin. 187. Alleviating pain by performing a back massage is consistent with the gate control theory. 188. Rombergs test is a test for balance or gait. 189. Pain seems more intense at night because the patient isnt distracted by daily activities. 190. Older patients commonly dont report pain because of fear of treatment, lifestyle changes, or dependency. 191. No pork or pork products are allowed in a Muslim diet. 192. Two goals of Healthy People 2010 are: 193. Help individuals of all ages to increase the quality of life and the number of years of optimal health 194. Eliminate health disparities among different segments of the population. 195. A community nurse is serving as a patients advocate if she tells a malnourished patient to go to a meal program at a local park. 196. If a patient isnt following his treatment plan, the nurse should first ask why. 197. Falls are the leading cause of injury in elderly people. 198. Primary prevention is true prevention. Examples are immunizations, weight control, and smoking cessation. 199. Secondary prevention is early detection. Examples include purified protein derivative (PPD), breast self-examination, testicular self-examination, and chest X-ray. 200. Tertiary prevention is treatment to prevent long-term complications. 201. A patient indicates that hes coming to terms with having a chronic disease when he says, Im never going to get any better. 202. On noticing religious artifacts and literature on a patients night stand, a culturally aware nurse would ask the patient the meaning of the items. 203. A Mexican patient may request the intervention of a curandero, or faith healer, who involves the family in healing the patient. 204. In an infant, the normal hemoglobin value is 12 g/dl. 205. The nitrogen balance estimates the difference between the intake and use of protein. 206. Most of the absorption of water occurs in the large intestine. 207. Most nutrients are absorbed in the small intestine. 208. When assessing a patients eating habits, the nurse should ask, What have you eaten in the last 24 hours? 209. A vegan diet should include an abundant supply of fiber. 210. A hypotonic enema softens the feces, distends the colon, and stimulates peristalsis. 211. First-morning urine provides the best sample to measure glucose, ketone, pH, and specific gravity values. 212. To induce sleep, the first step is to minimize environmental stimuli. 213. Before moving a patient, the nurse should assess the patients physical abilit ies and ability to understand instructions as well as the amount of strength required to move the patient.

163

214. To lose 1 lb (0.5 kg) in 1 week, the patient must decrease his weekly intake by 3,500 calories (approximately 500 calories daily). To lose 2 lb (1 kg) in 1 week, the patient must decrease his weekly caloric intake by 7,000 calories (approximately 1,000 calories daily). 215. To avoid shearing force injury, a patient who is completely immobile is lifted on a sheet. 216. To insert a catheter from the nose through the trachea for suction, the nurse should ask the patient to swallow. 217. Vitamin C is needed for collagen production. 218. Only the patient can describe his pain accurately. 219. Cutaneous stimulation creates the release of endorphins that block the transmission of pain stimuli. 220. Patient-controlled analgesia is a safe method to relieve acute pain caused by surgical incision, traumatic injury, labor and delivery, or cancer. 221. An Asian American or European American typically places distance between himself and others when communicating. 222. The patient who believes in a scientific, or biomedical, approach to health is likely to expect a drug, treatment, or surgery to cure illness. 223. Chronic illnesses occur in very young as well as middle-aged and very old people. 224. The trajectory framework for chronic illness states that preferences about daily life activities affect treatment decisions. 225. Exacerbations of chronic disease usually cause the patient to seek treatment and may lead to hospitalization. 226. School health programs provide cost-effective health care for low-income families and those who have no health insurance. 227. Collegiality is the promotion of collaboration, development, and interdependence among members of a profession. 228. A change agent is an individual who recognizes a need for change or is selected to make a change within an established entity, such as a hospital. 229. The patients bill of rights was introduced by the American Hospital Association. 230. Abandonment is premature termination of treatment without the patients permission and without appropriate relief of symptoms. 231. Values clarification is a process that individuals use to prioritize their personal values. 232. Distributive justice is a principle that promotes equal treatment for all. 233. Milk and milk products, poultry, grains, and fish are good sources of phosphate. 234. The best way to prevent falls at night in an oriented, but restless, elderly patient is to raise the side rails. 235. By the end of the orientation phase, the patient should begin to trust the nurse. 236. Falls in the elderly are likely to be caused by poor vision. 237. Barriers to communication include language deficits, sensory deficits, cognitive impairments, structural deficits, and paralysis. 238. The three elements that are necessary for a fire are heat, oxygen, and combustible material. 239. Sebaceous glands lubricate the skin. 240. To check for petechiae in a dark-skinned patient, the nurse should assess the oral mucosa. 241. To put on a sterile glove, the nurse should pick up the first glove at the folded border and adjust the fingers when both gloves are on. 242. To increase patient comfort, the nurse should let the alcohol dry before giving an intramuscular injection. 243. Treatment for a stage 1 ulcer on the heels includes heel protectors. 244. Seventh-Day Adventists are usually vegetarians. 245. Endorphins are morphine-like substances that produce a feeling of well-being. 246. Pain tolerance is the maximum amount and duration of pain that an individual is willing to endure.

164

You might also like